You are on page 1of 132

INSIGHTSIAS

IA SIMPLIFYING IAS EXAM PREPARATION

RTM COMPILATIONS
PRELIMS 2021

June 2021

www.insightsactivelearn.com | www.insightsonindia.com
Revision Through MCQs (RTM) Compilation (June 2021)

Telegram: https://t.me/insightsIAStips
2
Youtube: https://www.youtube.com/channel/UCpoccbCX9GEIwaiIe4HLjwA
Revision Through MCQs (RTM) Compilation (June 2021)

Table of Contents
RTM- REVISION THROUGH MCQS –1st -Jun-2021 ............................................................... 4
RTM- REVISION THROUGH MCQS –2nd -Jun-2021............................................................... 8
RTM- REVISION THROUGH MCQS –3rd -Jun-2021 ............................................................. 14
RTM- REVISION THROUGH MCQS –4th -Jun-2021 ............................................................. 19
RTM- REVISION THROUGH MCQS –5th -Jun-2021 ............................................................. 24
RTM- REVISION THROUGH MCQS – 7th -Jun-2021 ............................................................ 30
RTM- REVISION THROUGH MCQS – 8th -Jun-2021 ............................................................ 34
RTM- REVISION THROUGH MCQS – 9th -Jun-2021 ............................................................ 39
RTM- REVISION THROUGH MCQS – 10th -Jun-2021 .......................................................... 44
RTM- REVISION THROUGH MCQS – 11th -Jun-2021 .......................................................... 49
RTM- REVISION THROUGH MCQS – 12th -Jun-2021 .......................................................... 55
RTM- REVISION THROUGH MCQS – 14th -Jun-2021 .......................................................... 59
RTM- REVISION THROUGH MCQS – 15th -Jun-2021 .......................................................... 64
RTM- REVISION THROUGH MCQS – 16th -Jun-2021 .......................................................... 69
RTM- REVISION THROUGH MCQS – 17th -Jun-2021 .......................................................... 74
RTM- REVISION THROUGH MCQS – 18th -Jun-2021 .......................................................... 79
RTM- REVISION THROUGH MCQS – 19th -Jun-2021 .......................................................... 83
RTM- REVISION THROUGH MCQS – 21st -Jun-2021 ........................................................... 88
RTM- REVISION THROUGH MCQS – 22nd -Jun-2021 .......................................................... 93
RTM- REVISION THROUGH MCQS – 23rd-Jun-2021 ........................................................... 98
RTM- REVISION THROUGH MCQS – 24th -Jun-2021 ........................................................ 104
RTM- REVISION THROUGH MCQS – 24th -Jun-2021 ........................................................ 109
RTM- REVISION THROUGH MCQS – 26th -Jun-2021 ........................................................ 113
RTM- REVISION THROUGH MCQS – 28th -Jun-2021 ........................................................ 118
RTM- REVISION THROUGH MCQS – 29th -Jun-2021 ........................................................ 123
RTM- REVISION THROUGH MCQS – 30th -Jun-2021 ........................................................ 127

Telegram: https://t.me/insightsIAStips
3
Youtube: https://www.youtube.com/channel/UCpoccbCX9GEIwaiIe4HLjwA
Revision Through MCQs (RTM) Compilation (June 2021)

RTM- REVISION THROUGH MCQS –1st -Jun-2021


1. Consider the following statements regarding Sedition Law in India.
1. Section 124A of the Indian Penal Code (IPC), which deals with sedition, was drafted
by Thomas Babington Macaulay.
2. Mahatma Gandhi was never held under the sedition trial during freedom movement.
Which of the above statements is/are correct?
(a) 1 only
(b) 2 only
(c) Both 1 and 2
(d) Neither 1 nor 2
Ans: (a)
Explanation: Famous sedition trials during freedom movement:
• The first is the trial of Jogendra Chandra Bose in 1891. Bose, the editor of the
newspaper, Bangobasi, wrote an article criticizing the Age of Consent Bill for
posing a threat to the religion and for its coercive relationship with Indians.
• The other most well-known cases are the sedition trials of Bal Gangadhar Tilak
and the trial of Mahatma Gandhi in 1922. Mahatma Gandhi for his articles
published in Young India.

Refer: https://www.insightsonindia.com/2021/06/01/its-time-to-define-limits-of-sedition-
says-sc/

2. Which of the following can be considered as neglected tropical diseases?


1. Chagas disease
2. Hookworm infection
3. Leprosy
4. Lymphatic filariasis
Select the correct answer using the code below:
(a) 1 and 3 only
(b) 2, 3 and 4 only
(c) 2 and 4 only
(d) 1, 2, 3 and 4
Ans: (d)
Explanation:
• 13 neglected tropical diseases: ascariasis, Buruli ulcer, Chagas disease,
dracunculiasis, hookworm infection, human African trypanosomiasis,
Leishmaniasis, leprosy, lymphatic filariasis, onchocerciasis, schistosomiasis,
trachoma, and trichuriasis.

Refer: https://www.insightsonindia.com/2021/06/01/january-30-now-world-neglected-
tropical-diseases-day/

3. Bhasan Char, an Island recently in news is located in


(a) Arabian Sea
(b) Bay of Bengal
(c) South China Sea
(d) Sea of Japan
Ans: (b)
Explanation:

Telegram: https://t.me/insightsIAStips
4
Youtube: https://www.youtube.com/channel/UCpoccbCX9GEIwaiIe4HLjwA
Revision Through MCQs (RTM) Compilation (June 2021)

• Bhasan Char (Floating Island) also known as Char Piya or Thengar Char
Island, is an island in Hatiya, Bangladesh.
• The island was formed from a build-up of silt in the Bay of Bengal only 20 years
ago, and concerns have been consistently raised about Bhasan Char’s exposure
to extreme weather and distance from the mainland in emergencies.

Refer: https://www.insightsonindia.com/2021/06/01/thousands-of-rohingya-protest-at-
bhashan-char/

4. With reference to 5G mobile network, which of the following statements is/are correct?
1. 5G technology is meant to deliver higher multi-Gbps peak data speeds, ultra-high
latency, more reliability and massive network capacity.
2. 5G wireless devices will connect to the Internet and telephone network by radio
waves through a local antenna.
3. At present 5G technology is available only in US, China and some European
countries.
Select the correct answer using the code below:
(a) 1 and 2 only
(b) 1 and 3 only
(c) 2 only
(d) 2 and 3 only
Ans: (c)
Explanation:
• S1: 5G wireless technology is meant to deliver higher multi-Gbps peak data
speeds, ultra-low latency, more reliability, massive network capacity, increased
availability, and a more uniform user experience to more users. Higher
performance and improved efficiency empower new user experiences and
connects new industries.
• S2: Like its predecessors, 5G networks are cellular networks, in which the
service area is divided into small geographical areas called cells. All 5G wireless
devices in a cell are connected to the Internet and telephone
network by radio waves through a local antenna in the cell.
• S3: 5G has been deployed in 35+ countries and counting.

o
Refer: https://www.insightsonindia.com/2021/06/01/5g-technology/

5. With reference to Global Biodiversity, recently discovered ‘Litoria mira’ is a/an/the


(a) Bird
(b) Primate
(c) Reptile
(d) Amphibian

Telegram: https://t.me/insightsIAStips
5
Youtube: https://www.youtube.com/channel/UCpoccbCX9GEIwaiIe4HLjwA
Revision Through MCQs (RTM) Compilation (June 2021)

Ans: (d)
Explanation: Litoria mira:
• They are new frog species discovered recently in the rainforests of New Guinea.
They are chocolate coloured.
• The name is inspired by the Latin adjective mirum, which means surprised
or strange, stemming from the scientist’s surprise in discovering an undescribed
member of the predominately Australian Litoria genus of tree frogs.
• Litoria mira can be distinguished from all other Litoria by its unique
combination of moderately large size, webbing on hand, relatively short and
robust limbs, and small violet patch of skin on the edge of its eyes

Refer: Facts for Prelims: https://www.insightsonindia.com/2021/06/01/insights-daily-


current-affairs-pib-summary-1-june-2021/

6. Consider the following statements about Central Board of Direct Taxes (CBDT):
1. It is a statutory body established as per the Central Board of Revenue Act, 1963.
2. It is India’s official financial action task force unit.
3. It is administered by the Department of Financial Services under the Ministry of
Finance.
Which of the given above statements is/are correct?
(a) 1 and 2 only
(b) 1 and 3 only
(c) 2 and 3 only
(d) 1, 2 and 3
Ans: (a)
Explanation: About CBDT:
• It is a statutory body established as per the Central Board of Revenue Act,
1963.
• It is India’s official financial action task force unit.
• It is administered by the Department of Revenue under the Ministry of
Finance.

Refer: Facts for Prelims: https://www.insightsonindia.com/2021/06/01/insights-daily-


current-affairs-pib-summary-1-june-2021/

7. With reference to Asiatic Wild Dogs (Dhole), consider the following statements:
1. It is a highly social animal.
2. It is listed as Endangered on the IUCN Red List.
3. It is protected under Schedule 2 of the Wildlife Protection Act, 1972.
Which of the given above statements is/are correct?
(a) 1 and 2 only
(b) 3 only
(c) 2 and 3 only
(d) 1, 2 and 3
Ans: (d)
Explanation:
• S3: In India, the dhole is protected under Schedule 2 of the Wildlife
Protection Act, 1972. The creation of reserves under Project Tiger provided
some protection for dhole populations sympatric with tigers. In 2014, the Indian

Telegram: https://t.me/insightsIAStips
6
Youtube: https://www.youtube.com/channel/UCpoccbCX9GEIwaiIe4HLjwA
Revision Through MCQs (RTM) Compilation (June 2021)

government sanctioned its first dhole conservation breeding centre at the Indira
Gandhi Zoological Park (IGZP) in Visakhapatnam.
• S2: Endangered–IUCN.
• S1: The dhole is a highly social animal, living in large clans without rigid
dominance hierarchies and containing multiple breeding females.

Refer: https://www.insightsonindia.com/wp-content/uploads/2021/04/INSTA-PT-2021-
Exclusive-Environment.pdf

8. With reference to Indian Bullfrog, consider the following statements:


1. It is the largest frog found in the Indian Subcontinent.
2. It is listed as Vulnerable on the IUCN Red List.
3. It is protected under Schedule 2 of the Wildlife Protection Act, 1972.
Which of the given above statements is/are correct?
(a) 1 only
(b) 2 only
(c) 1 and 2 only
(d) 1, 2 and 3
Ans: (a)
Explanation:
• Scientific name: Hoplobatrachus tigerinus.
• IUCN status: Least Concern category.
• Habitats: South and South-East Asia.
• It is the largest frog found in the Indian Subcontinent.
• They often engage in cannibalism by feeding on smaller individuals of their own
kind and on other frogs.
• Its loud croaking call, attracts the opposite sex, but also predators.
• It is protected under Schedule IV of the Wildlife Protection Act of India,
1971.

Refer: https://www.insightsonindia.com/wp-content/uploads/2021/04/INSTA-PT-2021-
Exclusive-Environment.pdf

9. In which one of the following State is Valmiki Tiger Reserve (VTR) locted?
(a) Odisha
(b) Madhya Pradesh
(c) Bihar
(d) Chhattisgarh
Ans: (c)
Explanation:
• Valmiki Tiger Reserve forms the eastern most limit of the Himalayan Terai
forests in India, and is the only tiger reserve of Bihar.

Refer: https://www.insightsonindia.com/wp-content/uploads/2021/04/INSTA-PT-2021-
Exclusive-Environment.pdf

10. The ‘Golden Langur Conservation Breeding Programme’ is implemented by which one of
the following state?
(a) Sikkim
(b) West Bengal

Telegram: https://t.me/insightsIAStips
7
Youtube: https://www.youtube.com/channel/UCpoccbCX9GEIwaiIe4HLjwA
Revision Through MCQs (RTM) Compilation (June 2021)

(c) Assam
(d) Meghalaya
Ans: (c)
Explanation:
• Assam’s Golden Langur Conservation Breeding Programme has achieved its
maiden success with the birth of a female offspring at the Assam State Zoo,
widely known as Guwahati Zoo. With funds being provided by Central Zoo
Authority (CZA), the project for conservation was started in 2011-12.
• Habitat:
o Semi evergreen and mixed deciduous forests.
o Found in Small regions of western Assam and in the neighbouring
foothills of the black mountains of Bhutan.
• Protection status:
o Schedule Ispeciesin the Wildlife Protection Act (1972).
o CITES Appendix I.
o Endangeredin IUCN Red List.

Refer: https://www.insightsonindia.com/wp-content/uploads/2021/04/INSTA-PT-2021-
Exclusive-Environment.pdf

RTM- REVISION THROUGH MCQS –2nd -Jun-2021

11. Consider the following statements regarding Avian influenza or Bird flu.
1. It is a highly contagious viral disease caused by Influenza Type A viruses.
2. Avian influenza does not affect the egg production in poultry birds.
3. Bird flu can be spread through their droppings while flying.
Which of the above statements is/are correct?
(a) 1 only
(b) 2 and 3 only
(c) 1 and 3 only
(d) 1, 2 and 3
Ans: (c)
Explanation:
• What is avian influenza or bird flu?
o It is a highly contagious viral disease caused by Influenza Type A
viruses which generally affects poultry birds such as chickens and
turkeys. There are many strains of the virus – some of them are mild and
may merely cause a low egg production or other mild symptoms among
chickens, while others are severe and lethal.
• How does the bird flu spread?
o Wild aquatic birds such as ducks and geese are the natural reservoir of
Influenza A viruses and the central players in the ecology of these
viruses.
• Many birds carry the flu without developing sickness, and shed it in their
droppings. Since birds excrete even while flying, they provide “a nice aerosol of
influenza virus, shedding it all over the world”

Refer: https://www.insightsonindia.com/2021/06/02/h10n3-bird-flu-strain/

Telegram: https://t.me/insightsIAStips
8
Youtube: https://www.youtube.com/channel/UCpoccbCX9GEIwaiIe4HLjwA
Revision Through MCQs (RTM) Compilation (June 2021)

12. Consider the following statements about National Commission for Protection of Child
Rights (NCPCR):
1. It was set up under the Juvenile Justice (Care and Protection of Children) Act, 2015.
2. It works under the aegis of Ministry of Women and Child Development.
3. It is mandated to monitor and regulate in-country and inter-country adoptions.
Which of the given above statements is/are correct?
(a) 1 and 3 only
(b) 2 only
(c) 2 and 3 only
(d) 1, 2 and 3
Ans: (b)
Explanation: About NCPCR:
• S1: Set up in March 2007 under the Commission for Protection of Child
Rights Act, 2005.
• S: It works under the administrative control of the Ministry of Women & Child
Development.
• S3: The Commission’s Mandate is to ensure that all Laws, Policies, Programmes,
and Administrative Mechanisms are in consonance with the Child Rights
perspective as enshrined in the Constitution of India and also the UN
Convention on the Rights of the Child.

Refer: https://www.insightsonindia.com/2021/06/02/national-commission-for-protection-
of-child-rights-ncpcr-2/

13. Under the RTE Act, 2009, the National Commission for Protection of Child Rights
(NCPCR) can:
1. inquire into complaints about violation of the law.
2. summon an individual and demand evidence.
3. seek a magisterial enquiry.
4. file a writ petition in the High Court or Supreme Court.
5. recommend interim relief to those affected.
Select the correct answer using the code below:
(a) 1, 2 and 3 only
(b) 1, 2, 3 and 4 only
(c) 2 and 5 only
(d) 1, 2, 3, 4 and 5
Ans: (d)
Explanation:
• Under the RTE Act, 2009, the NCPCR can inquire into complaints about
violation of the law and can summon an individual, demand evidence, seek a
magisterial enquiry as well as file a writ petition in the High Court or Supreme
Court. It can also approach the government concerned for prosecution of the
offender and recommend interim relief to those affected.

Refer: https://www.insightsonindia.com/2021/06/02/national-commission-for-protection-
of-child-rights-ncpcr-2/

Telegram: https://t.me/insightsIAStips
9
Youtube: https://www.youtube.com/channel/UCpoccbCX9GEIwaiIe4HLjwA
Revision Through MCQs (RTM) Compilation (June 2021)

14. With reference to Maintenance and Welfare of Parents and Senior Citizens Act, 2007,
consider the following statements:
1. This Act makes it legally obliging for adult children and heirs to provide for parents
by way of a monthly allowance.
2. This Act provides an inexpensive and speedy procedure to claim monthly
maintenance for parents and senior citizens.
3. According to this Act, parents could mean biological but not adoptive or step-
parents.
Which of the given above statements is/are correct?
(a) 1 and 3 only
(b) 2 and 3 only
(c) 1 and 2 only
(d) 1, 2 and 3
Ans: (c)
Explanation:
• Overview of the Maintenance and Welfare of Parents and Senior Citizens
Act, 2007:
• This Act makes it legally obliging for adult children and heirs to provide for
parents by way of a monthly allowance.
• This Act provides an inexpensive and speedy procedure to claim monthly
maintenance for parents and senior citizens.
• According to this Act, parents could mean biological, adoptive or step-
parents.
• Under this Act, there are also provisions to protect the life and property of such
persons (elderly).

Refer: https://www.insightsonindia.com/2021/06/02/maintenance-and-welfare-of-
parents-and-senior-citizens-act-2007/

15. Consider the following statements about “17+1” initiative:


1. It is a China-led format founded in 2012 in Budapest.
2. It aims to expand cooperation between Beijing and the Central and Southeastern
Asia member countries.
Which of the given above statements is/are correct?
(a) 1 only
(b) 2 only
(c) Both 1 and 2
(d) Neither 1 nor 2
Ans: (a)
Explanation:
• The 17+1 initiative is a China-led format founded in 2012 in Budapest with an
aim to expand cooperation between Beijing and the Central and Eastern
European (CEE) member countries, with investments and trade for the
development of the CEE region.
• The framework also focuses on infrastructure projects such as bridges,
motorways, railway lines and modernisation of ports in the member states.

Telegram: https://t.me/insightsIAStips
10
Youtube: https://www.youtube.com/channel/UCpoccbCX9GEIwaiIe4HLjwA
Revision Through MCQs (RTM) Compilation (June 2021)


Refer: https://www.insightsonindia.com/2021/06/02/china-and-central-eastern-
european-cee-171-mechanism/

16. Consider the following statements about Securities and Exchange Board of India:
1. It is a regulatory body for securities and commodity market in India.
2. It works under the jurisdiction of Ministry of Finance, Government of India.
3. It issues Participatory Notes or P-Notes (PNs) to overseas investors.
Which of the given above statements is/are correct?
(a) 1 and 3 only
(b) 2 only
(c) 1, 2 and 3
(d) 1 and 2 only
Ans: (d)
Explanation:
• S3: P-notes are issued by registered foreign portfolio investors (FPIs) to
overseas investors who wish to be part of the Indian stock market without
registering themselves directly.
• S2 and 1: Sebi is the regulatory body for securities and commodity market in
India under the jurisdiction of Ministry of Finance , Government of India.

Refer: https://www.insightsonindia.com/2021/06/02/what-is-the-ipo-grey-market/

17. Consider the following statements:


As per the National Security Act, the grounds for preventive detention of a person
include:
1. acting in any manner prejudicial to the defence of India.
2. strong criticism against government policies and personalities.
3. depictions of an unresponsive or insensitive regime.
Which of the statements given above is/are correct?
(a) 1 and 3 only
(b) 1 only
(c) 2 and 3 only
(d) 1, 2 and 3
Ans: (b)
Explanation:
• As per the National Security Act, the grounds for preventive detention of a
person include:
o acting in any manner prejudicial to the defence of India, the relations of
India with foreign powers, or the security of India.

Telegram: https://t.me/insightsIAStips
11
Youtube: https://www.youtube.com/channel/UCpoccbCX9GEIwaiIe4HLjwA
Revision Through MCQs (RTM) Compilation (June 2021)

o regulating the continued presence of any foreigner in India or with a


view to making arrangements for his expulsion from India.
o preventing them from acting in any manner prejudicial to the security of
the State or from acting in any manner prejudicial to the maintenance of
public order or from acting in any manner prejudicial to the
maintenance of supplies and services essential to the community it is
necessary so to do.
• Sedition’ is an offence incorporated into the Indian Penal Code (IPC) in
1870. Section 124A of the IPC defines sedition and says:
o whoever by words either spoken or written or by signs or by visible
representation or otherwise brings or attempts to bring into hatred or
contempt, the government established by law; or
o whoever by the above means excites or attempts to excite disaffection
towards the government established by law, has committed the offence
of sedition.
• Under the present law, it offers scope to consider as seditious
o strong criticism against government policies and personalities
o slogans voicing disapprobation of leaders
o depictions of an unresponsive or insensitive regime

Refer: https://www.insightsonindia.com/2021/06/02/national-security-act-nsa-1980/

18. Consider the following statements about the United Nations Human Rights Council
(UNHRC):
1. It is a specialised agency of the United Nations.
2. It has more than 50 members elected for staggered three-year terms on a regional
group basis.
3. It passes binding resolutions on human rights issues through a periodic review of all
193 UN member states.
Which of the given above statements is/are not correct?
(a) 1 and 2 only
(b) 2 and 3 only
(c) 1 and 3 only
(d) 1, 2 and 3
Ans: (d)
Explanation: Here the directive word is not correct!!
• S1: It is not a specialised agency of the UN (list of UN Specialized Agencies)
• S2: The UNHRC has 47 members serving at any time with elections held to fill
up seats every year, based on allocations to regions across the world to ensure
geographical representation.
• S3: The UNHRC passes non-binding resolutions on human rights issues
through a periodic review of all 193 UN member states called the Universal
Periodic Review (UPR).
• Who are UN Special Rapporteurs?
o They are independent experts working on behalf of the United Nations.
They work on a country or a thematic mandate specified by the United
Nations Human Rights Council.

Telegram: https://t.me/insightsIAStips
12
Youtube: https://www.youtube.com/channel/UCpoccbCX9GEIwaiIe4HLjwA
Revision Through MCQs (RTM) Compilation (June 2021)

o They have mandates to report and advise on human rights (civil, cultural,
economic, political, and social) from a thematic or country-specific
perspective.
o Rapporteurs do not receive any financial compensation for their work
from the United Nations.

Refer: facts for prelims: https://www.insightsonindia.com/2021/06/02/insights-daily-


current-affairs-pib-summary-2-june-2021/

19. Consider the following statements regarding National Human Rights Commission.
1. It was established by a Government resolution, for the protection and promotion of
human rights.
2. It has its own investigating staff for investigation into complaints of human rights
violations.
3. It has no power to punish the violators of human rights.
Which of the above statements is/are correct?
(a) 1 and 2 only
(b) 1 and 3 only
(c) 2 and 3 only
(d) 1, 2 and 3
Ans: (c)
Explanation:
• NHRC, established in 1993, is an independent statutory body as per the
provisions of the Protection of Human Rights Act of 1993.
• The Commission has its own investigating staff headed by a Director General of
Police for investigation into complaints of human rights violations.
• While inquiring into complaints under the Act, the Commission shall have all the
powers of a civil court trying a suit under the Code of Civil Procedure, 1908.
• It has no power to punish the violators of human rights. It has no power to
award any monetary relief.

Refer: facts for prelims: https://www.insightsonindia.com/2021/06/02/insights-daily-


current-affairs-pib-summary-2-june-2021/

20. With reference to “Retrofit of Air-conditioning to improve Indoor Air Quality for Safety
and Efficiency” (RAISE) initiative, consider the following statements:
1. It is a joint initiative of Bureau of Energy Efficiency (BEE) and the U.S. Agency for
International Development’s (USAID) MAITREE Program.
2. It aims to focus on improving indoor air quality (IAQ), thermal comfort, and energy
efficiency (EE) in workspaces across the nation.
Which of the given above statements is/are correct?
(a) 1 only
(b) 2 only
(c) Both 1 and 2
(d) Neither 1 nor 2
Ans: (b)
Explanation:
• S1: It is a joint initiative of Energy Efficiency Services Limited (EESL) and
the S. Agency for International Development’s (USAID) MAITREE Program.

Telegram: https://t.me/insightsIAStips
13
Youtube: https://www.youtube.com/channel/UCpoccbCX9GEIwaiIe4HLjwA
Revision Through MCQs (RTM) Compilation (June 2021)

• S2: RAISE initiative can potentially alleviate the issue of bad air quality in
workspaces across the nation and pioneer ways to make them healthier and
greener.

Refer: https://www.insightsonindia.com/wp-content/uploads/2021/04/INSTA-PT-2021-
Exclusive-Environment.pdf

RTM- REVISION THROUGH MCQS –3rd -Jun-2021

21. Which of the following statements is true about the Model Tenancy Act (MTA) in India?
1. The new Act seeks to cover urban and as well as rural areas.
2. The new Act will not affect existing tenancies.
3. After enforcement of this Act, no person can let or take on rent any premises except
by an agreement in writing.
Select the correct answer using the code below:
(a) 1 and 2 only
(b) 2 and 3 only
(c) 1 and 3 only
(d) 1, 2 and 3
Ans: (d)
Explanation:
• S1: The Act seeks to cover urban and as well as rural areas.
• S2: The new Act will be applicable prospectively and will not affect existing
tenancies.
• S3: After enforcement of this Act, no person can let or take on rent any premises
except by an agreement in writing.

Refer: https://www.insightsonindia.com/2021/06/03/model-tenancy-act/

22. With reference to India’s decision to levy 2 per cent Digital Services Tax or DST on
revenues generated from digital services by non-resident entities and others, which of
the following statements is/are correct?
1. It is introduced as a part of the Income Tax Act.
2. The levy applied only on digital advertising services.
Select the correct answer using the code given below:
(a) 1 only
(b) 2 only
(c) Both 1 and 2
(d) Neither 1 nor 2
Ans: (d)
Explanation:
• India’s 2 per cent DST is levied on revenues generated from digital services
offered in India.
• S1: It is part of the Finance act 2020.
• S2: Applicability:
o India has expanded the scope of the equalisation levy over the last few
years, to tax non-resident digital entities.

Telegram: https://t.me/insightsIAStips
14
Youtube: https://www.youtube.com/channel/UCpoccbCX9GEIwaiIe4HLjwA
Revision Through MCQs (RTM) Compilation (June 2021)

o While the levy applied only to digital advertising services till 2019-
20 at the rate of 6 percent, the government in April last year widened the
scope to impose a 2 per cent tax on non-resident e-commerce players
with a turnover of Rs 2 crore.
o The scope was further widened in the Finance Act 2021-22 to cover e-
commerce supply or service when any activity takes place online.
o Since May 2021, this also includes any entity that systematically and
continuously does business with more than 3 lakh users in India.

Refer: https://www.insightsonindia.com/2021/06/03/digital-tax-in-india/

23. With reference to agriculture in India, how can the technique of `genome sequencing’,
often seen in the news, be used in the immediate future?
1. Genome sequencing can be used to identify genetic markers for disease resistance
and drought tolerance in various crop plants.
2. This technique helps in reducing the time required to develop new varieties of crop
plants.
3. It can be used to decipher the host-pathogen relationships in crops.
Select the correct answer using the code given below:
(a) 1 only
(b) 2 and 3 only
(c) 1 and 3 only
(d) 1, 2 and 3
Ans: (d)
Explanation:
• Justification: Genome sequencing is figuring out the order of DNA nucleotides,
or bases, in a genome—the order of As, Cs, Gs, and Ts that make up an
organism’s DNA.
• Justification: Statement 1: Currently available newborn screening (genome) for
childhood diseases allows detection of rare disorders that can be prevented or
better treated by early detection and intervention.
• https://www.scientificamerican.com/article/whole-genome-sequencing-predict-
disease/
• Statement 2: Naturally if the gene potential can be identified by screening and
sequencing, it will help in better genetic engineering.
• Statement 3: This can be reasoned logically based on the above.
• https://en.wikipedia.org/wiki/Whole_genome_sequencing

Refer: https://www.insightsonindia.com/2021/06/03/only-delta-is-a-variant-of-concern-
who/

24. Consider the following statements regarding United Nations General Assembly (UNGA):
1. It is the main deliberative, policymaking and representative organ of the United
Nations (UN).
2. It is the only UN body with universal representation.
3. Its president is elected every two year by assembly to serve a two-year term of office.
Which of the given above statements is/are correct?
(a) 1 and 2 only
(b) 2 and 3 only
(c) 1 and 3 only

Telegram: https://t.me/insightsIAStips
15
Youtube: https://www.youtube.com/channel/UCpoccbCX9GEIwaiIe4HLjwA
Revision Through MCQs (RTM) Compilation (June 2021)

(d) 1, 2 and 3
Ans: (a)
Explanation: About UNGA:
• The General Assembly is the main deliberative, policymaking and representative
organ of the UN.
• All 193 Member States of the UN are represented in the General Assembly,
making it the only UN body with universal representation.
• The President of the General Assembly is elected each year by assembly to serve
a one-year term of office.
• The presidency rotates annually between the five geographic groups: African,
Asia-Pacific, Eastern European, Latin American and Caribbean, and Western
European and other States.

Refer: https://www.insightsonindia.com/2021/06/03/unga-head/

25. Consider the following statements about Bureau of Indian Standards (BIS):
1. It is the only national body that frames standards in India.
2. It works under the aegis of Ministry of Commerce and Industry, Government of
India.
Select the correct answer using the code given below:
(a) 1 only
(b) 2 only
(c) Both 1 and 2
(d) Neither 1 nor 2
Ans: (a)
Explanation: About the Bureau of Indian Standards (BIS):
• BIS is the only national body that frames standards.
• It works under the aegis of Ministry of Consumer Affairs, Food & Public
Distribution, Government of India.
• BIS is responsible for the harmonious development of the activities of
standardization, marking and quality certification of goods and for matters
connected therewith or incidental thereto.

Refer: https://www.insightsonindia.com/2021/06/03/research-design-standards-
organization-rdso/

26. With reference to china’s ‘Artificial Sun’ experiment, consider the following statements:
1. The mission mimics the energy generation process of the sun.
2. This project is part of the International Thermonuclear Experimental Reactor (ITER)
facility.
3. As of now, china is the only country that has achieved high plasma temperatures.
Which of the given above statements is/are correct?
(a) 1 and 2 only
(b) 2 and 3 only
(c) 1 and 3 only
(d) 1, 2 and 3
Ans: (a)
Explanation:
• S1: The mission mimics the energy generation process of the sun.

Telegram: https://t.me/insightsIAStips
16
Youtube: https://www.youtube.com/channel/UCpoccbCX9GEIwaiIe4HLjwA
Revision Through MCQs (RTM) Compilation (June 2021)

o The reactor consists of an advanced nuclear fusion experimental research


device located in Hefei, China.
• S2: The EAST project is part of the International Thermonuclear
Experimental Reactor (ITER) facility, which will become the world’s largest
nuclear fusion reactor when it becomes operational in 2035.
o The ITER project includes the contributions of several countries,
including India, South Korea, Japan, Russia and the United States.
• S3: China is not the only country that has achieved high plasma
temperatures. In 2020, South Korea’s KSTAR reactor set a new record by
maintaining a plasma temperature of over 100 million degrees Celsius for 20
seconds.

Refer: https://www.insightsonindia.com/2021/06/03/chinese-artificial-sun-experimental-
fusion-reactor-sets-new-world-record/

27. The National Disaster Management Authority (NDMA) is headed by the:


(a) Prime Minister of India
(b) President of India
(c) Home Minister of India
(d) Environment Minister of India
Ans: (a)
Explanation:
• On March 24, 2020, the Centre, through the National Disaster Management
Authority (NDMA) headed by the Prime Minister, invoked the provisions of the
Act to streamline the management of the pandemic, empowering district
magistrates to take decisions and centralise other decisions on the supply of
oxygen and movement of vehicles.

Refer: https://www.insightsonindia.com/2021/06/03/dm-act-is-of-2005-vintage-was-first-
enforced-during-pandemic/

28. In which one of the following state is Sakteng wildlife sanctuary located?
(a) Arunachal Pradesh
(b) Manipur
(c) Meghalaya
(d) None of the above
Ans: (d)
Explanation:
• Sakteng is based in Eastern Bhutan, or Trashigang Dzongkhag (district) that
borders Arunachal Pradesh.
• It protects several endemic speciesincluding the eastern blue pine and the black-
rumped magpie.
• It was created in part to protect the migoi, a yeti-like cryptid whose existence has
not been scientifically confirmed, but in which the local population strongly
believes.

Telegram: https://t.me/insightsIAStips
17
Youtube: https://www.youtube.com/channel/UCpoccbCX9GEIwaiIe4HLjwA
Revision Through MCQs (RTM) Compilation (June 2021)


Refer: https://www.insightsonindia.com/wp-content/uploads/2021/04/INSTA-PT-2021-
Exclusive-Environment.pdf

29. Consider the following statements:


1. It is located in the Dibrugarh and Tinsukia districts of Assam.
2. It forms the largest stretch of lowland rainforest in the country.
3. It was declared as Elephant Reserve under Project Elephant.
The above given statements refers to which one of the following protected area?
(a) Dibru-Saikhowa National Park
(b) Orang National Park
(c) Dehing Patkai National Park
(d) Kaziranga National Park
Ans: (c)
Explanation:
• Assam government has decided to upgrade the Dehing Patkai Wildlife
Sanctuary into a national park.
• About Dehing Patkai Wildlife Sanctuary:
o Also known as the Jeypore Rainforest is a part of Dehing Patkai
Elephant Reserve .
o Located in Dibrugarh and Tinsukia districts, the 111.19 sq km Dehing
Patkai was declared a wildlife sanctuary in 2004.
o It is home to 47 mammal, 47 reptile, and 310 butterfly species.
o It forms the largest stretch of lowland rainforest in the country.

Refer: https://www.insightsonindia.com/wp-content/uploads/2021/04/INSTA-PT-2021-
Exclusive-Environment.pdf

30. Which of the following pairs is/are correctly matched?


1. Mansar lake : Haryana
2. Lonar lake : Maharashtra
3. Startsapuk Tso: Sikkim
Select the correct answer using the code below:
(a) 3 only
(b) 2 only
(c) 1 and 2 only
(d) 1, 2 and 3
Ans: (b)
Explanation:
• Startsapuk Tso: Ladakh

Telegram: https://t.me/insightsIAStips
18
Youtube: https://www.youtube.com/channel/UCpoccbCX9GEIwaiIe4HLjwA
Revision Through MCQs (RTM) Compilation (June 2021)

• Lonar lake: Maharashtra


• Mansar lake: Jammu

Refer: https://www.insightsonindia.com/wp-content/uploads/2021/04/INSTA-PT-2021-
Exclusive-Environment.pdf

RTM- REVISION THROUGH MCQS –4th -Jun-2021

31. Consider the following statements about Central Vigilance Commission:


1. It was set up by the Government in 1964 on the recommendations of the First
Administrative Reforms Commission.
2. It has the status of an autonomous body, free of control from any Ministry/
Department.
3. It submits its report to the President of India.
Which of the given above statements is/are correct?
(a) 3 only
(b) 2 and 3 only
(c) 1 and 2 only
(d) 1, 2 and 3
Ans: (b)
Explanation: About CVC:
• The CVC was set up by the Government in February, 1964 on the
recommendations of the Committee on Prevention of Corruption, headed by
Shri K. Santhanam.
• In 2003, the Parliament enacted CVC Act conferring statutory status on the
CVC.
• The CVC is not controlled by any Ministry/Department. It is an independent
body which is only responsible to the Parliament.
• It submits its report to the President of India.
• It exercises superintendence over the functioning of the Delhi Special Police
Establishment (CBI) insofar as it relates to the investigation of offences under
the Prevention of Corruption Act, 1988.

Refer: https://www.insightsonindia.com/2021/06/04/norms-for-employing-retired-
officials-defined/

32. Which of the following is/are the key features of the Mahatma Gandhi National Rural
Employment Guarantee Act?
1. The State Employment Agency after due verification will issue a Job Card
2. A minimum of 10 job seekers shall apply to sanction a new work under MGNREGA
3. At least one-half beneficiaries shall be women who have registered and requested
work under the scheme
Select the correct answer using the code below:
(a) 3 only
(b) 2 and 3 only
(c) 2 only
(d) 1 and 2 only
Ans: (c)

Telegram: https://t.me/insightsIAStips
19
Youtube: https://www.youtube.com/channel/UCpoccbCX9GEIwaiIe4HLjwA
Revision Through MCQs (RTM) Compilation (June 2021)

Explanation:
• S2: A minimum of 10 job seekers shall apply to sanction a new work under
MGNREGA
• S1: The Gram Panchayat after due verification will issue a Job Card. The Job
Card will bear the photograph of all adult members of the household willing to
work under MGNREGA and is free of cost
• S3: At least one-third beneficiaries shall be women who have registered and
requested work under the scheme.
• Refer: more salient features: https://vikaspedia.in/social-welfare/rural-poverty-
alleviation-1/schemes/faqs-on-mgnrega/mahatma-gandhi-national-rural-
employment-guarantee-act

Refer: https://www.insightsonindia.com/2021/06/04/caste-categories-for-nregs-pay/

33. The “Balfour Declaration” is sometimes mentioned in the news in the context of the
affairs of
(a) Afghanistan
(b) Palestine
(c) Columbia
(d) Yemen
Ans: (b)
Explanation:
• The Balfour Declaration was issued after Britain gained control with the aim of
establishing a home for the Jews in Palestine. However during that period the
Arabs were in majority in Palestine.

Refer: https://www.insightsonindia.com/2021/06/04/palestine-flays-indias-abstention-
from-vote/

34. Consider the following statements:


1. Venus is the second planet from the sun.
2. Venus is one of just two planets that rotate from east to west.
3. Recently, SpaceX announced ‘Davinci+’ and ‘Veritas’ mission to Venus.
Which of the given above statements is/are correct?
(a) 1 only
(b) 2 only
(c) 1 and 2 only
(d) 1, 2 and 3
Ans: (c)
Explanation:
• S1: Venus is the second planet from the sun and the hottest planet in the
solar system with a surface temperature of 500C – high enough to melt lead.
• S2: Venus is one of just two planets that rotate from east to west. Only Venus
and Uranus have this “backwards” rotation.
• S3: NASA has announced two new missions (‘Davinci+’ and ‘Veritas’) to Venus.

Refer: https://www.insightsonindia.com/2021/06/04/nasa-announces-two-new-missions-
to-venus/

35. With reference to historic missions to Venus, which of the following pairs is/are
correctly matched?

Telegram: https://t.me/insightsIAStips
20
Youtube: https://www.youtube.com/channel/UCpoccbCX9GEIwaiIe4HLjwA
Revision Through MCQs (RTM) Compilation (June 2021)

1. Venus Express – NASA mission


2. Magellan – European mission
3. Akatsuki– Japanese spacecraft
Select the correct answer using the code below:
(a) 1 only
(b) 2 and 3 only
(c) 3 only
(d) 1, 2 and 3
Ans: (c)
Explanation:
• Historic missions to Venus:
o Magellan – a Nasa mission that ended in 1994.
o Venus Express– A European mission- focused on atmospheric science.
o Akatsuki– Japanese spacecraft- focused on atmospheric science.
• Future missions:
o The European Space Agency (Esa) is evaluating a Venus mission, called
EnVision, alongside two astronomy proposals – Theseus and Spica. Other
concepts are also being proposed to Nasa.

Refer: https://www.insightsonindia.com/2021/06/04/nasa-announces-two-new-missions-
to-venus/

36. Consider the following statements:


1. Agricultural soils release nitrogen oxides into environment.
2. Cattle release ammonia into environment.
3. Poultry industry releases reactive nitrogen compounds into environment.
Which of the statements given above is/are correct?
(a) 1 and 3 only
(b) 2 and 3 only
(c) 2 only
(d) 1, 2 and 3
Ans: (d)
Explanation:
• Agricultural soils contributed to over 70% of N2O emissions from India in 2010,
followed by waste water (12%) and residential and commercial activities (6%).
Since 2002, N2O has replaced methane as the second largest Greenhouse Gas
(GHG) from Indian agriculture.
• Cattle account for 80% of the ammonia production, though their annual growth
rate is 1%, due to a stable population.
• The poultry industry, on the other hand, with an annual growth rate of 6%,
recorded an excretion of reactive nitrogen compounds of 0.415 tonnes in 2016.

Refer: https://www.insightsonindia.com/2021/06/04/eighth-global-nitrogen-conference/

37. Consider the following statements regarding the International Nitrogen Initiative
Conference:
1. It was set up in 2001 by a shared partnership between the International Geosphere-
Biosphere Program (IGBP) and the Intergovernmental Panel on Climate Change
(IPCC).
2. It aims to optimize nitrogen’s beneficial role in sustainable food production.

Telegram: https://t.me/insightsIAStips
21
Youtube: https://www.youtube.com/channel/UCpoccbCX9GEIwaiIe4HLjwA
Revision Through MCQs (RTM) Compilation (June 2021)

Which of the given above statements is/are correct?


(a) 1 only
(b) 2 only
(c) Both 1 and 2
(d) Neither 1 nor 2
Ans: (b)
Explanation:
• The International Nitrogen Initiative (INI) is an international program, set up
in 2003 under sponsorship of the Scientific Committee on Problems of the
Environment (SCOPE) and from the International Geosphere-Biosphere
Program (IGBP). The key aims of the INI are to:
o optimize nitrogen’s beneficial role in sustainable food production,
and
o minimize nitrogen’s negative effects on human health and the
environment resulting from food and energy production.
• The program is currently a sustained partner of Future Earth.

Refer: https://www.insightsonindia.com/2021/06/04/eighth-global-nitrogen-conference/

38. Consider the following statements:


1. Tarballs are dark-coloured, sticky balls of oil that form when crude oil floats on the
ocean surface.
2. Oilzapper technology for bio-remediation of oily sludge and oil spills was developed
by DRDO.
Which of the given above statements is/are correct?
(a) 1 only
(b) 2 only
(c) Both 1 and 2
(d) Neither 1 nor 2
Ans: (a)
Explanation:
• S1: Tarballs are dark-coloured, sticky balls of oil that form when crude oil
floats on the ocean surface. Tarballs are formed by weathering of crude oil in
marine environments. They are transported from the open sea to the shores by
sea currents and waves.
o Most of the times, the presence of several tarballs indicate an oil
spill. However, its annual occurrence on the west coast during the
monsoon has led marine biologists and experts to demand an
investigation in the matter.
• S2: Oilzapper technology
o Developed by TERI after seven years of research work and partly
supported by the DBT (Department of Biotechnology), Ministry of Science
and Technology, Government of India, the Oilzapper is essentially a
cocktail of five different bacterial strains that are immobilized and mixed
with a carrier material (powdered corncob).

Refer: https://www.insightsonindia.com/2021/06/04/sri-lanka-braces-for-oil-spill-from-
sinking-cargo-vessel/

Telegram: https://t.me/insightsIAStips
22
Youtube: https://www.youtube.com/channel/UCpoccbCX9GEIwaiIe4HLjwA
Revision Through MCQs (RTM) Compilation (June 2021)

39. The ‘Atlas Mountains’ are a series of mountain ranges which act as a divide between
which of the following regions?
(a) The Mozambique Channel and the Kalahandi desert
(b) The Red Sea and the Libyan desert
(c) The Mediterranean basin and the Sahara desert
(d) The Gulf of Sudra and the Katanga plateau
Ans: (c)
Explanation:
• Britannica: Atlas Mountains, series of mountain ranges in northwestern Africa,
running generally southwest to northeast to form the geologic backbone of the
countries of the Maghrib (the western region of the Arab world)—Morocco,
Algeria, and Tunisia. They extend from the Moroccan port of Agadir in the
southwest, to the Tunisian capital of Tunis in the northeast. Their thick rim
rises to form a high sill separating the Mediterranean basin to the north from the
Sahara to the south, thus constituting a barrier that hinders, without completely
preventing, communication between the two regions. Across, the mountains
filter both air masses and human migrations.


Refer: https://indianexpress.com/article/trending/trending-globally/sahara-snow-algeria-
saudi-arabia-7152475/

40. The term ‘polar wandering’, recently seen in the news, refers to which of the following?
(a) The ice loss from Greenland and Antarctica.
(b) The process of exploration of the polar regions of Earth.
(c) The migration of the magnetic poles over Earth's surface through geologic time.
(d) The changing pole star, from north to south direction.
Ans: (c)
Explanation:
• Polar wandering is the migration of the magnetic poles over Earth’s surface
through geologic time.
• It can be used, for example, to measure the degree to which Earth's magnetic
poles have been observed to move relative to the Earth's rotation axis.
• Also read: https://www.firstpost.com/science/melting-glaciers-due-to-climate-
change-caused-earths-axis-to-shift-since-mid-90s-9573721.html

Refer: https://www.britannica.com/science/polar-wandering

Telegram: https://t.me/insightsIAStips
23
Youtube: https://www.youtube.com/channel/UCpoccbCX9GEIwaiIe4HLjwA
Revision Through MCQs (RTM) Compilation (June 2021)

RTM- REVISION THROUGH MCQS –5th -Jun-2021

41. The “Sustainable Development Goals (SDG) India Index” is released by which of the
following?
(a) National Statistical Office (NSO)
(b) National Productivity Council (NPC)
(c) National Innovation Foundation (NIF)
(d) National Institution for Transforming India (NITI)
Ans: (d)
Explanation:
• The NITI Aayog released the Baseline Report of the Sustainable
Development Goals (SDG) India Index, which comprehensively documents the
progress made by India's States and Union Territories towards implementing the
2030 SDG targets.

Refer: https://www.insightsonindia.com/2021/06/05/what-is-the-sdg-india-index/

42. Consider the following statements about ‘Industrial Deep Decarbonization Initiative’
(IDDI):
1. It is an initiative of UNEP and International Transport Forum (ITF).
2. India has been a member of IDDI since 2008.
Which of the given above statements is/are correct?
(a) 1 only
(b) 2 only
(c) Both 1 and 2
(d) Neither 1 nor 2
Ans: (d)
Explanation: What is Industrial Deep Decarbonization Initiative (IDDI)?
• It is a global coalition of public and private organisations who are working to
stimulate demand for low carbon industrial materials.
• In collaboration with national governments, IDDI works to standardise carbon
assessments, establish ambitious public and private sector procurement targets,
incentivise investment into low-carbon product development and design industry
guidelines.
• Coordinated by United Nations Industrial Development Organization
(UNIDO).
• Members: The IDDI is co-led by the UK and India and current members include
Germany and Canada.

Refer: https://www.insightsonindia.com/2021/06/05/clean-energy-ministerials-cem-
industrial-deep-decarbonization-initiative-iddi/

43. The Clean Energy Ministerial (CEM), a high-level global forum to promote policies and
programs that advance the deployment of clean energy technology, emerged at:
(a) The Earth Summit on Sustainable Development 2002, Johannesburg
(b) The United Nations Framework Convention on Climate Change 2009, Copenhagen
(c) The United Nations Conference on Sustainable Development 2012, Rio de Janeiro
(d) The World Sustainable Development Summit 2016, New Delhi
Ans: (b)

Telegram: https://t.me/insightsIAStips
24
Youtube: https://www.youtube.com/channel/UCpoccbCX9GEIwaiIe4HLjwA
Revision Through MCQs (RTM) Compilation (June 2021)

Explanation: About Clean Energy Ministerial (CEM):


• Establishment:
• It was established in December 2009 at the UN’s Framework Convention on
Climate Change conference of parties in Copenhagen.
• CEM is a high-level global forum to promote policies and programs that advance
clean energy technology, to share lessons learned and best practices, and to
encourage the transition to a global clean energy economy.
• 29 countries are part of CEM including India.

Refer: https://www.insightsonindia.com/2021/06/05/clean-energy-ministerials-cem-
industrial-deep-decarbonization-initiative-iddi/

44. The ‘Five to 50’ service, sometimes seen in the news, is related to
(a) Hospitality Management and Tourism
(b) Waste Management
(c) Wireless Communication
(d) Solar Power Production
Ans: (c)
Explanation: About OneWeb’s LEO internet programme:
• Using LEO satellites OneWeb seeks to offer connectivity across the UK, Alaska,
Northern Europe, Greenland, the Arctic Seas and Canada. The company expects
the service to be switched on before the end of the year.
• It calls this programme the ‘Five to 50’ service of offering internet
connectivity to all regions north of 50 degrees latitude.

Refer: https://www.insightsonindia.com/2021/06/05/internet-through-leo-satellites/

45. “They were discovered in 1773 by a German pastor named Johann August Ephraim
Goeze. Three years later, the Italian clergyman and scientist Lazzaro Spallanzani
discovered that they had superpowers. He added water to sediment from a rain gutter,
and looked under a microscope. He found hundreds of little bear-shaped creatures
swimming around.”
The above paragraph refers to which one of the following micro-animals?
(a) Rotifer
(b) Loricifera
(c) Gastrotrich
(d) Tardigrade
Ans: (d)
Explanation:
• Micro-animals are animals so small that they can be visually observed only
under a microscope.
• Tardigrades are one of nature's smallest animals. They are never more than
1.5 mm long, and can only be seen with a microscope. They are commonly
known as "water bears".
• Tardigrades were discovered in 1773 by a German pastor named Johann
August Ephraim Goeze. Three years later, the Italian clergyman and scientist
Lazzaro Spallanzani discovered that they had superpowers.
• Spallanzani added water to sediment from a rain gutter, and looked under a
microscope. He found hundreds of little bear-shaped creatures swimming

Telegram: https://t.me/insightsIAStips
25
Youtube: https://www.youtube.com/channel/UCpoccbCX9GEIwaiIe4HLjwA
Revision Through MCQs (RTM) Compilation (June 2021)

around. In his book "Opuscoli di Fisica Animale, e Vegetabile", he named them


"il Tardigrado", meaning "slow-stepper", because they moved so slowly.

Refer: https://www.insightsonindia.com/2021/06/05/why-is-nasa-sending-water-bears-
baby-squid-to-the-international-space-station/

46. Consider the following statements about International Space Station (ISS).
1. It is a habitable artificial satellite in the low Earth orbit.
2. It is the largest artificial object in space.
3. It serves as a microgravity and space environment research laboratory.
Which of the above statements is/are correct?
(a) 1 and 2 only
(b) 2 only
(c) 1 and 3 only
(d) 1, 2 and 3
Ans: (d)
Explanation:
• The International Space Station (ISS) is a space station, or a habitable
artificial satellite, in low Earth orbit. The ISS is now the largest artificial body
in orbit.
o The ISS serves as a microgravity and space environment research
laboratory in which crew members conduct experiments in biology,
human biology, physics, astronomy, meteorology and other fields.
o The station is suited for the testing of spacecraft systems and equipment
required for missions to the Moon and Mars.
o The ISS programme is a joint project among five participating space
agencies: NASA, Roscosmos, JAXA, ESA, and CSA.
o The ownership and use of the space station is established by
intergovernmental treaties and agreements.
o The station is divided into two sections, the Russian Orbital Segment
(ROS) and the United States Orbital Segment (USOS), which is shared by
many nations

Refer: https://www.insightsonindia.com/2021/06/05/why-is-nasa-sending-water-bears-
baby-squid-to-the-international-space-station/

47. Consider the following statements:


1. Hail is produced by cumulonimbus clouds.
2. Cumulonimbus clouds are delicate, feathery clouds that are made mostly of ice
crystals.
Which of the given above statements is/are correct?
(a) 1 only
(b) 2 only
(c) Both 1 and 2
(d) Neither 1 nor 2
Ans: (a)
Explanation:
• S1: Hail is produced by cumulonimbus clouds, which are generally large and
dark and may cause thunder and lightning.

Telegram: https://t.me/insightsIAStips
26
Youtube: https://www.youtube.com/channel/UCpoccbCX9GEIwaiIe4HLjwA
Revision Through MCQs (RTM) Compilation (June 2021)

• S2: More commonly known as thunderclouds, cumulonimbus is the only cloud


type that can produce hail, thunder and lightning. The base of the cloud is
often flat, with a very dark wall-like feature hanging underneath, and may only
lie a few hundred feet above the Earth's surface.
• Cirrus clouds are delicate, feathery clouds that are made mostly of ice crystals.

Refer: https://www.insightsonindia.com/2021/06/05/anti-hail-gun-test-by-himachal-
pradesh/

48. Which of the following is/are the potential impacts associated with increasing Black
Carbon in the atmosphere?
1. It prevents clouds from being formed
2. It accelerates melting of snow and ice
3. It reduces sunlight
4. It affects the plant productivity
Select the correct answer using the code below:
(a) 1, 2 and 3 only
(b) 1, 3 and 4 only
(c) 2 and 4 only
(d) 1, 2, 3 and 4
Ans: (d)
Explanation:
• Black carbon, or soot, is part of fine particulate air pollution (PM2.5) and
contributes to climate change.
• Black carbon is formed by the incomplete combustion of fossil fuels, wood and
other fuels.


• Black carbon is a short-lived climate pollutant with a lifetime of only days to
weeks after release in the atmosphere. During this short period of time, black
carbon can have significant direct and indirect impacts on the climate, the
cryosphere (snow and ice), agriculture and human health.

Refer: https://www.insightsonindia.com/2021/06/05/strong-policies-on-black-carbon-
can-sharply-cut-glacier-melt-world-bank-study/

49. Consider the following statements:


1. As per the right to education (RTE) Act, to be eligible for appointment as a teacher in
a state, a person would be required to possess the minimum qualification laid down
by the concerned State council of Teacher education.

Telegram: https://t.me/insightsIAStips
27
Youtube: https://www.youtube.com/channel/UCpoccbCX9GEIwaiIe4HLjwA
Revision Through MCQs (RTM) Compilation (June 2021)

2. As per the RTE Act, for teaching primary classes, a candidate is required to pass a
Teacher Eligibility Test conducted in accordance with the National Council of
Teacher Education guidelines.
3. In India, more than 90 % of teacher education institutions are directly under the
State Governments.
Which of the statements given above is/are correct?
(a) 1 and 2
(b) 2 only
(c) 1 and 3
(d) 3 only
Ans: (b)
Explanation:
• S1 is incorrect and S2 is correct because under the Right of Children to Free
and Compulsory Education (RTE), Act 2009, National Council of Teacher
Education (NCTE), has laid down the minimum educational & professional
qualifications for a person to be eligible for an appointment as a teacher for
classes I-VIII, which are applicable to all schools imparting elementary
education, including the schools under the State Governments and to qualify
under a Teacher Eligibility Test (TET).
• Teachers Eligibility Test:
o Teachers Eligibility Test is the minimum qualification required in India
for a person to be eligible for appointment as a teacher for Classes I to
VIII.
o The test is mandatory for teaching jobs in Indian government
schools.
o The TET is conducted by both the central and state governments of
India.
o The test is conducted to meet the objectives of the Right of Children to
Free and Compulsory Education Act, 2009.
• Why in News?
o The Ministry of Education has extended the validity of the Teachers
Eligibility Test (TET) qualifying certificate from seven years to life with
retrospective effect from 2011.

Refer: facts for prelims: https://www.insightsonindia.com/2021/06/05/insights-daily-


current-affairs-pib-summary-5-june-2021/

50. Which one of the following has launched SAGE Portal?


(a) NITI Aayog
(b) Ministry of Micro, Small and Medium Enterprises (MSME)
(c) Ministry of Skill Development and Entrepreneurship (MSDE)
(d) None of the above
Ans: (d)
Explanation: SAGE Portal:
• The Minister of Social Justice and Empowerment, launched the SAGE
(Seniorcare Aging Growth Engine) initiative and SAGE portal to support
India’s elderly persons.
• The portal will be a “one-stop access” of elderly care products and services by
credible start-ups.

Telegram: https://t.me/insightsIAStips
28
Youtube: https://www.youtube.com/channel/UCpoccbCX9GEIwaiIe4HLjwA
Revision Through MCQs (RTM) Compilation (June 2021)

• The start-ups selected under SAGE will be those which will provide new
innovative products and services to elderly persons in various areas like health,
travel, finance, legal, housing, and food among others.
• The Ministry of Social Justice & Empowerment will act as a facilitator for
this scheme.
• A fund of uptoRs.1 crore as one-time equity will be granted to each selected
start-up.

Refer: facts for prelims: https://www.insightsonindia.com/2021/06/05/insights-daily-


current-affairs-pib-summary-5-june-2021/

Telegram: https://t.me/insightsIAStips
29
Youtube: https://www.youtube.com/channel/UCpoccbCX9GEIwaiIe4HLjwA
Revision Through MCQs (RTM) Compilation (June 2021)

RTM- REVISION THROUGH MCQS – 7th -Jun-2021


51. Consider the following statements:
1. The Constitution of India empowers the Parliament to enact a law to provide for
matters relating to citizenship.
2. The children of foreign diplomats posted in India can acquire Indian citizenship by
birth.
3. All persons acquiring citizenship of India must take an oath of allegiance before they
are registered as citizens of India.
Which of the given above statements is/are correct?
(a) 1 and 3 only
(b) 1 only
(c) 2 and 3 only
(d) 1, 2 and 3
Ans: (a)
Explanation:
• S2: The children of foreign diplomats posted in India and enemy aliens cannot
acquire Indian citizenship by birth.
• S1: The Constitution deals with the citizenship from Articles 5 to 11 under Part
II. It empowers the Parliament to enact a law to provide for such matters and
any other matter relating to citizenship.
• S3: All categories of persons acquiring citizenship of India must take an oath of
allegiance before they are registered as citizens of India.

Refer: https://www.insightsonindia.com/2021/06/07/npr-slips-valid-for-long-term-visas/

52. Consider the following statements regarding the “Performance Grading Index”:
1. It is a survey mechanism used to measure the level of student participation at
universities and colleges in India.
2. It is initiated by the Department of School Education and Literacy (DoSEL).
Which of the given above statements is/are correct?
(a) 1 only
(b) 2 only
(c) Both 1 and 2
(d) Neither 1 nor 2
Ans: (b)
Explanation:
• The Ministry of Education has released the Performance Grading Index for
2019-20.
o Performance Grading Index is initiated by the Department of School
Education and Literacy (DoSEL).
• PGI is a tool to grade all States and UTs on their performance across 70
indicators on school education.
• Objective: To encourage States & UTs to adopt best practices like online
recruitment and transfer of teachers, electronic attendance of students &
teachers.

Refer: https://www.insightsonindia.com/2021/06/07/education-index-ranking/

Telegram: https://t.me/insightsIAStips
30
Youtube: https://www.youtube.com/channel/UCpoccbCX9GEIwaiIe4HLjwA
Revision Through MCQs (RTM) Compilation (June 2021)

53. Consider the following statements:


1. In viral-vectored vaccines, a virus is used to carry the target antigen gene into
human cells.
2. Covaxin is an adenovirus viral vector vaccine for COVID-19 jointly developed by
Bharat Biotech and the Indian Council of Medical Research.
Which of the given above statements is/are correct?
(a) 1 only
(b) 2 only
(c) Both 1 and 2
(d) Neither 1 nor 2
Ans: (a)
Explanation:
• A vector, in infectious disease biology, is what works as a vehicle to transport
a disease-causing agent.
o In viral-vectored vaccines, a virus is used to carry the target antigen
gene into human cells.
o Covishield (the vaccine by Oxford University-AstraZeneca) is based
on the viral-vectored platform.
• Sputnik V is an adenovirus viral vector vaccine for COVID-19 developed
by the Gamaleya Research Institute of Epidemiology and Microbiology.
• Covaxin (jointly developed by Bharat Biotech and the Indian Council of
Medical Research), is an inactivated vaccine.

Refer: https://www.insightsonindia.com/2021/06/07/more-antibodies-produced-by-
covishield-than-covaxin-study/

54. With reference to Organisation for the Prohibition of Chemical Weapons (OPCW),
consider the following statements:
1. It is a Paris-based autonomous intergovernmental organization.
2. It prohibits the use of chemical weapons and requires their destruction.
3. It was awarded the 2013 Nobel Peace Prize "for its extensive efforts to eliminate
chemical weapons".
Which of the given above statements is/are correct?
(a) 1 only
(b) 2 and 3 only
(c) 1 and 2 only
(d) 1, 2 and 3
Ans: (b)
Explanation:
• The Nobel Peace Prize for 2013 has been awarded to the Organisation for the
Prohibition of Chemical Weapons (OPCW) “for its extensive efforts to eliminate
chemical weapons”.
• The Hague-based autonomous body, which works within the framework of the
United Nations, was established in 1997 by the Chemical Weapons Convention
to carry out its mandate.
• The organisation promotes and verifies the adherence to the Chemical Weapons
Convention, which prohibits the use of chemical weapons and requires their
destruction. Verification consists both of evaluation of declarations by member
states and onsite inspections.

Telegram: https://t.me/insightsIAStips
31
Youtube: https://www.youtube.com/channel/UCpoccbCX9GEIwaiIe4HLjwA
Revision Through MCQs (RTM) Compilation (June 2021)

Refer: https://www.insightsonindia.com/2021/06/07/syria-has-likely-used-chemical-
weapons-17-times-opcw/

55. The CEO Water Mandate is a special initiative of


(a) The UN Secretary-General and the UN Global Compact
(b) The UNEP Secretariat and the UN Global Compact
(c) The UNFCCC Secretariat and the UN Global Compact
(d) None of the above
Ans: (a)
Explanation:
• The CEO Water Mandate is a special initiative of the UN Secretary-General and
the UN Global Compact, implemented in partnership with the Pacific
Institute.

Refer: https://www.insightsonindia.com/2021/06/07/ntpc-joins-uns-ceo-water-mandate/

56. With reference to UN Global Compact initiative, consider the following statements:
1. It is a non-binding United Nations pact to encourage businesses and firms
worldwide to adopt sustainable and socially responsible policies.
2. It is the world’s largest corporate sustainability initiative.
Which of the given above statements is/are corcet?
(a) 1 only
(b) 2 only
(c) Both 1 and 2
(d) Neither 1 nor 2
Ans: (c)
Explanation:
• The UN Global Compact is the world’s largest corporate sustainability
initiative, with more than 12,000 corporate participants and stakeholders from
more than 140 countries.
• It is based on ten principles in the areas of human rights, labour standards, the
environment, and anti-corruption.
• It is a non-binding United Nations pact to encourage businesses and firms
worldwide to adopt sustainable and socially responsible policies, and to report
on their implementation.
• Launched in 2000.
• It is a principle-based framework for businesses, stating ten principles in the
areas of human rights, labor, the environment and anti-corruption.
• Under the Global Compact, companies are brought together with UN agencies,
labor groups and civil society.
• Cities can join the Global Compact through the Cities Programme.

Refer: https://www.insightsonindia.com/2021/06/07/ntpc-joins-uns-ceo-water-mandate/

57. The “Tiananmen square massacre” was in news recently, is mentioned in the news in
the context of the affairs of:
(a) Israel
(b) China
(c) Russia
(d) Germany

Telegram: https://t.me/insightsIAStips
32
Youtube: https://www.youtube.com/channel/UCpoccbCX9GEIwaiIe4HLjwA
Revision Through MCQs (RTM) Compilation (June 2021)

Ans: (b)
Explanation: Tiananmen square massacre:
• Context: 32nd anniversary of the event was observed recently.
• What happened?
• On June 4, 1989 Chinese troops cracked down on pro-democracy protesters
around Beijing’s Tiananmen Square. No official death toll has ever been released,
but rights groups estimate hundreds, if not thousands were killed.
• The protests were primarily student-led demonstrations calling for democracy,
free speech and a free press in China.

Refer: facts for prelims: https://www.insightsonindia.com/2021/06/07/insights-daily-


current-affairs-pib-summary-7-june-2021/

58. Consider the following statements:


1. Since 1974, World Environment Day has been celebrated every year on 5 June.
2. Every World Environment Day is hosted by a different country in which official
celebrations take place and this year's host is Pakistan.
Which of the given above statements is/are corcet?
(a) 1 only
(b) 2 only
(c) Both 1 and 2
(d) Neither 1 nor 2
Ans: (c)
Explanation:
• The United Nations Assembly established World Environment Day in 1972,
which was the first day of the Stockholm Conference on the human
environment.
• Since 1974, World Environment Day has been celebrated every year on 5 June,
engaging governments, businesses and citizens in an effort to address pressing
environmental issues.
• The theme of World Environment Day 2021 is ‘Reimagine. Recreate. Restore.’
The theme is based on the UN Decade on Ecosystem Restoration, which will
be launched by the United Nations this year.
• Pakistan will be the global host for 2021.

Refer: facts for prelims: https://www.insightsonindia.com/2021/06/07/insights-daily-


current-affairs-pib-summary-7-june-2021/

59. ‘YounTab scheme’ is sometimes mentioned in the news with reference to


(a) social networking
(b) online internship
(c) digital learning
(d) vocational training
Ans: (c)
Explanation:
• Ladakh Lt Governor has launched the YounTab scheme for students in the
Union Territory to encourage digital learning.
• Under the scheme, 12,300 tablets with pre-loaded online and offline content,
including textbooks, video lectures and online class applications, would be
distributed to government school students from Class 6th to 12th.

Telegram: https://t.me/insightsIAStips
33
Youtube: https://www.youtube.com/channel/UCpoccbCX9GEIwaiIe4HLjwA
Revision Through MCQs (RTM) Compilation (June 2021)

Refer: facts for prelims: https://www.insightsonindia.com/2021/06/07/insights-daily-


current-affairs-pib-summary-7-june-2021/

60. ‘I-Familia’ is a global database to identify missing persons, launched by:


(a) Europol
(b) INTERPOL
(c) Central Bureau of Investigation
(d) International Labour Organization
Ans: (b)
Explanation: I-Familia:
• It is a Global database to identify missing persons.
• Launched by Interpol.
• It can identify missing persons through family DNA and help the police solve
cold cases in member countries.

Refer: facts for prelims: https://www.insightsonindia.com/2021/06/07/insights-daily-


current-affairs-pib-summary-7-june-2021/

RTM- REVISION THROUGH MCQS – 8th -Jun-2021


61. Consider the following statements:
1. Juvenile Justice (Care and Protection of Children) Act, 2015 mandates setting up
Juvenile Justice Boards and Child Welfare Committees in every district.
2. As per the new amendments to JJ Act 2015, no new children’s home can be opened
without the sanction of the district magistrates (DMs).
Which of the given above statements is/are correct?
(a) 1 only
(b) 2 only
(c) Both 1 and 2
(d) Neither 1 nor 2
Ans: (c)
Explanation:
• S1: JJ Act, 2015 mandates setting up Juvenile Justice Boards and Child
Welfare Committees in every district. Both must have at least one-woman
member each.
• S2: Child Care Institutions can be government-run, government-aided,
privately run or run through government, private or foreign funding. These
institutions, while falling under the CWC and the state child protection units
had very little oversight and monitoring. Even to receive a license, after an
application was made, if the children’s home were to not receive a reply from the
government within 3 months’ time, it would be “deemed registered’’ for a period
of six months, even without government permission. The new amendment
ensures that this can no longer happen and that no new children’s home
can be opened without the sanction of the DM.
• DM’s are also responsible now for ensuring that CCIs falling in their district are
following all norms and procedures. During the NCPCR survey, for instance, it
had been CCI’s with large funds, including foreign funding, had been found
keeping children in unsanitary conditions in portacabins.

Refer: https://www.insightsonindia.com/2021/06/08/sc-urged-to-stop-illegal-adoption/

Telegram: https://t.me/insightsIAStips
34
Youtube: https://www.youtube.com/channel/UCpoccbCX9GEIwaiIe4HLjwA
Revision Through MCQs (RTM) Compilation (June 2021)

62. Consider the following statements about the Group of Seven (G7):
1. It is an intergovernmental organization made up of the world’s largest developed
economies.
2. It has no legislative or authoritative power to enforce the recommended policies and
plans it compiles.
Which of the given above statements is/are correct?
(a) 1 only
(b) 2 only
(c) Both 1 and 2
(d) Neither 1 nor 2
Ans: (c)
Explanation:
• The Group of Seven (G-7) is an intergovernmental organization that meets
periodically to address international economic and monetary issues.
• G-7 countries consist of the U.S., U.K., France, Germany, Italy, Canada, and
Japan.
• The G-7 was formerly referred to as the G-8 until Russia was suspended from
the group in 2014 after illegally annexing Crimea.
• The G-7 is not an official, formal entity and, therefore, has no legislative or
authoritative power to enforce the recommended policies and plans it
compiles.

Refer: https://www.insightsonindia.com/2021/06/08/g7-corporate-tax-deal/

63. With reference to Association of Southeast Asian Nations, consider the following
statements:
1. It is an economic union comprising 10 member states in Southeast Asia.
2. It launched the South Asian Free Trade Area in 2006.
3. Its secretariat is based in Indonesia, Jakarta.
Which of the given above statements is/are correct?
(a) 1 only
(b) 2 and 3 only
(c) 1 and 3 only
(d) 1, 2 and 3
Ans: (c)
Explanation:
• S1: The Association of Southeast Asian Nations is a regional organization
which was established to promote political and social stability amid rising
tensions among the Asia-Pacific’s post-colonial states.
• S2: The South Asian Association for Regional Cooperation (SAARC) launched
the South Asian Free Trade Area in 2006.
• S3: ASEAN Secretariat – Indonesia, Jakarta.

Refer: https://www.insightsonindia.com/2021/06/08/china-hosts-asean-foreign-
ministers/

64. Consider the following statements about National Asset Reconstruction Company Ltd
(NARCL):
1. It was established under the Insolvency and Bankruptcy Code, 2016.

Telegram: https://t.me/insightsIAStips
35
Youtube: https://www.youtube.com/channel/UCpoccbCX9GEIwaiIe4HLjwA
Revision Through MCQs (RTM) Compilation (June 2021)

2. It has regulatory oversight over the Insolvency Professionals and Insolvency


Professional Agencies in India.
Which of the given above statements is/are correct?
(a) 1 only
(b) 2 only
(c) Both 1 and 2
(d) Neither 1 nor 2
Ans: (d)
Explanation: What is NARCL?
• Setting up of NARCL, the proposed bad bank for taking over stressed assets
of lenders, was announced in the Budget for 2021-22.
• The plan is to create a bad bank to house bad loans of ₹500 crore and above, in
a structure that will contain an asset reconstruction company (ARC) and an
asset management company (AMC) to manage and recover dud assets.
• The new entity is being created in collaboration with both public and private
sector banks.
• How is NARCL different from existing ARCs? How can it operate
differently?
o The proposed bad bank will have a public sector character since the
idea is mooted by the government and majority ownership is likely to rest
with state-owned banks.
o At present, ARCs typically seek a steep discount on loans. With the
proposed bad bank being set up, the valuation issue is unlikely to come
up since this is a government initiative.
o The government-backed ARC will have deep pockets to buy out big
accounts and thus free up banks from carrying these accounts on their
books.

Refer: https://www.insightsonindia.com/2021/06/08/national-asset-reconstruction-
company-ltd-narcl/

65. With reference to “Sea Snot”, consider the following statements:


1. It is a collection of mucus-like organic matter found in the sea.
2. It is formed when algae are overloaded with nutrients.
Which of the given above statements is/are correct?
(a) 1 only
(b) 2 only
(c) Both 1 and 2
(d) Neither 1 nor 2
Ans: (c)
Explanation:
• Sea snot, sea saliva or marine mucilage is a collection of
mucus-like organic matter found in the sea.
• It is formed when algae are overloaded with nutrients as
a result of water pollution combined with the effects of
climate change.

Refer: https://www.insightsonindia.com/2021/06/08/what-is-the-sea-snot-outbreak-in-
turkey/

Telegram: https://t.me/insightsIAStips
36
Youtube: https://www.youtube.com/channel/UCpoccbCX9GEIwaiIe4HLjwA
Revision Through MCQs (RTM) Compilation (June 2021)

66. With reference to Sea of Marmara, consider the following statements:


1. It is bordered by Bulgaria, Romania and Turkey.
2. It connects the Black Sea to the Aegean Sea.
Which of the given above statements is/are correct?
(a) 1 only
(b) 2 only
(c) Both 1 and 2
(d) Neither 1 nor 2
Ans: (b)
Explanation:
• Sea of Marmara is the inland sea, entirely within the borders of Turkey.
• It connects the Black Sea to the Aegean Sea, thus separating Turkey's Asian
and European lands.
• The Bosphorus strait connects it to the Black Sea and the Dardanelles strait to
the Aegean Sea. The former also separates Istanbul into its Asian and European
sides.


Refer: https://www.insightsonindia.com/2021/06/08/what-is-the-sea-snot-outbreak-in-
turkey/

67. The Black Sea is bordered by which of the following countries?


1. Ukraine
2. Turkey
3. Greece
4. Georgia
Select the correct answer using the code below:
(a) 1, 2 and 4 only
(b) 1, 3 and 4 only
(c) 1, 2 and 3 only
(d) 2, 3 and 4 only
Ans: (a)
Explanation:

Telegram: https://t.me/insightsIAStips
37
Youtube: https://www.youtube.com/channel/UCpoccbCX9GEIwaiIe4HLjwA
Revision Through MCQs (RTM) Compilation (June 2021)


Refer: World Physical Map

68. In which one of the following states is Gumti Wildlife Sanctuary located?
(a) Tripura
(b) Nagaland
(c) Assam
(d) Arunachal Pradesh
Ans: (a)
Explanation:
• Gumti Wildlife Sanctuary is a Wildlife Sanctuary in Tripura.

Refer: https://en.wikipedia.org/wiki/Nature_reserve

69. Consider the following statements regarding ‘Earth Day’:


1. It is an initiative of the World Wide Fund for Nature (WWF).
2. It is a movement in which the participants switch off the lights for one hour on a
certain day every year.
3. It is a movement to raise the awareness about protection of the environment
Which of the above statements is/are correct?
(a) 1 and 2 only
(b) 1 and 3 only
(c) 3 only
(d) 1, 2 and 3
Ans: (c)
Explanation:
• Earth Day is an annual event celebrated on April 22. Worldwide, various
events are held to demonstrate support for environmental protection. First
celebrated in 1970, Earth Day events in more than 193 countries are now
coordinated globally by the Earth Day Network.
• Earth Hour is a worldwide movement organized by the World Wide Fund for
Nature (WWF). The event is held annually encouraging individuals,
communities, and businesses to turn off non-essential electric lights for one
hour, from 8:30 to 9:30 pm on a specific day towards the end of March, as a
symbol of commitment to the planet. It was started as a lights-off event in
Sydney, Australia, in 2007. Since then, it has grown to engage more than 7,000
cities and towns across 187 countries and territories.

Telegram: https://t.me/insightsIAStips
38
Youtube: https://www.youtube.com/channel/UCpoccbCX9GEIwaiIe4HLjwA
Revision Through MCQs (RTM) Compilation (June 2021)

Refer: https://www.earthday.org/ and https://www.earthhour.org/

70. Consider the following statements about Cartagena Protocol on Biosafety (CPB).
1. It is the first international regulatory framework for safe transfer, handling and use
of Living Modified Organisms (LMOs).
2. It was negotiated under the aegis of United Nations Framework Convention on
Climate Change (UNFCCC).
3. It establishes a Biosafety Clearing-House (BCH) to facilitate information exchange
with special attention to developing countries.
Which of the above statements is/are correct?
(a) 1 and 3 only
(b) 1 and 2 only
(c) 2 and 3 only
(d) 1, 2 and 3
Ans: (a)
Explanation:
• The Cartagena Protocol on Biosafety (CPB), the first international regulatory
framework for safe transfer, handling and use of Living Modified Organisms
(LMOs) was negotiated under the aegis of the Convention on Biological
Diversity (CBD). The protocol was adopted on 29th January 2000. The Protocol
entered into force on 11 September 2003.
• The objective of the Protocol is to contribute to ensuring an adequate level of
protection in the field of the safe transfer, handling and use of LMOs
resulting from modern biotechnology that may have adverse effects on the
conservation and sustainable use of biological diversity, taking also into account
risks to human health, and specifically focusing on transboundary movements.
• The Protocol establishes a Biosafety Clearing-House (BCH) to facilitate
information exchange, and contains provisions on capacity building and
financial resources, with special attention to developing countries and those
without domestic regulatory systems.

Refer: https://bch.cbd.int/protocol/

RTM- REVISION THROUGH MCQS – 9th -Jun-2021


71. Consider the following statements:
1. The president of India can summon a session of parliament at such a place as
he/she thinks fit.
2. The constitution of India provides for three sessions of the parliament in a year but
it is not mandatory to conduct all the sessions.
3. There is no minimum number of days that the parliament is required to meet in a
year.
Which of the above given statements is/are correct?
(a) 1 only
(b) 2 only
(c) 1 and 3 only
(d) 2 and 3 only
Ans: (c)
Explanation:

Telegram: https://t.me/insightsIAStips
39
Youtube: https://www.youtube.com/channel/UCpoccbCX9GEIwaiIe4HLjwA
Revision Through MCQs (RTM) Compilation (June 2021)

• S1 and S2: Article 85(1) of the Constitution empowers the President to summon
each House of Parliament to meet at such time and place as he thinks fit, but six
months shall not intervene between its last sitting in one Session and the date
appointed for its first sitting in the next Session.
• S3: This is correct, refer to the explanation above.
• http://legislative.gov.in/sites/default/files/coi-4March2016.pdf

Refer: https://www.insightsonindia.com/2021/06/09/monsoon-session-of-parliament-
likely-to-begin-in-july/

72. Consider the following statements:


1. The Government of India Act, 1935, contained a provision relating to the
summoning of the legislature in India.
2. The Constitution that came into force in 1950 required Parliament to be summoned
twice in a year, with a gap of no more than six months between its sessions.
Which of the given above statements is/are correct?
(a) 1 only
(b) 2 only
(c) Both 1 and 2
(d) Neither 1 nor 2
Ans: (c)
Explanation:
• S2: The Constitution that came into force in 1950 required Parliament to be
summoned twice in a year, with a gap of no more than six months between its
sessions. The First Amendment changed this in 1951, and Article 85 got its
present form.
o Article 85 only requires that there should not be a gap of more than six
months between two sessions of Parliament.
• S1: The Government of India Act, 1935, contained a provision relating to the
summoning of the legislature in India. It specified that the central legislature
had to be summoned to meet at least once a year, and that not more than 12
months could elapse between two sessions.

Refer: https://www.insightsonindia.com/2021/06/09/monsoon-session-of-parliament-
likely-to-begin-in-july/

73. With reference to Sixth Schedule of the Constitution of India, consider the following
statements:
1. It consists of provisions for the administration of tribal areas in Assam, Meghalaya,
Tripura and Mizoram
2. It seeks to safeguard the rights of tribal population through the formation of
Autonomous District Councils (ADC).
3. It provides for separate Regional Councils for each area constituted as an
autonomous region.
Which of the given above statements is/are correct?
(a) 1 and 2 only
(b) 2 and 3 only
(c) 1 and 3 only
(d) 1, 2 and 3
Ans: (d)

Telegram: https://t.me/insightsIAStips
40
Youtube: https://www.youtube.com/channel/UCpoccbCX9GEIwaiIe4HLjwA
Revision Through MCQs (RTM) Compilation (June 2021)

Explanation:
• S1: The Sixth Schedule consists of provisions for the administration of tribal
areas in Assam, Meghalaya, Tripura and Mizoram, according to Article 244 of
the Indian Constitution.
• S2: Passed by the Constituent Assembly in 1949, it seeks to safeguard the rights
of tribal population through the formation of Autonomous District Councils
(ADC). ADCs are bodies representing a district to which the Constitution has
given varying degrees of autonomy within the state legislature.
• S3: Along with ADCs, the Sixth Schedule also provides for separate Regional
Councils for each area constituted as an autonomous region. In all, there are 10
areas in the Northeast that are registered as autonomous districts – three in
Assam, Meghalaya and Mizoram and one in Tripura. These regions are named as
district council of (name of district) and regional council of (name of region).

Refer: https://www.insightsonindia.com/2021/06/09/rengma-nagas-demand-
autonomous-council/

74. Consider the following statements:


1. The United Nations Charter established six main organs of the United Nations,
including the United Nations Security Council (UNSC).
2. The UNSC is the only UN body with the authority to issue binding resolutions on
member states.
Which of the given above statements is/are correct?
(a) 1 only
(b) 2 only
(c) Both 1 and 2
(d) Neither 1 nor 2
Ans: (c)
Explanation:
• S1: The United Nations Charter established six main organs of the United
Nations, including the Security Council. It gives primary responsibility for
maintaining international peace and security to the Security Council, which may
meet whenever peace is threatened.
• According to the Charter, the United Nations has four purposes:
o to maintain international peace and security;
o to develop friendly relations among nations;
o to cooperate in solving international problems and in promoting respect
for human rights;
o and to be a centre for harmonizing the actions of nations.
• S2: All members of the United Nations agree to accept and carry out the
decisions of the Security Council. While other organs of the United Nations make
recommendations to member states, only the Security Council has the power
to make decisions that member states are then obligated to implement
under the Charter.

Refer: https://www.insightsonindia.com/2021/06/09/un-security-council-endorses-
secretary-general-guterres-for-second-term/

Telegram: https://t.me/insightsIAStips
41
Youtube: https://www.youtube.com/channel/UCpoccbCX9GEIwaiIe4HLjwA
Revision Through MCQs (RTM) Compilation (June 2021)

75. Consider the following statements regarding BRICS grouping.


1. Not all BRICS members are the members of G20.
2. Since 2009, the BRICS nations have met annually at formal summits.
3. New Development Bank (NDB) is a multilateral development bank operated by the
BRICS countries with each country contributing equally to its equity.
Which of the above statements is/are correct?
(a) 1 and 2 only
(b) 2 and 3 only
(c) 1 and 3 only
(d) 2 only
Ans: (b)
Explanation:
• BRICS is the acronym coined for an association of five major emerging national
economies: Brazil, Russia, India, China and South Africa.
• The BRICS members are known for their significant influence on regional affairs;
all are members of G20. Since 2009, the BRICS nations have met annually at
formal summits. The 12th BRICS summit was hosted virtually by Russian
President Vladimir Putin.
• The main achievement of BRICS is the New Development Bank, with each
country contributing equally to its equity.

Refer: https://www.insightsonindia.com/2021/06/09/brics-opposes-exceptionalism-
china/

76. The “Operation Pangea XIV” was in news recently, is conducted by:
(a) Wildlife Crime Control Bureau
(b) National Tiger Conservation Authority
(c) Narcotics Control Bureau
(d) None of the above
Ans: (d)
Explanation:
• A record number of fake online pharmacies have been shut down under
Operation Pangea XIV targeting the sale of counterfeit and illicit medicines and
medical products.
• The operation coordinated by INTERPOL involved police, customs and health
regulatory authorities from 92 countries.

Refer: facts for prelims: Facts for Prelims:


https://www.insightsonindia.com/2021/06/09/insights-daily-current-affairs-pib-
summary-9-june-2021/

77. Consider the following pairs:


GI product State
1. Zardalu West Bengal
2. Chak-Hao Manipur
3. Arumbavur Wood Kerala
Which of the above pairs is/are correctly matched?
(a) 1 and 2 only
(b) 2 only
(c) 2 and 3 only

Telegram: https://t.me/insightsIAStips
42
Youtube: https://www.youtube.com/channel/UCpoccbCX9GEIwaiIe4HLjwA
Revision Through MCQs (RTM) Compilation (June 2021)

(d) 3 only
Ans: (b)
Explanation:
• Sixteen varieties of mangoes including three GI certified varieties exported to
Bahrain from West Bengal & Bihar.
o These include GI certified Khirsapati & Lakshmanbhog (West Bengal),
Zardalu (Bihar).
• GI Tags in India 2020

State Product

Jammu & Kashmir Saffron (Mongra, Lachha, Guchhi)


Tamil Nadu Kovilpatti Kadalai Mittai
Tamil Nadu Thanjavur Pith Works
Tamil Nadu Arumbavur Wood
Carvings
Uttar Pradesh Gorakhpur Terracotta
Manipur Chak-Hao
Telangana Telia Rumal
Jharkhand Sohrai – Khovar Painting
Refer: Refer: facts for prelims: Facts for Prelims:
https://www.insightsonindia.com/2021/06/09/insights-daily-current-affairs-pib-
summary-9-june-2021/

78. Consider the following statements about Geographical Indications (GI) tags:
1. Geographical Indications are part of the intellectual property rights that comes
under the Paris Convention for the Protection of Industrial Property.
2. In India, Geographical Indications registration is administered by the Geographical
Indications of Goods (Registration and Protection) Act of 1999.
3. The registration of a geographical indication is valid for a period of 20 years.
Which of the given above statements is/are correct?
(a) 1 and 2 only
(b) 2 and 3 only
(c) 1 and 3 only
(d) 1, 2 and 3
Ans: (a)
Explanation:
• A GI is primarily an agricultural, natural or a manufactured product
(handicrafts and industrial goods) originating from a definite geographical
territory.
• S1: Geographical Indications are part of the intellectual property rights that
comes under the Paris Convention for the Protection of Industrial Property.
• S2: In India, Geographical Indications registration is administered by the
Geographical Indications of Goods (Registration and Protection) Act of 1999.
• S3: The registration of a geographical indication is valid for a period of 10
years.

Refer: Facts for Prelims: https://www.insightsonindia.com/2021/06/09/insights-daily-


current-affairs-pib-summary-9-june-2021/

Telegram: https://t.me/insightsIAStips
43
Youtube: https://www.youtube.com/channel/UCpoccbCX9GEIwaiIe4HLjwA
Revision Through MCQs (RTM) Compilation (June 2021)

79. Consider the following statements regarding Energy Efficiency Services Limited (EESL):
1. It was formed under India's Ministry of New and Renewable Energy to facilitate
energy efficiency projects.
2. It is the world's largest public Energy Service Company (ESCO).
Which of the given above statements is/are correct?
(a) 1 only
(b) 2 only
(c) Both 1 and 2
(d) Neither 1 nor 2
Ans: (b)
Explanation:
• S2: Energy Efficiency Services Limited (EESL) is an energy service company
(ESCO) of the Government of India and is the world's largest public ESCO.
• S1: EESL was formed under India's Ministry of Power to facilitate energy
efficiency projects.
• CESL and Ladakh sign an MoU to make the Union Territory carbon neutral:
o Convergence Energy Services Limited (CESL) has signed a
Memorandum of Understanding (MoU) with the Administration of Union
Territory (UT) of Ladakh, to make it a clean and green UT.
o CESL is a wholly owned subsidiary of Energy Efficiency Services
Limited (EESL) under Ministry of Power.

Refer: Facts for Prelims: https://www.insightsonindia.com/2021/06/09/insights-daily-


current-affairs-pib-summary-9-june-2021/

80. “Surakshit Hum Surakshit Tum Abhiyaan” has been launched by


(a) NITI Aayog
(b) Ministry of Railways
(c) Ministry of Road Transport and Highways
(d) Ministry of Health and Family Welfare
Ans: (a)
Explanation:
• NITI Aayog & Piramal Foundation have launched Surakshit Hum Surakshit
Tum Abhiyaan in 112 Aspirational Districts.
• Campaign Will Provide Covid Home-Care Support to 20 Lakh Citizens.

Refer: Facts for Prelims: https://www.insightsonindia.com/2021/06/09/insights-daily-


current-affairs-pib-summary-9-june-2021/

RTM- REVISION THROUGH MCQS – 10th -Jun-2021


81. Consider the following statements:
1. The Election Commission of India (ECI) is an autonomous constitutional authority
responsible for administering Union and State election processes in India.
2. The Constitution grants the ECI with the power of direction, superintendence, and
control of elections to urban bodies.
3. The tenure of office and the conditions of service of all the Election commissioners is
determined by the Parliament.
Which of the given above statements is/are correct?
(a) 1 only

Telegram: https://t.me/insightsIAStips
44
Youtube: https://www.youtube.com/channel/UCpoccbCX9GEIwaiIe4HLjwA
Revision Through MCQs (RTM) Compilation (June 2021)

(b) 1 and 3 only


(c) 2 and 3 only
(d) 1, 2 and 3
Ans: (a)
Explanation:
• S1: The Election Commission of India is an autonomous constitutional
authority responsible for administering Union and State election processes in
India.
• S2: The Constitution grants the ECI with the power of direction,
superintendence, and control of elections to Parliament, state legislatures, the
office of president of India and the office of vice-president of India. The ECI does
not deal with the elections to the urban bodies such as Municipalities and
Panchayats in the states and hence, a separate State Election Commission.
• S3: The President appoints the Chief Election Commissioner and other election
commissioners. The tenure of office and the conditions of service of all the
commissioners is determined by the President.

Refer: https://www.insightsonindia.com/2021/06/10/election-commissioner/

82. Consider the following statements:


1. In India, the minimum support price (MSP) is fixed twice a year on the
recommendations of the Cabinet Committee on Economic Affairs (CCEA).
2. The Cabinet Committee on Economic Affairs (CCEA) headed by Finance Minister of
India.
Which of the given above statements is/are correct?
(a) 1 only
(b) 2 only
(c) Both 1 and 2
(d) Neither 1 nor 2
Ans: (d)
Explanation:
• S1: The MSP is fixed twice a year on the recommendations of the Commission
for Agricultural Costs and Prices (CACP).
• S2: The Cabinet Committee on Economic Affairs (CCEA) headed by Prime
minster.

Refer: https://www.insightsonindia.com/2021/06/10/centre-announces-hike-in-msp-for-
paddy-pulses-oilseeds/

83. Consider the following:


1. Areca nut
2. Barley
3. Coffee
4. Finger millet
5. Groundnut
6. Sesamum
7. Turmeric
The Cabinet Committee on Economic Affairs has announced the Minimum Support
Price for which of the above?
(a) 1, 2, 3 and 7 only

Telegram: https://t.me/insightsIAStips
45
Youtube: https://www.youtube.com/channel/UCpoccbCX9GEIwaiIe4HLjwA
Revision Through MCQs (RTM) Compilation (June 2021)

(b) 2, 4, 5 and 6 only


(c) 1, 3, 4, 5 and 6 only
(d) 1, 2, 3, 4, 5 and 7
Ans: (b)
Explanation:
• The Centre currently fixes MSPs for 23 farm commodities — 7 cereals
(paddy, wheat, maize, bajra, jowar, ragi and barley), 5 pulses (chana, arhar/tur,
urad, moong and masur), 7 oilseeds (rapeseed-mustard, groundnut, soyabean,
sunflower, sesamum, safflower and nigerseed) and 4 commercial crops (cotton,
sugarcane, copra and raw jute).

Refer: https://www.insightsonindia.com/2021/06/10/centre-announces-hike-in-msp-for-
paddy-pulses-oilseeds/

84. With reference to Republic of Ecuador, consider the following statements:


1. It is a country in northeastern South America.
2. It shares borders with Colombia and Peru only.
3. It experiences little variation in daylight hours during the course of a year.
Which of the given above statements is/are correct?
(a) 1 and 2 only
(b) 2 and 3 only
(c) 1 and 3 only
(d) 1, 2 and 3
Ans: (b)
Explanation:
• S1 and S2: It is a country in northwestern South America, bordered by
Colombia on the north, Peru on the east and south, and the Pacific Ocean on the
west. It is the westernmost country that lies on the equator.
• S3: Because of its location at the equator, Ecuador experiences little variation in
daylight hours during the course of a year. Both sunrise and sunset occur each
day at the two six o'clock hours.


Refer: https://www.insightsonindia.com/2021/06/10/legalisation-of-bitcoin-in-el-
salvador/

85. Consider the following statements:


1. Addu Atoll is the southernmost atoll of the Maldives.
2. Maldives is situated in the Indian Ocean.
Which of the given above statements is/are correct?
(a) 1 only
(b) 2 only
(c) Both 1 and 2

Telegram: https://t.me/insightsIAStips
46
Youtube: https://www.youtube.com/channel/UCpoccbCX9GEIwaiIe4HLjwA
Revision Through MCQs (RTM) Compilation (June 2021)

(d) Neither 1 nor 2


Ans: (c)
Explanation:
• S1: Addu Atoll is the southernmost atoll of the Maldives.
• S2: Maldives is a small archipelagic state in South Asia situated in the Indian
Ocean.
• Apart from its strategic location in the Indian Ocean, Addu is the second largest
city in the archipelago, home to over 30,000 people.


Refer: Facts for Prelims: https://www.insightsonindia.com/2021/06/10/insights-daily-
current-affairs-pib-summary-10-june-2021/

86. Consider the following statements about Argostemma quarantena:


1. It is a new plant species belonging to the tea family.
2. It was recently spotted in Cardamom Hills in Kerala.
Which of the given above statements is/are correct?
(a) 1 only
(b) 2 only
(c) Both 1 and 2
(d) Neither 1 nor 2
Ans: (d)
Explanation:
• Argostemma quarantena:
• It is a new plant species belonging to the coffee family.
• It was recently spotted in Wagamon hills in Kerala.
• It has been named Argostemma quarantena to commemorate the millions who
have died in the pandemic.

Refer: Facts for Prelims: https://www.insightsonindia.com/2021/06/10/insights-daily-


current-affairs-pib-summary-10-june-2021/

87. The “Nagorno-Karabakh” conflict was in news recently, is an ethnic and territorial
conflict between which of the following?
(a) Israel and Palestine
(b) Syria and Israel
(c) China and Mongolia
(d) Armenia and Azerbaijan
Ans: (d)
Explanation:

Telegram: https://t.me/insightsIAStips
47
Youtube: https://www.youtube.com/channel/UCpoccbCX9GEIwaiIe4HLjwA
Revision Through MCQs (RTM) Compilation (June 2021)

• Nagorno-Karabakh is part of Azerbaijan, but its population is majority


Armenian. As the Soviet Union saw increasing tensions in its constituent
republics in the 1980s, Nagorno-Karabakh voted to become part of Armenia –
sparking a war which stopped with a ceasefire in 1994.
• Since then, Nagorno-Karabakh has remained part of Azerbaijan but is controlled
by separatist ethnic Armenians backed by the Armenian government.

Refer: Facts for Prelims: https://www.insightsonindia.com/2021/06/10/insights-daily-


current-affairs-pib-summary-10-june-2021/

88. With reference to India’s Dehing Patkai National Park, consider the following
statements:
1. It is a dipterocarp-dominated lowland rainforest.
2. It is an ideal habitat for lion-tailed macaque.
3. It is a World Heritage Site.
Which of the given above statements is/are correct?
(a) 1 only
(b) 2 only
(c) 1 and 2 only
(d) 1, 2 and 3
Ans: (a)
Explanation:
• S1: It is located in the Dehing patkai landscape which is a dipterocarp-
dominated lowland rainforest.
• S2: The lion-tailed macaque or the wanderoo, is an Old World monkey endemic
to the Western Ghats of South India.
• S3: National Paks in Assam: Kaziranga, Manas, Nameri, Orang, Dibru-
Saikhowa and Raimona National Park. Kaziranga and Manas are UNESCO
World Heritage Sites. They are also tiger reserves along with Nameri and Orang.

Refer: Facts for Prelims: https://www.insightsonindia.com/2021/06/10/insights-daily-


current-affairs-pib-summary-10-june-2021/

89. Consider the following statements with reference to Kaziranga National Park and Tiger
Reserve:
1. It is the home of the world's most one-horned rhinos.
2. It is home to the highest density of tigers in Assam.
3. It is recognized as an Important Bird Area by International Union for Conservation of
Nature (IUCN).
Which of the given above statements is/are correct?
(a) 1 and 2 only
(b) 1 only
(c) 2 and 3 only
(d) 1, 2 and 3
Ans: (b)
Explanation:
• S1: The sanctuary, which hosts two-thirds of the world’s great one-horned
rhinoceroses, is a World Heritage Site.

Telegram: https://t.me/insightsIAStips
48
Youtube: https://www.youtube.com/channel/UCpoccbCX9GEIwaiIe4HLjwA
Revision Through MCQs (RTM) Compilation (June 2021)

• S2: it is home to the highest density of tigers among protected areas in the world
and was declared a Tiger Reserve in 2006 (now the highest tiger density is in
Orang National Park, Assam). Also read
• S3: It is recognized as an Important Bird Area by BirdLife International.

Refer: Facts for Prelims: https://www.insightsonindia.com/2021/06/10/insights-daily-


current-affairs-pib-summary-10-june-2021/

90. In which one of the following States is Raimona National Park located?
(a) Arunachal Pradesh
(b) Manipur
(c) Meghalaya
(d) Assam
Ans: (d)
Explanation:
• Raimona National Park is a national park in Assam, India, located in
Gossaigaon subdivision of Kokrajhar district.
• It was declared to be National Park on 5 June 2021 by the announcement of
Assam's Chief Minister Himanta Biswa Sarma on the occasion of World
Environment Day at Gandhi Mandap, Guwahati.

Refer: Facts for Prelims: https://www.insightsonindia.com/2021/06/10/insights-daily-


current-affairs-pib-summary-10-june-2021/

RTM- REVISION THROUGH MCQS – 11th -Jun-2021

91. A new “Atlantic Charter” is sometimes mentioned in the news in the context of the
affairs of the:
(a) Israel and Palestine
(b) European Union (EU) and United Kingdom
(c) United States and the United Kingdom
(d) Armenia and Azerbaijan
Ans: (c)
Explanation:
• President Joe Biden and British Prime Minister Boris Johnson recently inspected
documents related to the Atlantic Charter, a declaration signed by British
Prime Minister Winston Churchill and U.S. President Franklin D. Roosevelt in
August 1941.
• The two leaders plan to sign what they’re calling a new Atlantic Charter,
pledging to “defend the principles, values, and institutions of democracy and
open societies.”
o The Atlantic Charter was a joint declaration issued during World War II
(1939-45) by the United States and Great Britain that set out a vision for
the postwar world.

Refer: https://www.insightsonindia.com/2021/06/11/atlantic-charter/

Telegram: https://t.me/insightsIAStips
49
Youtube: https://www.youtube.com/channel/UCpoccbCX9GEIwaiIe4HLjwA
Revision Through MCQs (RTM) Compilation (June 2021)

92. Consider the following statements regarding Parliamentary privileges:


1. The parliamentary privileges do not extend to the president of India.
2. A member of the parliament cannot be arrested during the session of Parliament if
he/she is booked in civil case.
3. Courts can inquire into the validity of any proceedings in Parliament on the ground
of an alleged irregularity of procedure.
Which of the given above statements is/are correct?
(a) 1 and 2 only
(b) 2 and 3 only
(c) 1 and 3 only
(d) 1, 2 and 3
Ans: (a)
Explanation:
• S1: The parliamentary privileges do not extend to the president who is also an
integral part of the Parliament.
• S2: Members cannot be arrested during the session of Parliament and 40 days
before the beginning and 40 days after the end of a session. This privilege is
available only in civil cases and not in criminal cases or preventive
detention cases.
• S3: The courts are prohibited to inquire into the proceedings of a House or
its committees.

Refer: https://www.insightsonindia.com/2021/06/11/breach-of-privilege-motion/

93. With reference to Indian Parliament, consider the following statements:


1. In the Lok Sabha, the Speaker nominates a committee of privileges consisting of 15
members as per respective party strengths.
2. In the Rajya Sabha, the Leader of the House is the first level of scrutiny of a privilege
motion.
Which of the given above statements is/are correct?
(a) 1 only
(b) 2 only
(c) Both 1 and 2
(d) Neither 1 nor 2
Ans: (a)
Explanation:
• S2: The Speaker/RS chairperson is the first level of scrutiny of a privilege
motion. The Speaker/Chair can decide on the privilege motion himself or herself
or refer it to the privileges committee of Parliament.
• S1: In the Lok Sabha, the Speaker nominates a committee of privileges
consisting of 15 members as per respective party strengths. A report is then
presented to the House for its consideration. The Speaker may permit a half-
hour debate while considering the report. The Speaker may then pass final
orders or direct that the report be tabled before the House. A resolution may
then be moved relating to the breach of privilege that has to be unanimously
passed. In the Rajya Sabha, the deputy chairperson heads the committee of
privileges that consists of 10 members.

Refer: https://www.insightsonindia.com/2021/06/11/breach-of-privilege-motion/

Telegram: https://t.me/insightsIAStips
50
Youtube: https://www.youtube.com/channel/UCpoccbCX9GEIwaiIe4HLjwA
Revision Through MCQs (RTM) Compilation (June 2021)

94. With reference to Nagaland, a state in northeastern India, consider the following
statements:
1. It became a full-fledged State on December 1, 1963.
2. It is known as the land of the rising sun.
3. As per the India State of Forest Report (ISFR) 2019, the recorded forest area of the
state is less than 50% of its geographical area.
Which of the given above statements is/are not correct?
(a) 1 only
(b) 1 and 2 only
(c) 2 and 3 only
(d) 1, 2 and 3
Ans: (c)
Explanation: here the directive word is not correct!!
• S1: Nagaland became a full-fledged State on December 1, 1963.
• S2: Arunachal Pradesh is known as the land of the rising sun with reference
to its position as the eastern most State of India.
• S3: The recorded forest area of the state is 8629.30 sq.km which is 52% of its
geographical area.
• The Forest Cover in the State is 12,486.40 sq km which is 75.31 % of the
State's geographical area read more>> https://fsi.nic.in/isfr19/vol2/isfr-2019-
vol-ii-nagaland.pdf

Refer: https://www.insightsonindia.com/2021/06/11/nagaland-to-form-panel-on-naga-
issue/

95. Consider the following statements regarding Pradhan Mantri Fasal Bhima Yogna
(PMFBY):
1. The Scheme covers all Food & Oilseeds crops for which past yield data is available
and for which requisite number of Crop Cutting Experiments (CCEs) are conducted
being under General Crop Estimation Survey (GCES).
2. The scheme is compulsory for loanee farmers availing Crop Loan /KCC account for
notified crops.
3. The scheme is being administered by Ministry of Agriculture.
Which of the given above statements is/are correct?
(a) 1 and 3 only
(b) 2 and 3 only
(c) 1 and 2 only
(d) 1, 2 and 3
Ans: (d)
Explanation:
• PMFBY provides a comprehensive insurance cover against failure of the crop
thus helping in stabilising the income of the farmers.
• S1: The Scheme covers all Food & Oilseeds crops and Annual
Commercial/Horticultural Crops for which past yield data is available and for
which requisite number of Crop Cutting Experiments (CCEs) are conducted
being under General Crop Estimation Survey (GCES).

Telegram: https://t.me/insightsIAStips
51
Youtube: https://www.youtube.com/channel/UCpoccbCX9GEIwaiIe4HLjwA
Revision Through MCQs (RTM) Compilation (June 2021)

• The scheme is implemented by empanelled general insurance companies.


Selection of Implementing Agency (IA) is done by the concerned State
Government through bidding.
• S2: The scheme is compulsory for loanee farmers availing Crop Loan /KCC
account for notified crops and voluntary for other others.
• S3: The scheme is being administered by Ministry of Agriculture.

Refer: https://www.insightsonindia.com/2021/06/11/what-is-the-beed-model-of-crop-
insurance-maharashtra-is-pushing-for/

96. In which one of the following States is Pakhui Tiger Reserve located?
(a) Arunachal Pradesh
(b) Manipur
(c) Meghalaya
(d) Nagaland
Ans: (a)
Explanation:
• Pakke Tiger Reserve is also known as Pakhui Tiger Reserve.
• This Tiger Reserve has won India Biodiversity Award 2016 in the category of
‘Conservation of threatened species’ for its Hornbill Nest Adoption Programme.
• It is bounded by Bhareli or Kameng River in the west and north, and by
Pakke River in the east.
• Neighbours: Papum Reserve Forest in Arunachal Pradesh, Assam’s Nameri
National Park, Doimara Reserve Forest and Eaglenest Wildlife Sanctuary.
• The main perennial streams in the area are the Nameri, Khari and Upper
Dikorai. West of Kameng River is Sessa Orchid Sanctuary.
• It falls within the Eastern Himalaya Biodiversity Hotspot.

Refer: Facts for Prelims: https://www.insightsonindia.com/2021/06/11/insights-daily-


current-affairs-pib-summary-11-june-2021/

97. Consider the following statements:


1. Food Safety and Standards Authority of India is a statutory body established under
the Ministry of Food Processing Industries (MoFPI).
2. FSSAI has the mandate to regulate manufacture, distribute, sell or import “organic
foods” as per the provisions laid under the Environment (Protection) Act, 1986.
Which of the given above statements is/are correct?
(a) 1 only
(b) 2 only
(c) Both 1 and 2
(d) Neither 1 nor 2
Ans: (d)
Explanation:
• S1: Food Safety and Standards Authority of India (FSSAI) is an autonomous
statutory body established under the Food Safety and Standards Act, 2006
(FSS Act).
o Ministry of Health & Family Welfare, Government of India is the
administrative Ministry of FSSAI.
o To pursue any food related business, the owner needs to get a certificate
and license with the permission of FSSAI.

Telegram: https://t.me/insightsIAStips
52
Youtube: https://www.youtube.com/channel/UCpoccbCX9GEIwaiIe4HLjwA
Revision Through MCQs (RTM) Compilation (June 2021)

• S2: Food Safety and Standards Authority of India (FSSAI) has the mandate to
regulate manufacture, distribute, sell or import “organic foods” as per the
provisions laid under Section 22 of the Food Safety Standards Act, 2006.
Organic foods are regulated by FoodSafety and Standards (Organic Foods)
Regulations, 2017 notified under the provisions of the Act.

Refer: Facts for Prelims: https://www.insightsonindia.com/2021/06/11/insights-daily-


current-affairs-pib-summary-11-june-2021/

98. Consider the following statements about Gharials:


1. It is locally extinct in Pakistan, Bhutan and Myanmar.
2. It is listed in schedule II of the Wildlife (Protection) Act, 1972
3. It has been listed as critically endangered on the IUCN Red List.
Which of the given above statements is/are correct?
(a) 1 and 3 only
(b) 2 and 3 only
(c) 1 and 2 only
(d) 1, 2 and 3
Ans: (a)
Explanation:
• S1: Historically, gharial were found in the river system of India, Pakistan,
Bangladesh and southern part of Bhutan and Nepal. Today they survive only in
the waters of India and Nepal. Today they survive only in the waters of India
and Nepal. Read more>>
• S2 and S3: Listed in Schedule I of Wildlife (Protection) Act, 1972 and as
Critically Endangered on IUCN Red List

Refer: Facts for Prelims: https://www.insightsonindia.com/2021/06/11/insights-daily-


current-affairs-pib-summary-11-june-2021/

99. With reference to communication technologies, what is/are the difference / differences
between LTE (Long-Term Evolution) and VoLTE (Voice over Long-Term Evolution)?
1. LTE is commonly marketed as 3G and VoLTE is commonly marketed as advanced
3G.
2. LTE is data-only technology and VoLTE is voice-only technology.
Select the correct answer using the code given below.
(a) 1 only
(b) 2 only
(c) Both 1 and 2
(d) Neither 1 nor 2
Ans: (d)
Explanation:
• VoLTE is voice calls over a 4G LTE network, rather than the 2G or 3G
connections. VolTE can transmit data too. VoLTE has up to three times more
voice and data capacity than 3G UMTS and up to six times more than 2G GSM.
So, both 1 and 2 are wrong.
• Cabinet approves 5 MHz spectrum for Railways:
o The Union Cabinet has approved the allotment of 5 Mhz spectrum in the
premium 700 MHz band to the Railways for captive use in areas of public
safety and security services.

Telegram: https://t.me/insightsIAStips
53
Youtube: https://www.youtube.com/channel/UCpoccbCX9GEIwaiIe4HLjwA
Revision Through MCQs (RTM) Compilation (June 2021)

Refer: Facts for Prelims: https://www.insightsonindia.com/2021/06/11/insights-daily-


current-affairs-pib-summary-11-june-2021/

100. In which one of the following States is Khonoma Nature Conservation and Tragopan
Sanctuary located?
(a) Arunachal Pradesh
(b) Manipur
(c) Meghalaya
(d) Nagaland
Ans: (d)
Explanation:
• Khonoma Nature Conservation & Tragopan Sanctuary wins India
Biodiversity Awards 2021
• KNCTS in Nagaland won the award under the ‘Sustainable Use of Biological
Resources’ category. KNCTS was established on December 11, 1998.

Refer: https://morungexpress.com/khonoma-nature-conservation-tragopan-sanctuary-
wins-india-biodiversity-awards-2021

Telegram: https://t.me/insightsIAStips
54
Youtube: https://www.youtube.com/channel/UCpoccbCX9GEIwaiIe4HLjwA
Revision Through MCQs (RTM) Compilation (June 2021)

RTM- REVISION THROUGH MCQS – 12th -Jun-2021

101. With reference to One Nation One Ration Card Scheme, which of the following
statements are correct?
1. Under the initiative, eligible beneficiaries would be able to avail their entitled
foodgrains under the National Food Security Act (NFSA) from any Fair Price Shop in
state across India using the same ration card.
2. For national portability, the state governments have been asked to issue the ration
card in bi-lingual format.
3. The integrated Aadhaar Enabled Payment System (AePS) provides the technological
platform for the inter-state portability of ration cards.
Select the correct answer using the given code below:
(a) 1 and 2 only
(b) 2 and 3 only
(c) 1 and 3 only
(d) 1, 2 and 3
Ans: (a)
Explanation:
• S3: The Integrated Management of Public Distribution System (IM-PDS)
portal provides the technological platform for the inter-state portability of ration
cards.
• S2: For national portability, the state governments have been asked to issue the
ration card in bi-lingual format, wherein besides the local langauge, the other
language could be Hindi or English.
• S1: Under the initiative, eligible beneficiaries would be able to avail their entitled
foodgrains under the National Food Security Act (NFSA) from any Fair Price
Shop in the country using the same ration card.

Refer: https://www.insightsonindia.com/2021/06/12/one-nation-one-ration-card-scheme-
6/

102. Consider the following statements regarding the International Court of Justice (ICJ):
1. It is one of the six principal organs of the United Nations (UN).
2. It settles disputes between states in accordance with international law and gives
advisory opinions on international legal issues.
3. The seat of the Court is at the Peace Palace in The Hague (Netherlands).
Select the correct answer using the given code below:
(a) 1 and 2 only
(b) 2 and 3 only
(c) 1 and 3 only
(d) 1, 2 and 3
Ans: (d)
Explanation:
• The International Court of Justice (ICJ) is the principal judicial organ of the
United Nations (UN). It was established in June 1945 by the Charter of the
United Nations and began work in April 1946.

Telegram: https://t.me/insightsIAStips
55
Youtube: https://www.youtube.com/channel/UCpoccbCX9GEIwaiIe4HLjwA
Revision Through MCQs (RTM) Compilation (June 2021)

• The seat of the Court is at the Peace Palace in The Hague (Netherlands). Of
the six principal organs of the United Nations, it is the only one not located in
New York (United States of America).
• The Court’s role is to settle, in accordance with international law, legal
disputes submitted to it by States and to give advisory opinions on legal
questions referred to it by authorized United Nations organs and specialized
agencies.
• The Court is composed of 15 judges, who are elected for terms of office of nine
years by the United Nations General Assembly and the Security Council. It is
assisted by a Registry, its administrative organ. Its official languages are English
and French.

Refer: https://www.insightsonindia.com/2021/06/12/pak-passes-bill-to-let-jadhav-
appeal/

103. Consider the following statements:


According to the new Vehicle Scrappage Policy,
1. Commercial vehicles of more than 15 years will have to be mandatorily scrapped if
they don’t pass the fitness and emission tests.
2. Passenger vehicles of more than 20 years will have to be voluntarily scrapped if they
don’t pass the fitness and emission tests.
3. Government departments will have to let go of their vehicles after they are used for
15 years.
Which of the given above statements is/are correct?
(a) 1 and 2 only
(b) 3 only
(c) 1 and 3 only
(d) 1, 2 and 3
Ans: (c)
Explanation:
• According to the new policy, commercial vehicles of more than 15 years and
passenger vehicles of more than 20 years will have to be mandatorily scrapped if
they don’t pass the fitness and emission tests. Also, government departments
will have to let go of their vehicles after they are used for 15 years.

Refer: https://www.insightsonindia.com/2021/06/12/vehicle-scrappage-policy-2/

104. Consider the following statements:


1. EnVision is a European Space Agency (ESA) led mission with contributions from
ISRO.
2. NASA recently selected ‘DAVINCI+’ and ‘VERITAS’ missions to the planet Venus.
Which of the given above statements is/are correct?
(a) 1 only
(b) 2 only
(c) Both 1 and 2
(d) Neither 1 nor 2
Ans: (b)
Explanation:
• What is EnVision?
o EnVision is an ESA led mission with contributions from NASA.

Telegram: https://t.me/insightsIAStips
56
Youtube: https://www.youtube.com/channel/UCpoccbCX9GEIwaiIe4HLjwA
Revision Through MCQs (RTM) Compilation (June 2021)

o It will be launched on an Ariane 6 rocket, the spacecraft will take about


15 months to reach Venus and will take 16 more months to achieve orbit
circularisation.
o Objectives: To study the planet’s atmosphere and surface, monitor trace
gases in the atmosphere and analyse its surface composition. A radar
provided by NASA will help to image and map the surface.
• Other missions to Venus:
o NASA recently selected two missions to the planet Venus- DAVINCI+ and
VERITAS.
o Previously, an ESA-led mission to Venus called ‘Venus Express’ (2005-
2014) focussed on atmospheric research and pointed to volcanic hotspots
on the planet’s surface.
o The first spacecraft to Venus was the Soviet Union’s Venera series,
followed by NASA’s Magellan Mission that studied Venus from 1990-
1994.
o As of now, Japan’s Akatsuki mission is studying the planet from Orbit.

Refer: https://www.insightsonindia.com/2021/06/12/european-space-agencys-envision-
mission-to-venus/

105. Consider the following statements about planet Venus:


1. It is the second closest planet to the Sun.
2. It is called the Earth’s twin because of their similar atmosphere.
3. It does not have a moon and rings.
Which of the given above statements is/are correct?
(a) 1 and 2 only
(b) 2 and 3 only
(c) 1 and 3 only
(d) 1, 2 and 3
Ans: (c)
Explanation:
• S1: It is the second closest planet to the Sun.
• S2: It is called the Earth’s twin because of their similar sizes.
o Venus’ thick atmosphere traps heat and is the reason that it is the
hottest planet in the solar system, despite coming after Mercury, the
closest planet to the Sun. Surface temperatures on Venus can go up to
471 degrees Celsius, which is hot enough to melt lead.
• S3: Venus also does not have a moon and no rings.

Refer: https://www.insightsonindia.com/2021/06/12/european-space-agencys-envision-
mission-to-venus/

106. Project HIMANK is a project of which of the following?


(a) Indian Defense Service of Engineers (IDSE)
(b) Military Officers (Corps of Engineers)
(c) Border Roads Organisation (BRO)
(d) All of the above
Ans: (c)
Explanation:

Telegram: https://t.me/insightsIAStips
57
Youtube: https://www.youtube.com/channel/UCpoccbCX9GEIwaiIe4HLjwA
Revision Through MCQs (RTM) Compilation (June 2021)

• Project HIMANK, is a project of the Border Roads Organisation (BRO) being


implemented in the Ladakh region of Jammu & Kashmir. This project started in
1985. Under this project, BRO is responsible for the construction and
maintenance of roads and related infrastructure including the world's highest
motorable roads across the Khardung La, Tanglang La and Chang La passes.
• Umling La:
o At 5.793m (19,005ft) above the sea level, Umling La is one of the highest
motorable roads in the world.
o Located in the Ladakh region of Jammu and Kashmir.
o The 54 km long road passing through Umling La Pass connected the
villages of Chisumle and Demchok and took six years to be built by BRO
under Project Himank.

Refer: Facts for Prelims: https://www.insightsonindia.com/2021/06/12/insights-daily-


current-affairs-pib-summary-12-june-2021/

107. Consider the following statements:


1. The National Security Guard (NSG) is a counter-terrorism unit under the Indian
Ministry of Home Affairs (MHA).
2. It was set up in 1985 after the assassination of Prime Minister Indira Gandhi.
Which of the given above statements is/are correct?
(a) 1 only
(b) 2 only
(c) Both 1 and 2
(d) Neither 1 nor 2
Ans: (a)
Explanation:
• S1: The NSG is a counter-terrorism unit that formally came into existence in
1986 by an act of Parliament- ‘National Security Guard Act, 1986’. It operates
under the Ministry of Home Affairs and is a task-oriented force.
• S2: The Special Protection Group (SPG) was started in 1985 in the wake of the
killing of PM Indira Gandhi in 1984.

Refer: Facts for Prelims: https://www.insightsonindia.com/2021/06/12/insights-daily-


current-affairs-pib-summary-12-june-2021/

108. Kamlang wildlife sanctuary is located in


(a) Assam
(b) Arunachal Pradesh
(c) Nagaland
(d) Manipur
Ans: (b)
Explanation:
• The Kamlang Wildlife Sanctuary is situated in the Lohit District of Arunachal
Pradesh. The park is named after the Kamlang River which flows through it.

Refer: http://arunachalforests.gov.in/kamlang_wildlife_sanctuary.html

109. “Caspian Sea” is bordered by which one of the following groups of nations?
(a) Russia, Kazakhstan, Turkmenistan and Iran
(b) Russia, Kazakhstan, Afghanistan and Azerbaijan

Telegram: https://t.me/insightsIAStips
58
Youtube: https://www.youtube.com/channel/UCpoccbCX9GEIwaiIe4HLjwA
Revision Through MCQs (RTM) Compilation (June 2021)

(c) Russia, Kazakhstan, Uzbekistan and Iran


(d) Russia, Kazakhstan, Afghanistan and Iran
Ans: (a)
Explanation:
• Caspian Sea is bordered by following countries:


Refer: World physical map

110. In which of the following states/UTs you are most likely to find Tropical evergreen
forest, semi evergreen, deciduous, Pine and Temperate forests?
(a) Arunachal Pradesh
(b) Kerala
(c) Andaman and Nicobar
(d) Karnataka
Ans: (a)
Explanation:
• Situated on the northeastern tip of the country, the state of Arunachal Pradesh
is a part of Eastern Himalayan Ranges. The important forests types found in the
state are Tropical evergreen, semi evergreen, deciduous, Pine, Temperate, Alpine
and grassland.

Refer: India physical map

RTM- REVISION THROUGH MCQS – 14th -Jun-2021

111. Consider the following statements regarding Citizenship (Amendment) Act, 2019:
1. It exempts certain areas in the North-East from this provision.
2. According to the Citizenship Act, a foreigner may register as an Overseas Citizens of
India (OCI) if they are of Indian origin or their spouse is of Indian origin.
Which of the given above statements is/are correct?
(a) 1 only
(b) 2 only
(c) Both 1 and 2
(d) Neither 1 nor 2
Ans: (c)
Explanation:
• S1: it exempts certain areas in the North-East from this provision. It would not
apply to tribal areas of Assam, Meghalaya, Mizoram and Tripura. This
effectively means that Arunachal Pradesh, Nagaland and Mizoram along with

Telegram: https://t.me/insightsIAStips
59
Youtube: https://www.youtube.com/channel/UCpoccbCX9GEIwaiIe4HLjwA
Revision Through MCQs (RTM) Compilation (June 2021)

almost whole of Meghalaya and parts of Assam and Tripura would stay out of
the purview of the Citizenship (Amendment) Act.
• S2: According to the Citizenship Act, a foreigner may register as an Overseas
Citizens of India (OCI) if they are of Indian origin or their spouse is of Indian
origin. The Citizenship (Amendment) Act entitles the OCI cardholders to benefits
such as the right to travel to India, and to work and study in the country.

Refer: https://www.insightsonindia.com/2021/06/14/union-home-ministry-order-invites-
citizenship/

112. Consider the following statements:


1. The Group of 7 (G7) is an informal group of seven countries including Japan and
Italy.
2. The Group of 7 (G7) Secretariat is based in Paris, France.
Which of the given above statements is/are correct?
(a) 1 only
(b) 2 only
(c) Both 1 and 2
(d) Neither 1 nor 2
Ans: (a)
Explanation:
• S1: The Group of 7 (G7) is an informal group of seven countries — the United
States, Canada, France, Germany, Italy, Japan and the United Kingdom, the
heads of which hold an annual summit with European Union and other invitees.
• S2: The G7 is not based on a treaty and has no permanent secretariat or
office; its presidency rotates annually among the members, with the presiding
nation setting the group's priorities, and hosting and organizing its summit.

Refer: https://www.insightsonindia.com/2021/06/14/g7-meet/

113. Arrange the following locations in the direction of North to South:


1. Siachen glacier
2. Nubra valley
3. Khardung la
Select the correct answer using the code below:
(a) 1-2-3
(b) 1-3-2
(c) 2-1-3
(d) 2-3-1
Ans: (a)
Explanation:


Refer: https://www.insightsonindia.com/2021/06/14/a-year-after-galwan-clash/

Telegram: https://t.me/insightsIAStips
60
Youtube: https://www.youtube.com/channel/UCpoccbCX9GEIwaiIe4HLjwA
Revision Through MCQs (RTM) Compilation (June 2021)

114. Consider the following statements regarding Rare Earth elements.


1. Rare Earth elements are not radioactive in nature.
2. India has the world's largest rare earth elements reserves.
Which of the given above statements is/are correct?
(a) 1 only
(b) 2 only
(c) Both 1 and 2
(d) Neither 1 nor 2
Ans: (d)
Explanation:
• S1: Despite their classification, most of these elements are not really “rare”.
One of the Rare Earths, promethium, is radioactive.
• S2: China’s Rare Earths deposits account for 80% of identified global reserves.

Refer: https://www.insightsonindia.com/2021/06/14/rare-earth-metals-at-the-heart-of-
china-us-rivalry/

115. Recently, our scientists have discovered a 15-meter-tall tree that belongs to the genus
of the coffee family which is distinguished by a long stem with a whitish coating on the
trunk and oblong-obovate leaves with a cuneate base. In which part of India has it been
discovered?
(a) Andaman Islands
(b) Western Ghats
(c) Araku Valley
(d) Tropical rain forests of northeast
Ans: (a)
Explanation: Tree of coffee family discovered in Andaman and Nicobar
• A 15 metre tall tree that belongs to genus of the coffee family has recently been
discovered in Andaman Islands
• The new species Pyrostria laljii, is also the first record of the genus Pyrostria in
India. Trees belonging to these species are usually found in Madagascar
• The tree is distinguished by a long stem with a whitish coating on the trunk and
oblong-obovate leaves with a cuneate base and was first reported from Wandoor
forest in South Andaman
• Other places where tree could be located are: Jarawa reserve forest, chidia tapu
forest
• Pyrostria laljii: Critically endangered as per IUCN status

Refer: Facts for Prelims: https://www.insightsonindia.com/2021/06/14/insights-daily-


current-affairs-pib-summary-14-june-2021/

116. What is “New Shepherd”, sometimes seen in the news?


(a) A first-generation Chinese aircraft carrier
(b) An American anti-missile system
(c) A launch vehicle developed by SpaceX
(d) None of the above
Ans: (d)
Explanation:

Telegram: https://t.me/insightsIAStips
61
Youtube: https://www.youtube.com/channel/UCpoccbCX9GEIwaiIe4HLjwA
Revision Through MCQs (RTM) Compilation (June 2021)

• New Shepard is a vertical-takeoff, vertical-landing, crew-rated suborbital launch


vehicle that is being developed by Blue Origin as a commercial system for
suborbital space tourism.

Refer: Facts for Prelims: https://www.insightsonindia.com/2021/06/14/insights-daily-


current-affairs-pib-summary-14-june-2021/

117. Which of the following temples is/are famous for Lord Shiva?
1. Srisailam temple
2. Ramanathaswamy temple
3. Ghrishneshwara temple
Select the correct answer using the code given below:
(a) 1 only
(b) 2 and 3 only
(c) 1 and 3 only
(d) 1, 2 and 3
Ans: (d)
Explanation:
• Copperplate inscriptions found at Srisailam temple
o The Bhramarambha Mallikarjuna Devasthanam, engaged in reviving
ancient Ganta Matham near Srisailam temple complex, chanced upon six
sets of copperplate inscriptions on Sunday.
o There were 18 copper leaves found that could be dated back to
somewhere between 14 and 16 centuries.
o The inscriptions depict the far and wide patronage by people.
o Out of the six sets, four have inscriptions in Sanskrit and Nandi-Nāgarī
script and the other two plates are in Telugu script, he added.
• A Jyotirlinga or Jyotirlingam, is a devotional representation of the Hindu god
Shiva.
• The twelve jyotirlingas are:
o Somnath in Gir Somnath, Gujarat
o Mallikarjuna in Srisailam, Andhra Pradesh
o Mahakaleshwar in Ujjain, Madhya Pradesh
o Omkareshwar in Khandwa, Madhya Pradesh
o Kedarnath in Rudraprayag, Uttarakhand
o Bhimashankar in Pune, Maharashtra
o Kashi Vishwanath in Varanasi, Uttar Pradesh
o Trayambakeshwar in Nashik, Maharashtra
o Vaidyanath in Deoghar, Jharkhand
o Nageshwara in Dwarka, Gujarat
o Ramanathaswamy (Rameshwar) in Rameswaram, Tamil Nadu
o Ghrishneshwara in Aurangabad, Maharashtra

Refer: Facts for Prelims: https://www.insightsonindia.com/2021/06/14/insights-daily-


current-affairs-pib-summary-14-june-2021/

118. Which of the following is geographically closest to Madagascar?


(a) Comoros
(b) Mauritius
(c) Seychelles

Telegram: https://t.me/insightsIAStips
62
Youtube: https://www.youtube.com/channel/UCpoccbCX9GEIwaiIe4HLjwA
Revision Through MCQs (RTM) Compilation (June 2021)

(d) Maldives
Ans: (a)
Explanation:
• Self-explanatory


Refer: World physical map

119. Consider the following pairs:


City Country
1. Mogadishu Kenya
2. Zanzibar Tanzania
3. Mombasa Somalia
Which of the given above pairs is/are correctly matched?
(a) 1 and 2 only
(b) 2 only
(c) 1 and 3 only
(d) 2 and 3 only
Ans: (b)
Explanation:
• Mogadishu:: Somalia
• Zanzibar:: Tanzania
• Mombasa:: Kenya


Refer: World physical map

120. Consider the following statements:


1. The Nuclear Security Summits are periodically held under the aegis of the United
Nations.
2. The International Panel on Fissile Materials is an organ of International Atomic
Energy Agency.
Which of the statements given above is/are correct?
(a) 1 only

Telegram: https://t.me/insightsIAStips
63
Youtube: https://www.youtube.com/channel/UCpoccbCX9GEIwaiIe4HLjwA
Revision Through MCQs (RTM) Compilation (June 2021)

(b) 2 only
(c) Both 1 and 2
(d) Neither 1 nor 2
Ans: (d)
Explanation:
• Statement 1: It is wrong.
o In his 2009 speech, Obama (Former US President) stated that nuclear
terrorism “is the most immediate and extreme threat to global security.”
o To mitigate this threat, he urged that “we act with purpose and without
delay,” announcing “a new international effort to secure vulnerable
nuclear material around the world” that would begin with “a Global
Summit on Nuclear Security that the United States will host.”
o http://www.nss2016.org/about-nss/history/
• Statement 2: This is also wrong.
o Established in 2006, it is a group of independent nuclear experts from 18
countries: Brazil, Canada, China, France, Germany, India, Iran, Japan,
the Netherlands, Mexico, Norway, Pakistan, South Korea, Russia, South
Africa, Sweden, the United Kingdom, and the United States.
o It aims to advance international initiatives to “secure and to sharply
reduce all stocks of highly enriched uranium and separated plutonium,
the key materials in nuclear weapons, and to limit any further
production”
o www.fissilematerials.org/ipfm/about.html

Refer: UPSC CSE 2017

RTM- REVISION THROUGH MCQS – 15th -Jun-2021

121. Consider the following statements:


1. Tulu is listed in the Eighth Schedule of the Constitution through 95th
Constitutional Amendment Act.
2. Tulu-speaking people are larger in number than speakers of Manipuri and Sanskrit.
Which of the given above statements is/are correct?
(a) 1 only
(b) 2 only
(c) Both 1 and 2
(d) Neither 1 nor 2
Ans: (b)
Explanation:
• S2: The Census (2011) reports 18,46,427 native speakers of Tulu in India. The
Tulu-speaking people are larger in number than speakers of Manipuri and
Sanskrit, which have the Eighth Schedule status.
• S1: The 22 languages which are listed in the Eighth Schedule are Assamese,
Bengali, Bodo, Dogri, Gujarati, Hindi, Kannada, Kashmiri, Konkani, Maithili,
Malayalam, Manipuri, Marathi, Nepali, Oriya, Punjabi, Sanskrit, Santali, Sindhi,
Tamil, Telugu and Urdu.
• Tulu and English are not listed in Eighth Schedule of the Indian Constitution.

Telegram: https://t.me/insightsIAStips
64
Youtube: https://www.youtube.com/channel/UCpoccbCX9GEIwaiIe4HLjwA
Revision Through MCQs (RTM) Compilation (June 2021)

Refer: https://www.insightsonindia.com/2021/06/15/the-history-of-tulu-and-the-
demand-for-official-language-status/

122. With reference to the provisions made under the National Food Security Act, 2013,
consider the following statements:
1. The families coming under the category of ‘below poverty line (BPL)’ only are eligible
to receive subsidies food grains.
2. The eldest woman in a household, of age 18 years or above, shall be the head of the
household for the purpose of issuance of a ration card.
3. Pregnant women and lactating mothers are entitled to a ‘take-home ration’ of 1600
calories per day during pregnancy and for six months thereafter
Which of the statement given above is/are correct?
(a) 1 and 2 only
(b) 2 only
(c) 1 and 3 only
(d) 2 and 3 only
Ans: (b)
Explanation:
• S1: Coverage and entitlement under Targeted Public Distribution System (TPDS):
Upto 75% of the rural population and 50% of the urban population will be
covered under TPDS, with uniform entitlement of 5 kg per person per month. Not
restricted to only BPL families.
• S3: Nutritional Support to women and children: Children in the age group of
6 months to 14 years and pregnant women and lactating mothers will be entitled
to meals as per prescribed nutritional norms under Integrated Child
Development Services (ICDS) and Mid-Day Meal (MDM) schemes.
Malnourished children up to the age of 6 have been prescribed for higher
nutritional norms.
o Maternity Benefit: Pregnant women and lactating mothers will also be
receiving maternity benefit of Rs. 6,000.
• S2: As a step towards women empowerment, the eldest woman of the
household of age 18 years or above is mandated to be the head of the
household for the purpose of issuing of ration cards under the Act.

Refer: https://www.insightsonindia.com/2021/06/15/national-food-security-act-nfsa/

123. Consider the following statements about Food Corporation of India.


1. It is a statutory body created and run by the Government of India.
2. It is under the jurisdiction of Ministry of Agriculture & Farmers Welfare.
Which of the given above statements is/are correct?
(a) 1 only
(b) 2 only
(c) Both 1 and 2
(d) Neither 1 nor 2
Ans: (a)
Explanation:
• S1: The Food Corporation of India was setup under the Food Corporation's Act
1964 , in order to fulfill following objectives of the Food Policy:

Telegram: https://t.me/insightsIAStips
65
Youtube: https://www.youtube.com/channel/UCpoccbCX9GEIwaiIe4HLjwA
Revision Through MCQs (RTM) Compilation (June 2021)

o Effective price support operations for safeguarding the interests of the


farmers.
o Distribution of foodgrains throughout the country for public distribution
system.
o Maintaining satisfactory level of operational and buffer stocks of
foodgrains to ensure National Food Security
• S2: It is under the jurisdiction of Ministry of Consumer Affairs, Food and
Public Distribution, Government of India.

Refer: https://www.insightsonindia.com/2021/06/15/national-food-security-act-nfsa/

124. Consider the following statements about North Atlantic Treaty Organization:
1. It is an intergovernmental military alliance.
2. Only Canada, France, Italy, the United Kingdom and the United States are the
original founding members of the NATO.
Which of the given above statements is/are correct?
(a) 1 only
(b) 2 only
(c) Both 1 and 2
(d) Neither 1 nor 2
Ans: (a)
Explanation:
• S1: About North Atlantic Treaty Organization:
o It is an intergovernmental military alliance.
o Established by Washington treaty.
o Treaty that was signed on 4 April 1949.
o Headquarters — Brussels, Belgium.
o Headquarters of Allied Command Operations — Mons, Belgium.
• S2: Its original members were Belgium, Canada, Denmark, France, Iceland,
Italy, Luxembourg, the Netherlands, Norway, Portugal, the United Kingdom, and
the United States.

Refer: https://www.insightsonindia.com/2021/06/15/nato-summit/

125. Consider the following statements:


1. Russia and the China together possessed over 90% of global nuclear weapons.
2. India has more nuclear warheads than Pakistan.
Which of the given above statements is/are correct?
(a) 1 only
(b) 2 only
(c) Both 1 and 2
(d) Neither 1 nor 2
Ans: (d)
Explanation: Swedish think tank Stockholm International Peace Research Institute
(SIPRI) has released its Year Book 2021.
• Key findings:
o S2: India possessed an estimated 156 nuclear warheads at the start of
2021, compared with 150 at the start of last year, while Pakistan had
165 warheads, up from 160 in 2020.

Telegram: https://t.me/insightsIAStips
66
Youtube: https://www.youtube.com/channel/UCpoccbCX9GEIwaiIe4HLjwA
Revision Through MCQs (RTM) Compilation (June 2021)

o China’s nuclear arsenal consisted of 350 warheads, up from 320 at the


start of 2020.
o The nine nuclear armed states — the U.S., Russia, the U.K., France,
China, India, Pakistan, Israel and North Korea — together possessed an
estimated 13,080 nuclear weapons at the start of 2021.
o S1: Russia and the U.S. together possessed over 90% of global nuclear
weapons.

Refer: https://www.insightsonindia.com/2021/06/15/stockholm-international-peace-
research-institute-sipri-year-book-2021/

126. Consider the following pairs:


The rocket system designed/built by
1. New Shephard Amazon
2. Stardust 1.0 bluShift
3. Falcon 9 SpaceX
Which of the given above pairs is/are correctly matched?
(a) 1 and 3 only
(b) 3 only
(c) 2 and 3 only
(d) 2 only
Ans: (c)
Explanation:
• S1: New Shepard is a crew-rated suborbital launch vehicle that is being
developed by Blue Origin as a commercial system for suborbital space tourism.
• S2: Stardust 1.0 is manufactured by bluShift, an aerospace company based in
Maine that is developing rockets that are powered by bio-derived fuels.
• S3: Falcon 9 is a partially reusable two-stage-to-orbit medium-lift launch vehicle
designed and manufactured by SpaceX in the United States.

Refer: https://www.insightsonindia.com/2021/06/15/what-is-the-new-shephard-rocket-
system/

127. India enacted The Geographical Indications of Goods (Registration and Protection) Act,
1999 in order to comply with the obligations
(a) ILO
(b) IMF
(c) UNCTAD
(d) WTO
Ans: (d)
Explanation:
• Article 22 of the TRIPS Agreements (WTO Agreement) defines Geographical
Indication as “indications which identify a good as originating in the territory of
a Member, or a region or locality in that territory, where a given quality,
reputation or other characteristic of the good is essentially attributable to its
geographical origin”.
• Consequently, upon India joining as a member state of the TRIPS Agreement sui-
geneis legislation for the protection of geographical indication was enacted in
1999. Geographical Indications are covered as an element of Intellectual
Property Rights under the TRIPS Agreement (WTO Agreement).

Telegram: https://t.me/insightsIAStips
67
Youtube: https://www.youtube.com/channel/UCpoccbCX9GEIwaiIe4HLjwA
Revision Through MCQs (RTM) Compilation (June 2021)

Refer: https://www.insightsonindia.com/2021/06/15/india-and-pakistans-battle-over-
basmati/

128. Consider the following statements:


1. National Securities Depository Limited (NSDL) is the regulatory body for securities
and commodity market in India.
2. It is under the jurisdiction of Ministry of Finance, Government of India.
Which of the given above statements is/are correct?
(a) 1 only
(b) 2 only
(c) Both 1 and 2
(d) Neither 1 nor 2
Ans: (b)
Explanation:
• S1: The Securities and Exchange Board of India (SEBI) is the regulatory body for
securities and commodity market in India.
• S2: National Securities Depository Limited (NSDL) is an Indian central securities
depository under the jurisdiction of Ministry of Finance.
• National Securities Depository Limited:
o The enactment of Depositories Act in August 1996 paved the way for
establishment of NSDL in August 1996.
o It handles most of the securities held and settled in dematerialized form
in the Indian capital market.
o NSDL works to support the investors and brokers in the capital market of
the country.
o It aims at ensuring the safety and soundness of Indian marketplaces by
developing settlement solutions that increase efficiency, minimize risk
and reduce costs.

Refer: facts for prelims: https://www.insightsonindia.com/2021/06/15/insights-daily-


current-affairs-pib-summary-15-june-2021/

129. It is a unique Mango from Bhagalpur district of Bihar and it is known for its light
yellow skin and special aroma. Also it received the GI tag in 2018. It is
(a) Alphonso Mango
(b) Chausa Mango
(c) Badami Mango
(d) Zardalu Mango
Ans: (d)
Explanation: Jardalu Mango:
• Jardalu or Zardalu mango is a unique variety of mango grown in Bhagalpur and
adjoining districts of Bihar.
• It received the GI tag in 2018.


Refer: facts for prelims: https://www.insightsonindia.com/2021/06/15/insights-daily-
current-affairs-pib-summary-15-june-2021/

Telegram: https://t.me/insightsIAStips
68
Youtube: https://www.youtube.com/channel/UCpoccbCX9GEIwaiIe4HLjwA
Revision Through MCQs (RTM) Compilation (June 2021)

130. Consider the following pairs:


Places in news Country
1. Stockholm Canada
2. Brussels Belgium
3. Carbis Bay United Kingdom
Which of the given above pairs is/are correctly matched?
(a) 1 and 3 only
(b) 3 only
(c) 2 and 3 only
(d) 2 only
Ans: (c)
Explanation:
• S1: Stockholm International Peace Research Institute (SIPRI) is an international
institute based in Stockholm, Sweden.
• S2: 2021 NATO Summit Held in Brussels, Belgium.
• S3: The 47th G7 summit was held on 11–13 June 2021 in Cornwall in the
United Kingdom.
o Carbis Bay is a village in Cornwall, UK.

Refer: facts for prelims: https://www.insightsonindia.com/2021/06/15/insights-daily-


current-affairs-pib-summary-15-june-2021/

RTM- REVISION THROUGH MCQS – 16th -Jun-2021

131. Consider the following statements about Ram Prasad Bismil:


1. He was associated with the Arya Samaj from an early age.
2. He participated in the Muzaffarpur Conspiracy of 1908.
3. He played a crucial role in Ghadar Movement.
Which of the given above statements is/are correct?
(a) 1 and 2 only
(b) 1 only
(c) 2 and 3 only
(d) 1, 2 and 3
Ans: (b)
Explanation:
• S2: Events he was associated with:
o He participated in the Mainpuri conspiracy of 1918.
o On August 9, 1925, Ram Prasad Bismil along with companions
Ashfaqulla Khan and others executed the plan of looting the train at
Kakori near Lucknow.
• S1: He was associated with the Arya Samaj from an early age.

Refer: https://www.insightsonindia.com/2021/06/16/ram-prasad-bismil/

132. Consider the following statements regarding the Arya Samaj:


1. It is a Hindu revivalist movements.
2. It was founded by Dayanand Saraswati in Lahore, Punjab.
3. It believes in the infallible authority of the Vedas.

Telegram: https://t.me/insightsIAStips
69
Youtube: https://www.youtube.com/channel/UCpoccbCX9GEIwaiIe4HLjwA
Revision Through MCQs (RTM) Compilation (June 2021)

Which of the given above statements is/are correct?


(a) 1 and 2 only
(b) 3 only
(c) 2 and 3 only
(d) 1, 2 and 3
Ans: (b)
Explanation:
• S1 and S2: It is a Hindu reform movement that was founded by Dayanand
Saraswati in 1875 in Bombay.
• S3: The movement believes in the infallible authority of the Vedas.
o The central objectives of Arya Samaj is to, “eradicate Ignorance (Agyan),
Indigence or Poverty (Abhav) and Injustice (Anayay) from this earth.
o Members of the Arya Samaj believe in one God and reject the worship
of idols.

Refer: https://www.insightsonindia.com/2021/06/16/ram-prasad-bismil/

133. Consider the following statements about the United Nations Convention on the Law of
the Sea (UNCLOS):
1. It was developed by the International Maritime Organization (IMO).
2. It is the globally recognized regime dealing with all matters relating to the law of the
sea.
Which of the given above statements is/are correct?
(a) 1 only
(b) 2 only
(c) Both 1 and 2
(d) Neither 1 nor 2
Ans: (b)
Explanation:
• S1: UNCLOS is an international agreement that resulted from the third
United Nations Conference on the Law of the Sea (UNCLOS III), which took
place between 1973 and 1982.
• S2: The Law of the Sea Convention defines the rights and responsibilities of
nations with respect to their use of the world's oceans, establishing guidelines
for businesses, the environment, and the management of marine natural
resources.

Refer: https://www.insightsonindia.com/2021/06/16/supreme-court-closes-criminal-case-
against-italian-marines/

134. The ‘Enrica Lexie Case’ was in news recently, is related to which of the following?
(a) International patent system
(b) Minimum global corporation tax
(c) International drugs syndicate
(d) None of the above
Ans: (d)
Explanation:
• The Enrica Lexie case is an ongoing international controversy about a
shooting off the western coast of India.

Telegram: https://t.me/insightsIAStips
70
Youtube: https://www.youtube.com/channel/UCpoccbCX9GEIwaiIe4HLjwA
Revision Through MCQs (RTM) Compilation (June 2021)


Refer: https://www.insightsonindia.com/2021/06/16/supreme-court-closes-criminal-case-
against-italian-marines/

135. Consider the following statements about the Food and Agriculture Organization (FAO):
1. It is a specialized agency of the United Nations.
2. It is the only multilateral development organization that focuses solely on rural
economies and food security.
3. It is headquartered in Rome, Italy.
Which of the given above statements is/are correct?
(a) 1 and 2 only
(b) 2 and 3 only
(c) 1 and 3 only
(d) 1, 2 and 3
Ans: (c)
Explanation:
• S1: the Food and Agriculture Organization (FAO) is a specialized agency of the
United Nations that leads international efforts to defeat hunger and improve
nutrition and food security.
• S2: International Fund for Agricultural Development is the only multilateral
development organization that focuses solely on rural economies and food
security.
• S3: FAO is headquartered in Rome, Italy.

Refer: https://www.insightsonindia.com/2021/06/16/food-and-agriculture-organizations-
fao-conference/

136. Which one of the following is not published/released by the Food and Agriculture
Organization (FAO)?
(a) The State of the World's Forests

Telegram: https://t.me/insightsIAStips
71
Youtube: https://www.youtube.com/channel/UCpoccbCX9GEIwaiIe4HLjwA
Revision Through MCQs (RTM) Compilation (June 2021)

(b) The State of Agricultural Commodity Markets


(c) The Global Forest Resources Assessment
(d) All the above are published by FAO
Ans: (d)
Explanation: Important reports of FAO:
• The State of the World's Forests
• The State of Agricultural Commodity Markets
• The Global Forest Resources Assessment

Refer: https://www.insightsonindia.com/2021/06/16/food-and-agriculture-organizations-
fao-conference/

137. Consider the following statements about ‘Polar-Areas Stellar-Imaging in Polarisation


High-Accuracy Experiment’ (PASIPHAE):
1. It is the largest ever collaborative sky surveying project in the history of science.
2. It is spearheaded by the Massachusetts Institute of Technology, USA.
Which of the given above statements is/are correct?
(a) 1 only
(b) 2 only
(c) Both 1 and 2
(d) Nether 1 nor 2
Ans: (d)
Explanation:
• S1: Polar-Areas Stellar-Imaging in Polarisation High-Accuracy Experiment
(PASIPHAE) is an international collaborative sky surveying project.
o Scientists aim to study the polarisation in the light coming from millions
of stars.
• S2: PASIPHAE is an international collaboration between the Crete Astrophysics
Group, Caltech in the US, IUCAA in India, the South African Astronomical
Observatory, and the University of Oslo in Norway.

Refer: https://www.insightsonindia.com/2021/06/16/what-is-pasiphae-and-why-is-it-
important/

138. Consider the following statements regarding the United Nations Convention to Combat
Desertification (UNCCD):
1. It is the only convention stemming from a direct recommendation of the Rio
Conference’s Agenda 21.
2. It is the sole legally binding international agreement linking environment and
development to sustainable land management.
3. Its Secretariat is administered by UNFCCC and is located at Geneva, Switzerland.
Which of the given above statements is/are correct?
(a) 1 and 2 only
(b) 2 and 3 only
(c) 1 and 3 only
(d) 1, 2 and 3
Ans: (a)
Explanation: United Nations Convention to Combat Desertification (UNCCD):
• Established in 1994.

Telegram: https://t.me/insightsIAStips
72
Youtube: https://www.youtube.com/channel/UCpoccbCX9GEIwaiIe4HLjwA
Revision Through MCQs (RTM) Compilation (June 2021)

• It is the sole legally binding international agreement linking environment and


development to sustainable land management.
• It is the only convention stemming from a direct recommendation of the Rio
Conference’s Agenda 21.
• Focus areas: The Convention addresses specifically the arid, semi-arid and dry
sub-humid areas, known as the drylands, where some of the most vulnerable
ecosystems and peoples can be found.
• S3: The permanent secretariat of the Convention was established in Article 23
of the UNCCD. It has been located in Bonn, Germany since January 1999.

Refer: https://www.insightsonindia.com/2021/06/16/high-level-dialogue-on-
desertification-land-degradation-and-drought-dldd/

139. Consider the following pairs:


Tradition State
1. Shigmo Festival Manipur
2. Chillai Kalan Jharkhand
3. Raja Parba festival Odisha
Which of the above pairs is/are correctly matched?
(a) 1 and 2 only
(b) 3 only
(c) 2 and 3 only
(d) 1 and 3 only
Ans: (b)
Explanation
• S3: Raja Parba festival of Odisha:
o This is a three-day-long festival dedicated to Mother Earth (Bhuma
Devi) and womanhood at large.
o The first day of the festival is called Pahili Raja, the second is Mithuna
Sankranti and the third Bhu daha or Basi Raja.
o The fourth and final day is called Vasumati snana (bathing of Mother
Earth).
o This festival is also associated with the end of the summer season and
the arrival of the monsoon.
• S2: Chillai Kalan:: Kashmir valley read here>>
• S1: Shigmo festival:: Goa read here>>

Refer: facts for prelims: https://www.insightsonindia.com/2021/06/16/high-level-


dialogue-on-desertification-land-degradation-and-drought-dldd/

140. The ‘World Giving Index’ (WGI) is released by which of the following?
(a) Transparency International
(b) Amnesty International
(c) Walk Free Foundation
(d) Charities Aid Foundation
Ans: (d)
Explanation: World Giving Index 2021:
• The World Giving Index (WGI) is an annual report published by the Charities
Aid Foundation. The report is the world’s largest survey of charitable endeavors
from around the world. The first edition was released in September 2010.

Telegram: https://t.me/insightsIAStips
73
Youtube: https://www.youtube.com/channel/UCpoccbCX9GEIwaiIe4HLjwA
Revision Through MCQs (RTM) Compilation (June 2021)

Refer: facts for prelims: https://www.insightsonindia.com/2021/06/16/high-level-


dialogue-on-desertification-land-degradation-and-drought-dldd/

RTM- REVISION THROUGH MCQS – 17th -Jun-2021

141. Consider the following statements about Public Accounts Committee of Indian
Parliament:
1. The PAC is formed every year with a strength of not more than 22 members.
2. As per the panel’s rules, no subject can be deliberated upon till there is a consensus
among all members.
3. Its chief function is to examine the audit report of Comptroller and Auditor General
(CAG) before it is laid in the Parliament.
Which of the given above statements is/are correct?
(a) 1 and 3 only
(b) 1 and 2 only
(c) 2 and 3 only
(d) 1, 2 and 3
Ans: (b)
Explanation:
• S1: The PAC is formed every year with a strength of not more than 22
members of which 15 are from Lok Sabha and 7 from Rajya Sabha.
• S2: As per the panel’s rules, no subject can be deliberated upon till there is a
consensus among all members.
• S3: Its chief function is to examine the audit report of Comptroller and Auditor
General (CAG) after it is laid in the Parliament.

Refer: https://www.insightsonindia.com/2021/06/17/public-accounts-committee-pac/

142. Consider the following statements:


1. Tuberculosis (TB) is caused by a bacterium called Mycobacterium tuberculosis.
2. The bacteria that cause TB are spread when an infected person coughs or sneezes.
Which of the given above statements is/are correct?
(a) 1 only
(b) 2 only
(c) Both 1 and 2
(d) Neither 1 nor 2
Ans: (c)
Explanation:
• TB is an infectious disease caused by the bacillus Mycobacterium tuberculosis.
• It typically affects the lungs (pulmonary TB) but can also affect other sites.
• The disease is spread when people who are sick with pulmonary TB expel
bacteria into the air, for example by coughing.

Refer: https://www.insightsonindia.com/2021/06/17/who-global-tuberculosis-
programme/

Telegram: https://t.me/insightsIAStips
74
Youtube: https://www.youtube.com/channel/UCpoccbCX9GEIwaiIe4HLjwA
Revision Through MCQs (RTM) Compilation (June 2021)

143. In which one of the following groups are all the four countries Dialogue Partners of
ASEAN Defence Ministers Meeting (ADMM) Plus?
(a) China, India, Japan and New Zealand
(b) Australia, China, Democratic People's Republic of Korea and Japan
(c) India, Bangladesh, Russia and the USA
(d) Australia, China, India and Sri Lanka
Ans: (a)
Explanation:
• The ADMM-Plus is a platform for ASEAN and its eight Dialogue Partners to
strengthen security and defence cooperation for peace, stability, and
development in the region.
• Eight Dialogue Partners are Australia, China, India, Japan, New Zealand,
Republic of Korea, Russia and the USA (collectively referred to as the “Plus
Countries”).
• It aims to promote mutual trust and confidence between defence establishments
through greater dialogue and transparency.

Refer: https://www.insightsonindia.com/2021/06/17/asean-defence-ministers-meeting-
plus-admm-plus-3/

144. Consider the following statements about Inland Waterways Authority of India (IWAI):
1. It was constituted under National Waterways Act, 2016.
2. The meetings of the statutory authority is chaired by the Union Home Minister.
Which of the given above statements is/are correct?
(a) 1 only
(b) 2 only
(c) Both 1 and 2
(d) Neither 1 nor 2
Ans: (d)
Explanation:
• Inland Waterways Authority of India (IWAI) is the statutory authority in charge of
the waterways in India.
• It was constituted under IWAI Act-1985 by parliament of India.
• Its headquarters is located in Noida, UP
• Parent department:: Ministry of Ports, Shipping and Waterways

Refer: https://www.insightsonindia.com/2021/06/17/inland-vessels-bill/

145. The Deep Ocean Mission of India consists which of the following components?
1. development of ocean climate change advisory services
2. searching for deep sea flora and fauna
3. explore and identify potential sources of hydrothermal minerals
4. grooming experts in the field of ocean biology and engineering
Select the correct answer using the code below:
(a) 2 and 4 only
(b) 1, 3 and 4 only
(c) 1, 2 and 3 only
(d) 1, 2, 3 and 4
Ans: (d)

Telegram: https://t.me/insightsIAStips
75
Youtube: https://www.youtube.com/channel/UCpoccbCX9GEIwaiIe4HLjwA
Revision Through MCQs (RTM) Compilation (June 2021)

Explanation: The Deep Ocean Mission consists of the following six major component
• Development of Technologies for Deep Sea Mining, and Manned Submersible
• Development of Ocean Climate Change Advisory Services
• Technological innovations for exploration and conservation of deep-sea
biodiversity
• Deep Ocean Survey and Exploration
• Energy and freshwater from the Ocean
• Advanced Marine Station for Ocean Biology

Refer: https://www.insightsonindia.com/2021/06/17/india-set-to-launch-deep-sea-
mission-2/

146. Consider the following statements:


1. India is the largest producer of bananas in the world with a share of around 55% in
total output.
2. In India, Tamil Nadu is the largest producer of bananas in the last five years.
Which of the given above statements is/are correct?
(a) 1 only
(b) 2 only
(c) Both 1 and 2
(d) Neither 1 nor 2
Ans: (d)
Explanation:
• S1: India is the world’s leading producer of bananas with a share of around 25%
in total output.
• S2: Andhra Pradesh stands at the top in area under cultivation, production and
export of bananas in the country. Read more>>

Refer: Facts for Prelims: https://www.insightsonindia.com/2021/06/17/insights-daily-


current-affairs-pib-summary-17-june-2021/

147. Consider the following statements


1. National Highways Authority of India (NHAI) has made mandatory use of drones for
monthly video recording of National Highway projects.
2. All project documentation, contractual decisions and approvals are now being done
through NHAI’s portal ‘Data Lake’ only.
Which of the given above statements is/are correct?
(a) 1 only
(b) 2 only
(c) Both 1 and 2
(d) Neither 1 nor 2
Ans: (c)
Explanation:
• S1: National Highways Authority of India, under Ministry of Road Transport &
Highways has made mandatory use of drones for monthly video recording of
National Highway projects during all stages of development, construction,
operation and maintenance.
• Contractors and Concessionaires shall carry out the drone video recording in
presence of Team Leader of the Supervision Consultant and upload comparative
project videos of the current and last month on NHAI’s portal ‘Data Lake’.

Telegram: https://t.me/insightsIAStips
76
Youtube: https://www.youtube.com/channel/UCpoccbCX9GEIwaiIe4HLjwA
Revision Through MCQs (RTM) Compilation (June 2021)

Refer: Facts for Prelims: https://www.insightsonindia.com/2021/06/17/insights-daily-


current-affairs-pib-summary-17-june-2021/

148. Consider the following statements:


1. Longitude of Dholavira’s location is between those of Lothal and Ganweriwala.
2. Latitude of Harappa’s location is between those of Banawali and Rakhigarhi.
3. Mohenjo-Daro situated more southward than Kalibangan.
Which of the given above statements is/are not correct?
(a) 1 and 2 only
(b) 3 only
(c) 2 and 3 only
(d) 1, 2 and 3
Ans: (a)
Explanation: here the directive word is not correct!!
• Self-explanatory


Refer: Facts for Prelims: https://www.insightsonindia.com/2021/06/17/insights-daily-
current-affairs-pib-summary-17-june-2021/

149. Which one of the following has launched Adi Prashikshan Portal?
(a) NITI Aayog
(b) Ministry of Culture
(c) Ministry of Social Justice and Empowerment
(d) Ministry Of Tribal Affairs
Ans: (d)
Explanation:
• Ministry Of Tribal Affairs Has Launched Adi Prashikshan Portal.

Refer: Facts for Prelims: https://www.insightsonindia.com/2021/06/17/insights-daily-


current-affairs-pib-summary-17-june-2021/

150. Consider the following statements:


1. Lothal, in present-day Rajasthan, was an ancient city of the Indus Valley civilization.
2. The archeological findings discovered at lothal was dockyward.
Which of the given above statements is/are correct?
(a) 1 only
(b) 2 only
(c) Both 1 and 2
(d) Neither 1 nor 2
Ans: (b)

Telegram: https://t.me/insightsIAStips
77
Youtube: https://www.youtube.com/channel/UCpoccbCX9GEIwaiIe4HLjwA
Revision Through MCQs (RTM) Compilation (June 2021)

Explanation:
• S1: Lothal was one of the southernmost cities of the ancient Indus Valley
Civilization located in the Bhāl region of the modern state of Gujarāt.
• S2: Excavations have revealed the world’s oldest known artificial dock, which
was connected to an old course of the Sabarmati River. Other features include
the acropolis, the lower town, the bead factory, the warehouses, and the
drainage system.

Refer: Facts for Prelims: https://www.insightsonindia.com/2021/06/17/insights-daily-


current-affairs-pib-summary-17-june-2021/

Telegram: https://t.me/insightsIAStips
78
Youtube: https://www.youtube.com/channel/UCpoccbCX9GEIwaiIe4HLjwA
Revision Through MCQs (RTM) Compilation (June 2021)

RTM- REVISION THROUGH MCQS – 18th -Jun-2021

151. Consider the following statements


1. The Bureau of Indian Standards (BIS) is the national Standards Body of India
working under the aegis of Ministry of Statistics and Programme Implementation
(MoSPI).
2. In India, the Bureau of Indian Standard (BIS) operates gold and silver hallmarking
scheme.
Which of the given above statements is/are correct?
(a) 1 only
(b) 2 only
(c) Both 1 and 2
(d) Neither 1 nor 2
Ans: (b)
Explanation:
• The Bureau of Indian Standards (BIS) is the national Standards Body of India
working under the aegis of Ministry of Consumer Affairs, Food & Public
Distribution, Government of India.
o BIS is the National Standard Body of India established under the BIS Act
2016 for the harmonious development of the activities of standardization,
marking and quality certification of goods and for matters connected
therewith or incidental thereto
• S2: The Bureau of Indian Standard (BIS) operates gold and silver hallmarking
scheme in India.

Refer: https://www.insightsonindia.com/2021/06/18/what-is-hallmarking-of-gold-and-
for-whom-is-it-now-mandatory/

152. Consider the following statements:


1. Neutrinos are the second most abundant particles in the world.
2. A neutrino’s spin always points in the opposite direction of its motion.
3. Neutrinos can be produced in the lab.
Which of the given above statements is/are correct?
(a) 1 and 2 only
(b) 2 and 3 only
(c) 1 and 3 only
(d) 1, 2 and 3
Ans: (d)
Explanation:
• S1: Detected for the first time in 1959, neutrinos are the second most abundant
particles in the world, after photons, or the light particle.
• S2: A neutrino’s spin always points in the opposite direction of its motion.
• S3: Neutrinos are created by various radioactive decays; during a supernova, by
cosmic rays striking atoms etc.
o They can also be produced in the lab.

Refer: https://www.insightsonindia.com/2021/06/18/neutrinos/

Telegram: https://t.me/insightsIAStips
79
Youtube: https://www.youtube.com/channel/UCpoccbCX9GEIwaiIe4HLjwA
Revision Through MCQs (RTM) Compilation (June 2021)

153. What are ‘Tiangong-1’ and ‘Tiangong-2’, sometimes mentioned in news?


(a) Chines underwater attack submarines
(b) Japanese space robots
(c) South Korea’s indigenous anti-missile programme
(d) None of the above
Ans: (d)
Explanation:
• China has sent two previous space stations into orbit- the Tiangong-1 and
Tiangong-2 were trial stations.

Refer: https://www.insightsonindia.com/2021/06/18/three-chinese-astronauts-enter-
space-station-after-successful-docking-operation/

154. Recently, Twitter has lost its intermediary status in India over non-compliance of the
new IT rules that came into effect on May 26. In this context, consider the following
statements:
1. In India, the intermediary status is a registration granted by the Government of
India.
2. In India, the intermediaries are protected under Section 79 of the Information
Technology Act.
Which of the given above statements is/are correct?
(a) 1 only
(b) 2 only
(c) Both 1 and 2
(d) Neither 1 nor 2
Ans: (b)
Explanation:
• As per Section 2 (1) of the Information Technology Act, an intermediary is a
person/entity that receives, stores and transmits information or provides service
for transmission of information.
• S1: Please note, intermediary status is not a registration granted by the
government.
• S2: Intermediaries like Twitter are protected under Section 79 of the
Information Technology Act that states that they cannot be held liable for
the third party content published on their platform as long as they comply
with the legal order to take down content from courts or other authorities.

Refer: https://www.insightsonindia.com/2021/06/18/what-loss-of-safe-harbour-means-
for-twitter/

155. What is Kerala’s SilverLine project, sometimes mentioned in the news?


(a) It is a flagship semi high-speed railway project
(b) It is an ambitious Fibre Optic Network (K-FON) project
(c) It is a project to create Miyawaki model micro forests in the state
(d) None of the above
Ans: (a)
Explanation:
• It is Kerala’s flagship semi high-speed railway project aimed at reducing travel
time between the state’s northern and southern ends.

Telegram: https://t.me/insightsIAStips
80
Youtube: https://www.youtube.com/channel/UCpoccbCX9GEIwaiIe4HLjwA
Revision Through MCQs (RTM) Compilation (June 2021)

• It links Kerala’s southern end and state capital Thiruvananthapuram with its
northern end of Kasaragod.
• The line is proposed to be 529.45 kms long, covering 11 districts.
• The project is being executed by the Kerala Rail Development Corporation
Limited (KRDCL). KRDCL, or K-Rail, is a joint venture between the Kerala
government and the Union Ministry of Railways.


Refer: facts for prelims: https://www.insightsonindia.com/2021/06/18/insights-daily-
current-affairs-pib-summary-18-june-2021/

156. ‘Desertification and Land Degradation Atlas of India’ has been published by which of
the following?
(a) Department of Science and Technology
(b) Department of Land Resources
(c) National Disaster Management Authority
(d) Indian Space Research Organisation
Ans: (d)
Explanation: Desertification and Land Degradation Atlas of India:
• It has been published by Space Application Centre, ISRO, Ahmedabad.
• The Atlas provides state wise area of degraded lands for the time frame 2018-19.
• It was released on the occasion of the World Day to Combat Desertification
and Drought, observed on 17 June.
• The theme for 2021 is “Restoration. Land. Recovery. We build back better with
healthy land”.

Refer: facts for prelims: https://www.insightsonindia.com/2021/06/18/insights-daily-


current-affairs-pib-summary-18-june-2021/

157. The shy ‘white-bellied heron’ was spotted in the high Himalayas for the first time in
India. In this context, consider the following statements
1. It is categorised as ‘critically endangered’ in International Union for Conservation of
Nature (IUCN) Red Data Book.
2. It is listed in Schedule I in the Wildlife Protection Act, 1972.
3. In India, it is found only in the Namdapha Tiger Reserve.
Which of the given above statements is/are correct?
(a) 1 and 2 only
(b) 1 only
(c) 2 and 3 only
(d) 2 only
Ans: (b)

Telegram: https://t.me/insightsIAStips
81
Youtube: https://www.youtube.com/channel/UCpoccbCX9GEIwaiIe4HLjwA
Revision Through MCQs (RTM) Compilation (June 2021)

Explanation:
• The white-bellied heron is categorised as ‘critically endangered’ in the
International Union for Conservation of Nature (IUCN) Red Data Book and is
listed in Schedule IV in the Wildlife Protection Act, 1972.
• It is one of the rarest birds in the world and is at present found only in Bhutan,
Myanmar and the Namdapha Tiger Reserve in Arunachal Pradesh.
• It had also been recorded in the adjacent Kamlang Tiger Reserve in Lohit
district in camera trap images.

Refer: https://science.thewire.in/environment/rare-white-bellied-heron-spotted-in-
arunachal-pradesh/

158. Consider the following statements:


1. It lies in between Lang River on the North side and famous Namdapha National Park
on its south.
2. This Wildlife Sanctuary contains all four big cats (Tiger, Leopard, Clouded Leopard
and Snow Leopard).
3. Glao or Glow Lake is the most significant water body inside this sanctuary.
The above given statements refers to which one of the following protected area?
(a) Namdapha Tiger Reserve
(b) Kamlang Wildlife Sanctuary
(c) Pakhui Wildlife Sanctuary
(d) Dibang Wildlife Sanctuary
Ans: (b)
Explanation:
• Kamlang Wildlife Sanctuary is situated in the South-Eastern part of Lohit
District of Arunachal Pradesh. The name comes from the River Kamlang, which
flows through the Sanctuary and joins Brahmaputra.

Refer: http://arunachalforests.gov.in/kamlang_wildlife_sanctuary.html

159. Consider the following statements:


1. Western hoolock gibbon is listed as ‘Endangered’ under the IUCN Redlist.
2. Eastern hoolock gibbon is listed as ‘Vulnerable’ under the IUCN Redlist.
Which of the given above statements is/are correct?
(a) 1 only
(b) 2 only
(c) Both 1 and 2
(d) Neither 1 nor 2
Ans: (c)
Explanation:
• Western hoolock gibbon: Endangered under the IUCN Redlist.
• Eastern hoolock gibbon: Vulnerable under the IUCN Redlist.

Refer: https://india.mongabay.com/2021/04/not-two-only-one-species-of-hoolock-gibbon-
in-india-study/

160. What is/are the purpose/purposes of `District Mineral Foundations’ in India?


1. Promoting mineral exploration activities in mineral-rich districts
2. Protecting the interests of the persons affected by mining operations

Telegram: https://t.me/insightsIAStips
82
Youtube: https://www.youtube.com/channel/UCpoccbCX9GEIwaiIe4HLjwA
Revision Through MCQs (RTM) Compilation (June 2021)

3. Authorizing State Governments to issue licences for mineral exploration


Select the correct answer using the code given below.
(a) 1 and 2 only
(b) 2 only
(c) 1 and 3 only
(d) 1, 2 and 3
Ans: (b)
Explanation:
• District Mineral Foundation (DMF) is a trust set up as a non-profit body, in
those districts affected by the mining works, to work for the interest and benefit
of persons and areas affected by mining related operations. It is funded
through the contributions from miners.
• So, all other statements apart from 2 are wrong.
• Setting up of District Mineral Foundations (DMFs) in all districts in the country
affected by mining related operations was mandated through the Mines and
Minerals (Development & Regulation) Amendment Act, (MMDRA) 2015.
• Its manner of operation comes under the jurisdiction of the relevant State
Government.

Refer: UPSC CSE 2016

RTM- REVISION THROUGH MCQS – 19th -Jun-2021


161. With reference to Flag Satyagraha, consider the following statements:
1. It was the first Satyagraha movement led by Sardar Vallabhbhai Patel in India.
2. It contributed to the revival of the indigenous artisan crafts and industries.
Which of the given above statements is/are correct?
(a) 1 only
(b) 2 only
(c) Both 1 and 2
(d) Neither 1 nor 2
Ans: (d)
Explanation:
• What is Flag Satyagraha?
o The Flag Satyagraha movement by the freedom fighters shook the British
government and it infused a new life into the freedom movement.
o Also called the Jhanda Satyagraha, it was held in Jabalpur and Nagpur
in 1923.
o The news of flag hoisting in Jabalpur spread like fire in the country and
after flags were hoisted at several places across the country.
• Significance:
o It is a campaign of peaceful civil disobedience that focused on exercising
the right and freedom to hoist the nationalist flag and challenge the
legitimacy of the British Rule in India through the defiance of laws
prohibiting the hoisting of nationalist flags and restricting civil freedoms.
• Outcomes:
o The arrest of nationalist protestors demanding the right to hoist the flag
caused an outcry across India especially as Gandhi had recently been
arrested.

Telegram: https://t.me/insightsIAStips
83
Youtube: https://www.youtube.com/channel/UCpoccbCX9GEIwaiIe4HLjwA
Revision Through MCQs (RTM) Compilation (June 2021)

o Nationalist leaders such as Sardar Vallabhbhai Patel, Jamnalal Bajaj,


Chakravarthi Rajagopalachari, Dr. Rajendra Prasad and Vinoba Bhave
organised the revolt and thousands of people from different regions
traveled to Nagpur and other parts of the Central Provinces to participate
in civil disobedience.
o In the end, the British negotiated an agreement with Patel and other
Congress leaders permitting the protestors to conduct their march
unhindered and obtaining the release of all those arrested.

Refer: https://www.insightsonindia.com/2021/06/19/flag-satyagraha/

162. Consider the following statements:


1. In the Indian judicial system, there are no definite rules on recusals by Judges.
2. Judicial disqualification is the act of abstaining from participation in an official
action due to a conflict of interest of the administrative officer.
Which of the given above statements is/are correct?
(a) 1 only
(b) 2 only
(c) Both 1 and 2
(d) Neither 1 nor 2
Ans: (c)
Explanation:
• S2: Judicial disqualification, referred to as recusal, is the act of abstaining
from participation in an official action such as a legal proceeding due to a
conflict of interest of the presiding court official or administrative officer.
• S1: There are no definite rules on recusals by Judges. However, in taking
oath of office, judges, both of the Supreme Court and of the high courts, promise
to perform their duties, to deliver justice, “without fear or favour, affection or ill-
will”.

Refer: https://www.insightsonindia.com/2021/06/19/recusal-of-judges-3/

163. Consider the following statements:


1. An election petition can be filed either by a candidate or a voter in the concerned
constituency of the State.
2. An Election Petition has to be filed within 45 days from the declaration of the results
of the State Assembly elections.
3. Election petition can be filed in the High Court of the State to which the State
Assembly elections pertain.
Which of the given above statements is/are correct?
(a) 1 and 2 only
(b) 2 and 3 only
(c) 1 and 3 only
(d) 1, 2 and 3
Ans: (d)
Explanation:
• S1: An election petition can be filed either by a candidate or a voter in the
concerned constituency of the State. Other than these persons, no one can
approach a High Court with an election petition. Voter is a person who was

Telegram: https://t.me/insightsIAStips
84
Youtube: https://www.youtube.com/channel/UCpoccbCX9GEIwaiIe4HLjwA
Revision Through MCQs (RTM) Compilation (June 2021)

entitled to vote at the election to which the election petition relates. Whether he
has voted at such election or not is immaterial. (Sec 81(1) explanation of RPA)
• S2: An Election Petition has to be filed within 45 days from the declaration
of the results of the State Assembly elections.
• S3: Election petition can be filed in the High Court of the State to which the
State Assembly elections pertain. (80A RPA)

Refer: https://www.insightsonindia.com/2021/06/19/how-are-poll-results-challenged-
and-when-courts-have-set-them-aside/

164. The Returning Officers for the State Assembly Elections are appointed by the:
(a) President of India
(b) Governor
(c) State Election Commission
(d) Election Commission of India
Ans: (d)
Explanation:
• A Returning Officer is responsible for overseeing the election in a constituency,
or sometimes in two constituencies, as directed by the Election Commission
(EC).
• The EC appoints the Returning Officer and Assistant Returning Officer for a
constituency in consultation with the governments of the State or Union
Territory as the case may be.
• The Returning Officer’s duties include accepting and scrutinising nomination
forms, publishing the affidavits of candidates, allotting symbols to the contesting
candidates, preparing the list of contesting candidates, preparing the EVMs and
VVPATs, training polling personnel, designating counting centres, and counting
the votes and declaring the result.

Refer: https://www.insightsonindia.com/2021/06/19/how-are-poll-results-challenged-
and-when-courts-have-set-them-aside/

165. Consider the following statements about Ordnance Factory Board (OFB):
1. It is currently a subordinate office of the Ministry of Defence (MoD).
2. Recently, the Union Cabinet has approved a plan to corporatize the Ordnance
Factory Board (OFB).
Which of the following statements is/are correct?
(a) 1 only
(b) 2 only
(c) Both 1 and 2
(d) Neither 1 nor 2
Ans: (c)
Explanation:
• The Union Cabinet has approved a plan to corporatize the Ordnance Factory
Board (OFB).
• Ordnance Factory Board (OFB):
o It is an umbrella body for the ordnance factories and related institutions,
and is currently a subordinate office of the Ministry of Defence (MoD).
o The first Indian ordnance factory was set up in the year 1712 by the
Dutch Company as a GunPowder Factory, West Bengal.

Telegram: https://t.me/insightsIAStips
85
Youtube: https://www.youtube.com/channel/UCpoccbCX9GEIwaiIe4HLjwA
Revision Through MCQs (RTM) Compilation (June 2021)

o OFBs will be responsible for different verticals of the products such as


the Ammunition and Explosives group will be engaged in production of
ammunition while a Vehicles group will engage in production of defence
mobility and combat vehicles.

Refer: https://www.insightsonindia.com/2021/06/19/corporatization-of-ordnance-factory-
board/

166. “Children and Digital Dumpsites” is released by which of the following?


(a) United Nations Children’s Fund (UNICEF)
(b) United Nations Educational, Scientific and Cultural Organization (UNESCO)
(c) World Health Organization (WHO)
(d) World Economic Forum (WEF)
Ans: (c)
Explanation:
• The new report, titled Children and Digital Dumpsites, was recently released
by the WHO.
• Key findings:
o More than 18 million children and adolescents working at e-waste
dumpsites in low- and middle-income countries are potentially at the risk
of severe health hazards.
o They face risk due to discarded electronic devices or e-waste being
dumped from high-income countries.

Refer: https://www.insightsonindia.com/2021/06/19/children-and-digital-dumpsites-
report/

167. In India, ‘extended producer responsibility’ was introduced as an important feature in


which of the following?
(a) The Bio-medical Waste (Management and Handling) Rules, 1998
(b) The Recycled plastic (Manufacturing and Usage) Rules, 1999
(c) The e-Waste (Management and Handling) Rules, 2011
(d) The Food Safety and Standard Regulations, 2011
Ans: (c)
Explanation:
• The e-Waste (Management and Handling) Rules, 2011 recognises producers’
liability for recycling and reducing e-waste in the country.
• Extended Producer Responsibility (EPR) is a policy approach under which
producers are given a significant responsibility – financial and/or physical – for
the treatment or disposal of post-consumer products.
• The Central Pollution Control Board (CPCB) has been given the Extended
Producer Responsibility (EPR) authorisation under the new e-waste rules

Refer: https://www.insightsonindia.com/2021/06/19/children-and-digital-dumpsites-
report/

168. With reference to India State of Forest Report (ISFR) 2019, which of the following
states are showing mangrove cover increase as compared to the previous assessment of
2017?
1. Gujarat
2. Maharashtra

Telegram: https://t.me/insightsIAStips
86
Youtube: https://www.youtube.com/channel/UCpoccbCX9GEIwaiIe4HLjwA
Revision Through MCQs (RTM) Compilation (June 2021)

3. Odisha
Select the correct answer using the code below:
(a) 1 and 2 only
(b) 1 only
(c) 2 and 3 only
(d) 1, 2 and 3
Ans: (d)
Explanation:
• The Mangrove ecosystems are unique & rich in biodiversity and they provide
numerous ecological services.
• Mangrove cover has been separately reported in the ISFR 2019 and the total
mangrove cover in the country is 4,975 sq km.
• An increase of 54 sq Km in mangrove cover has been observed as compared
to the previous assessment of 2017.
• Top three states showing mangrove cover increase are Gujarat (37 sq km)
followed by Maharashtra (16 sq km) and Odisha (8 sq km).4

Refer: https://www.insightsonindia.com/2021/06/19/barrier-to-cyclone-storms-odisha-
plans-to-plant-mangroves-along-its-coast/

169. Consider the following statements about Quality Council of India:


1. It was set up as an autonomous body attached to the Department of Industrial
Policy & Promotion, Ministry of Commerce and Industry.
2. It is headed by the Minister of Commerce and Industry of India.
Which of the given above statements is/are correct?
(a) 1 only
(b) 2 only
(c) Both 1 and 2
(d) Neither 1 nor 2
Ans: (a)
Explanation:
• S1: The Quality Council of India (QCI) was set up as an autonomous body
attached to the Department of Industrial Policy & Promotion, Ministry of
Commerce and Industry.
• S2: Chairman of QCI is appointed by the Prime Minister on recommendation of
the industry to the government.
• Indian Certification of Medical Devices (ICMED) Plus Scheme:
o Launched by Quality Council of India (QCI).
o This is an upgraded ICMED Scheme that was launched for Certification
of Medical Devices in 2016.

Refer: facts for prelims: https://www.insightsonindia.com/2021/06/19/insights-daily-


current-affairs-pib-summary-19-june-2021/

170. Consider the following statements:


1. The Bab-el-Mandeb strait connects the Gulf of Aden with the Red Sea.
2. The Gulf of Aden offers the only entrance from the Arabian Sea and the Indian
Ocean into the Persian Gulf.
Which of the given above statements is/are correct?
(a) 1 only

Telegram: https://t.me/insightsIAStips
87
Youtube: https://www.youtube.com/channel/UCpoccbCX9GEIwaiIe4HLjwA
Revision Through MCQs (RTM) Compilation (June 2021)

(b) 2 only
(c) Both 1 and 2
(d) Neither 1 nor 2
Ans: (a)
Explanation:
• S1: The Bab-el-Mandeb strait connects the Gulf of Aden with the Red Sea.

o
• S2: The Gulf of Aden is a gulf amidst Yemen to the north,the Arabian Sea and
Guardafui Channel to the east, Somalia to the south, and Djibouti to the west.
• S3: The Gulf of Oman offers the only entrance from the Arabian Sea and the
Indian Ocean into the Persian Gulf.

o
Refer: facts for prelims: https://www.insightsonindia.com/2021/06/19/insights-daily-
current-affairs-pib-summary-19-june-2021/

RTM- REVISION THROUGH MCQS – 21st -Jun-2021

171. On 21st June, the Sun


(a) does not set below the horizon at the Arctic Circle
(b) does not set below the horizon at Antarctic Circle
(c) shines vertically overhead at noon on the Equator
(d) shines vertically overhead at the Tropic of Capricorn
Ans: (a)
Explanation:
• June 21- this day is referred to as the summer solstice, the longest day of
the summer season. It occurs when the sun is directly over the Tropic of
Cancer.
• For that hemisphere, the summer solstice is when the Sun reaches its highest
position in the sky and is the day with the longest period of daylight. At the pole,
there is continuous daylight around the summer solstice.

Telegram: https://t.me/insightsIAStips
88
Youtube: https://www.youtube.com/channel/UCpoccbCX9GEIwaiIe4HLjwA
Revision Through MCQs (RTM) Compilation (June 2021)

Refer: https://www.insightsonindia.com/2021/06/21/summer-solstice/

172. With reference to Integrated power development scheme, consider the following
statements:
1. The scheme is spearheaded by the Energy Efficiency Services Limited (EESL).
2. All Discoms are eligible for financial assistance under the scheme.
Which of the given above statements is/are correct?
(a) 1 only
(b) 2 only
(c) Both 1 and 2
(d) Neither 1 nor 2
Ans: (b)
Explanation: The Integrated Power Development Scheme:
• S1: Power Finance Corporation (PFC) is the Nodal agency for implementation of
the scheme.
• S2: All Discoms will be eligible for financial assistance under the scheme.

Refer: https://www.insightsonindia.com/2021/06/21/integrated-power-development-
scheme-ipds/

173. Consider the following statements about Ebola:


1. It is caused by eating food or drinking water contaminated with the Ebola virus.
2. There is no cure or specific treatment for the Ebola virus disease that is currently
approved for market.
Which of the given above statements is/are correct?
(a) 1 only
(b) 2 only
(c) Both 1 and 2
(d) Neither 1 nor 2
Ans: (b)
Explanation:
• S1: Ebola is caused by viruses in the Ebolavirus and Filoviridae family. Ebola is
considered a zoonosis, meaning that the virus is present in animals and is
transmitted to humans.
o Ebola is spread by direct contact with blood or other body fluids
(such as: vomit, diarrhea, urine, breast milk, sweat, semen) of an infected
person who has symptoms of Ebola or who has recently died from Ebola.
• S2: There is no cure or specific treatment for the Ebola virus disease that is
currently approved for market, although various experimental treatments are
being developed. For past and current Ebola epidemics, treatment has been
primarily supportive in nature.

Refer: https://www.insightsonindia.com/2021/06/21/ebola-outbreak/

174. With reference to Republic of the Union of Myanmar, consider the following
statements:
1. The Republic of the Union of Myanmar is a member of the East Asia Summit, Non-
Aligned Movement, and ASEAN, but not a member of the Commonwealth of Nations.
2. It is bordered in the northwest by the Mizoram, Manipur, Nagaland and Arunachal
Pradesh states of India.

Telegram: https://t.me/insightsIAStips
89
Youtube: https://www.youtube.com/channel/UCpoccbCX9GEIwaiIe4HLjwA
Revision Through MCQs (RTM) Compilation (June 2021)

3. The equator passes through water belonging to the Republic of the Union of
Myanmar.
Which of the given above statements is/are correct?
(a) 1 and 3 only
(b) 2 and 3 only
(c) 1 and 2 only
(d) 1, 2 and 3
Ans: (c)
Explanation:
• S1: Myanmar is a member of the East Asia Summit, Non-Aligned Movement,
ASEAN, and BIMSTEC, but it is not a member of the Commonwealth of
Nations.
• S2: Myanmar is bordered in the northwest by the Chittagong Division of
Bangladesh and the Mizoram, Manipur, Nagaland and Arunachal Pradesh
states of India.
• S3: Of the 13 countries that lie on the equator, seven are in Africa—the most of
any continent—and South America is home to three of the nations. The
remaining countries are island nations in the Indian and Pacific oceans.


Refer: https://www.insightsonindia.com/wp-
admin/post.php?post=176718&action=edit&classic-editor

175. Consider the following statements:


1. Monoclonal antibodies plays an important role in hemoglobin production.
2. They can be created in the lab by exposing white blood cells to a particular antigen.
Which of the given above statements is/are correct?
(a) 1 only
(b) 2 only
(c) Both 1 and 2
(d) Neither 1 nor 2
Ans: (b)
Explanation: What are Monoclonal antibodies?
• They are artificially created antibodies that aim to aid the body’s natural
immune system.
• They target a specific antigen — a protein from the pathogen that induces
immune response.
• How are they created?
o Monoclonal antibodies can be created in the lab by exposing white blood
cells to a particular antigen.

Telegram: https://t.me/insightsIAStips
90
Youtube: https://www.youtube.com/channel/UCpoccbCX9GEIwaiIe4HLjwA
Revision Through MCQs (RTM) Compilation (June 2021)

o To increase the quantity of antibodies produced, a single white blood cell


is cloned, which in turn is used to create identical copies of the
antibodies.
o In the case of Covid-19, scientists usually work with the spike protein of
the SARS-CoV-2 virus, which facilitates the entry of the virus into the
host cell.

Refer: https://www.insightsonindia.com/2021/06/21/monoclonal-antibodies/

176. The terms ‘Cry1Ab’ and ‘Cry2Bc’ sometimes mentioned in the news recently are
related to
(a) New variants of Covid-19
(b) Crypto currencies
(c) Sleeping tablets
(d) GM crops
Ans: (d)
Explanation:
• Bt cotton remains the only GM crop allowed to be cultivated in the country.
o Developed by US giant Bayer-Monsanto, it involves insertion of two genes
viz ‘Cry1Ab’ and ‘Cry2Bc’ from the soil bacterium Bacillus
thuringiensis into cotton seeds.
• This modification codes the plant to produce protein toxic to Heliothis bollworm
(pink bollworm) thus making it resistant to their attack. The commercial release
of this hybrid was sanctioned by the government in 2002.

Refer: https://www.insightsonindia.com/2021/06/21/sale-of-illegal-htbt-cotton-seeds-
doubles/

177. Consider the following statements about the Hubble Space Telescope:
1. It is the only telescope designed to be serviced in space by astronauts.
2. It was built by the United States space agency NASA, with contributions from the
German Aerospace Center (DLR).
Which of the given above statements is/are correct?
(a) 1 only
(b) 2 only
(c) Both 1 and 2
(d) Neither 1 nor 2
Ans: (a)
Explanation:
• About the Hubble Space Telescope:
o The Hubble Space Telescope is a large telescope in space. NASA launched
Hubble in 1990.
o It was built by the United States space agency NASA, with
contributions from the European Space Agency.
o Hubble is the only telescope designed to be serviced in space by
astronauts.
o Expanding the frontiers of the visible Universe, the Hubble Space
Telescope looks deep into space with cameras that can see across the
entire optical spectrum from infrared to ultraviolet.

Telegram: https://t.me/insightsIAStips
91
Youtube: https://www.youtube.com/channel/UCpoccbCX9GEIwaiIe4HLjwA
Revision Through MCQs (RTM) Compilation (June 2021)

o The Hubble Space Telescope makes one orbit around Earth every 95
minutes.
• About SOFIA:
o SOFIA is a modified Boeing 747SP jetliner that flies at altitudes up to
45,000 feet.
o It is a joint project of NASA and the German Aerospace Center.
o It has an infrared camera that picks up the wavelength unique to water
molecules.
o SOFIA’s mission is to look at dark and distant objects. The Moon, on the
other hand, is so close and bright that it fills the SOFIA guide camera’s
entire field of view.

Refer: https://www.insightsonindia.com/2021/06/21/hubble-space-telescope/

178. Consider the following statements:


1. Pink hydrogen is generated through electrolysis powered by nuclear energy.
2. Grey Hydrogen is produced from natural gas, where the emissions are captured
using carbon capture and storage.
3. Blue Hydrogen is produced from natural gas where the associated emissions are
released to the air.
Which of the given above statements is/are correct?
(a) 1 only
(b) 2 and 3 only
(c) 1, 2 and 3
(d) 2 only
Ans: (a)
Explanation:
• Brown hydrogen is produced using coal where the emissions are released to the
air
• Grey hydrogen is produced from natural gas where the associated emissions are
released to the air
• Blue hydrogen is produced from natural gas, where the emissions are captured
using carbon capture and storage
• Green hydrogen is produced from electrolysis powered by renewable electricity.
• Pink hydrogen is generated through electrolysis powered by nuclear energy

Refer: https://www.insightsonindia.com/2021/06/21/summit-on-green-hydrogen-
initiatives/

179. The Expert Group under the Chairmanship of Ajit Mishra was in news recently, is
related to which of the following?
(a) Minimum Support Prices
(b) Fair and Remunerative Prices
(c) Fixation of Minimum Wages
(d) Economic Capital Framework
Ans: (c)
Explanation: Ajit Mishra expert group:
• The Central Government has constituted an Expert Group under the
Chairmanship of Professor Ajit Mishra, a renowned economist.

Telegram: https://t.me/insightsIAStips
92
Youtube: https://www.youtube.com/channel/UCpoccbCX9GEIwaiIe4HLjwA
Revision Through MCQs (RTM) Compilation (June 2021)

• Its mandate is to provide technical inputs and recommendations on fixation of


Minimum Wages and National Floor Wages to the Government.

Refer: facts for prelims: https://www.insightsonindia.com/2021/06/21/insights-daily-


current-affairs-pib-summary-21-june-2021/

180. The ‘Biotech-Krishi Innovation Science Application Network’ (Biotech-KISAN) is an


initiative of
(a) Department of Agriculture Cooperation & Farmers Welfare
(b) NITI Aayog
(c) Department of Science and Technology
(d) Ministry of Skill Development and Entrepreneurship
Ans: (c)
Explanation:
• Biotech-Krishi Innovation Science Application Network (Biotech-KISAN) is a
Department of Biotechnology, Ministry of Science and Technology
initiative that empowers farmers, especially women farmers. It aims to
understand the problems of water, soil, seed and market faced by the farmers
and provide simple solutions to them.
• It is a pan-India program, following a hub-and-spoke model and stimulates
entrepreneurship and innovation in farmers and empowers women farmers.
• It identifies and promotes local farm leadership in both genders. Such leadership
helps to develop science-based farming besides facilitating the transfer of
knowledge.

Refer: facts for prelims: https://www.insightsonindia.com/2021/06/21/insights-daily-


current-affairs-pib-summary-21-june-2021/

RTM- REVISION THROUGH MCQS – 22nd -Jun-2021

181. Consider the following statements:


1. The jurisdiction of a High Court can be shifted only through an Act of Parliament.
2. The initiation of the proposal for the transfer of a Judge (including Chief Justice)
from one High Court to any other High Court can be made by the President of India.
Which of the given above statements is/are correct?
(a) 1 only
(b) 2 only
(c) Both 1 and 2
(d) Neither 1 nor 2
Ans: (a)
Explanation:
• S1: The jurisdiction of a High Court can be shifted only through an Act of
Parliament.
o Article 241 states that Parliament may by law constitute a high court for
a Union Territory or declare any court in any such territory to be a high
court for all or any of the purposes of this Constitution.

Telegram: https://t.me/insightsIAStips
93
Youtube: https://www.youtube.com/channel/UCpoccbCX9GEIwaiIe4HLjwA
Revision Through MCQs (RTM) Compilation (June 2021)

• S2: Article 222 of the Constitution makes provision for the transfer of a Judge
(including Chief Justice) from one High Court to any other High Court.
• The initiation of the proposal for the transfer of a Judge should be made by the
Chief Justice of India whose opinion in this regard is determinative.

Refer: https://www.insightsonindia.com/2021/06/22/shifting-of-jurisdiction-of-a-high-
court/

182. Consider the following statements:


1. Under Article 82, the Parliament can enacts a Delimitation Act after every Census.
2. Once the Delimitation Act is in force, President of India sets up the Delimitation
Commission.
Which of the given above statements is/are correct?
(a) 1 Only
(b) 2 Only
(c) Both 1 and 2
(d) Neither 1 nor 2
Ans: (a)
Explanation:
• Delimitation is carried out by an independent Delimitation Commission.
• The Constitution mandates that its orders are final and cannot be questioned
before any court as it would hold up an election indefinitely.
• S1: Under Article 82, the Parliament enacts a Delimitation Act after every
Census.
• S2: Once the Act is in force, the Union government sets up a Delimitation
Commission.

Refer: https://www.insightsonindia.com/2021/06/22/delimitation-in-jammu-and-
kashmir/

183. Consider the following statements about Chief of Defence Staff (CDS):
1. The CDS is a five-star officer selected from among the serving officers of the Indian
Armed Forces.
2. The CDS heads the Department of Military Affairs under the Ministry of Defence, as
its secretary.
Which of the given above statements is/are correct?
(a) 1 only
(b) 2 only
(c) Both 1 and 2
(d) Neither 1 nor 2
Ans: (b)
Explanation:
• The CDS is a four-star officer selected from among the serving officers of the
Indian Armed Forces. While being "first among equals" among the service chiefs,
the CDS is a single-point military advisor to the defence minister.
• The CDS is assisted by a deputy, the Vice Chief of the Defence Staff. The CDS
heads the Department of Military Affairs under the Ministry of Defence, as its
secretary. Apart from heading the DMA, the CDS is the Permanent Chairperson
of the Chiefs of Staff Committee (PC-CoSC).

Telegram: https://t.me/insightsIAStips
94
Youtube: https://www.youtube.com/channel/UCpoccbCX9GEIwaiIe4HLjwA
Revision Through MCQs (RTM) Compilation (June 2021)

Refer: https://www.insightsonindia.com/2021/06/22/chief-of-defence-staff-3/

184. With reference to regulation of films and movies in India, consider the following
statements:
1. At present, there are no enabling provisions to check film piracy.
2. Currently a certificate issued by the Central Board of Film Certification (CBFC) is
valid only for 10 years.
Which of the given above statements is/are correct?
(a) 1 only
(b) 2 only
(c) Both 1 and 2
(d) Neither 1 nor 2
Ans: (c)
Explanation: The draft Cinematograph (Amendment) Bill 2021:
The draft was recently released by the Centre. It seeks to amend the Cinematograph
Act of 1952.
• Key Provisions:
• Revision of certification: This provision will give the Centre “revisionary powers”
and enable it to “re-examine” films already cleared by the Central Board of Film
Certification (CBFC).
• Age-based certification: It seeks to introduce age-based categorisation and
classification. It proposes to divide the existing categories (U, U/A and A) into
further age-based groups: U/A 7+, U/A 13+ and U/A 16+.
• S1: Provision against piracy: At present, there are no enabling provisions to
check film piracy. Violation shall be punishable with imprisonment and fine.
• S2: Eternal certificate: It proposes to certify films for perpetuity. Currently a
certificate issued by the CBFC is valid only for 10 years.

Refer: https://www.insightsonindia.com/2021/06/22/the-draft-cinematograph-
amendment-bill-2021/

185. Consider the following statements about National Initiative for School Heads and
Teachers Holistic Advancement (NISHTHA):
1. It is the largest teacher’s training programme of its kind in the world.
2. It has been launched under the Central Sector Scheme of Samagra Shiksha in
2019-20.
Which of the given above statements is/are correct?
(a) 1 only
(b) 2 only
(c) Both 1 and 2
(d) Neither 1 nor 2
Ans: (a)
Explanation:
• National Initiative for School Heads and Teachers Holistic Advancement
(NISHTHA) is a national mission to improve learning outcomes at the elementary
level. This is done through integrated teacher training.
o It is the largest teachers’ training programme of its kind in the world.
o It has been launched under the Centrally Sponsored Scheme of
Samagra Shiksha in 2109-20.

Telegram: https://t.me/insightsIAStips
95
Youtube: https://www.youtube.com/channel/UCpoccbCX9GEIwaiIe4HLjwA
Revision Through MCQs (RTM) Compilation (June 2021)

Refer: https://www.insightsonindia.com/2021/06/22/nishtha-teachers-training-
programme/

186. The term ‘gain of function research’ is sometimes mentioned in media in reference to:
(a) Human space travel
(b) Ocean thermal energy
(c) Bioremediation
(d) None of the above
Ans: (d)
Explanation:
• In virology, gain-of-function research involves deliberately altering an
organism in the lab, altering a gene, or introducing a mutation in a pathogen to
study its transmissibility, virulence and immunogenicity.
• It is believed that this allows researchers to study potential therapies, vaccine
possibilities and ways to control the disease better in future. “Gain-of-function
research involves manipulations that make certain pathogenic microbes more
deadly or more transmissible. This is done by genetically engineering the
virus and by allowing them to grow in different growth mediums, a technique
called as serial passage,”

Refer: https://www.insightsonindia.com/2021/06/22/what-is-gain-of-function/

187. “Jaan Hai To Jahaan Hai” Campaign was in news recently, is launched by:
(a) Ministry of Health and Family Welfare
(b) Ministry of Social Justice and Empowerment
(c) Ministry of Home Affairs
(d) Ministry of Minority Affairs
Ans: (d)
Explanation:
• “Jaan Hai To Jahaan Hai” Awareness Campaign:
• Launched by the Ministry of Minority Affairs.
• It is a nationwide awareness campaign to create awareness on Corona
vaccination in rural and remote areas of the country and also to “Crush and
Curb” the rumours and apprehensions regarding the on-going vaccination drive.

Refer: Facts for Prelims: https://www.insightsonindia.com/2021/06/22/insights-daily-


current-affairs-pib-summary-22-june-2021/

188. Consider the following statements about Black Softshell Turtle:


1. It is a freshwater species.
2. It is listed as Endangered on the IUCN Red List.
3. In India, hunting of Black Softshell Turtle is prohibited under Schedule I of the
Wildlife Protection Act of 1972.
Which of the given above statements is/are correct?
(a) 1 and 2 only
(b) 1 only
(c) 2 and 3 only
(d) 1, 2 and 3
Ans: (b)
Explanation: Black Softshell Turtle:

Telegram: https://t.me/insightsIAStips
96
Youtube: https://www.youtube.com/channel/UCpoccbCX9GEIwaiIe4HLjwA
Revision Through MCQs (RTM) Compilation (June 2021)

• It is a freshwater species.
• International Union for Conservation of Nature had in 2021 listed the turtle as
‘critically endangered’.
• It does not enjoy legal protection under the Indian Wildlife (Protection) Act
of 1972.
• Until sightings along the Brahmaputra River’s drainage in Assam, the black
softshell turtle was thought to be ‘extinct in the wild’ and confined only to ponds
of temples in northeastern India and Bangladesh.

Refer: facts for prelims: https://www.insightsonindia.com/2021/06/22/insights-daily-


current-affairs-pib-summary-22-june-2021/

189. Consider the following statements about Hayagriva Madhava Temple:


1. It is situated on the Nilachal Hill in western part of Guwahati city in Assam.
2. It is one of the oldest of the 51 Shakti Pithas.
Which of the given above statements is/are correct?
(a) 1 only
(b) 2 only
(c) Both 1 and 2
(d) Neither 1 nor 2
Ans: (d)
Explanation:
• Why in News?
• Hayagriva Madhava Temple Committee in Assam has signed a memorandum of
understanding with two green NGOs, the Assam State Zoo cum Botanical
Garden and the Kamrup district administration for long-term conservation of the
rare freshwater black softshell turtle (Nilssonia nigricans).
• Hayagriva Madhava Temple is situated on the Monikut hill. The hill is situated
in Hajo of Kamrup District in Assam, India.
o The present temple structure was constructed by the King Raghudeva
Narayan in 1583.
o According to some historians the King of Pala dynasty constructed it in
10th century.
o It is a stone temple and it enshrines an image of Hayagriva Madhava.
• Kamakhya Temple:
o It is a Sakta temple dedicated to the mother goddess Kamakhya.
o It is one of the oldest of the 51 Shakti Pithas.
o Situated on the Nilachal Hill in western part of Guwahati city in Assam,
India.

Refer: facts for prelims: https://www.insightsonindia.com/2021/06/22/insights-daily-


current-affairs-pib-summary-22-june-2021/

190. The Genetic Engineering Appraisal Committee is constituted under the


(a) Food Safety and Standards Act, 2006
(b) Geographical Indications of Goods (Registration and Protection) Act, 1999
(c) Environment (Protection) Act, 1986
(d) Wildlife (Protection) Act, 1972
Ans: (c)
Explanation:

Telegram: https://t.me/insightsIAStips
97
Youtube: https://www.youtube.com/channel/UCpoccbCX9GEIwaiIe4HLjwA
Revision Through MCQs (RTM) Compilation (June 2021)

• The Genetic Engineering Appraisal Committee (GEAC) is the apex body


constituted in the Ministry of Environment and Forests under ‘Rules for
Manufacture, Use, Import, Export and Storage of Hazardous
Microorganisms/Genetically Engineered Organisms or Cells 1989’, under
the Environment Protection Act, 1986.
• It is the apex body to accord notified under Rules 1989, for approval of activities
involving large scale use of hazardous microorganisms and recom¬binants in
research and industrial production from the environ¬mental angle. The GEAC is
also responsible for ap¬proval of proposals relating to release of genetically
engineered organisms and products into the environment, including
experimen¬tal field trials (Biosafety Research Level trial-I and II known as BRL-I
and BRL-II).

Refer: https://geacindia.gov.in/about-geac-india.aspx

RTM- REVISION THROUGH MCQS – 23rd-Jun-2021

191. The economic cost of food grains to the Food Corporation of India is Minimum
Support Price and bonus (if any) paid to the farmers plus
(a) transportation cost only
(b) interest cost only
(c) procurement incidentals and distribution cost
(d) procurement incidentals and charges for godowns
Ans: (c)
Explanation:
• The economic cost of food grains procured by the FCI is a total of MSP and
bonus (if any) paid to the farmers plus the procurement incidentals and
distribution cost. See https://www.indiabudget.gov.in/es2004-
05/chapt2005/chap513.pdf
• The procurement incidentals are the initial costs incurred during procurement of
foodgrains. The distribution costs include freight, handling charges, storage
costs in godowns, losses during transit etc.
• Why in news:
o Centre amends Food Security rules to prevent ration leakage,
corruption.
o The government said that this amendment has been made as an attempt
to take forward the reform process envisaged under Section 12 of the
National Food Security Act (NFSA), 2013 by way of improving the
transparency of the operation of the Targeted Public Distribution System
(TPDS) under NFSA.

Refer: https://www.insightsonindia.com/2021/06/23/food-security-assistance-to-state-
government-rules-2015-amended/

192. Suicide worldwide in 2019 report was recently released by


(a) Amnesty International
(b) International Association for Suicide Prevention
(c) World Health Organization

Telegram: https://t.me/insightsIAStips
98
Youtube: https://www.youtube.com/channel/UCpoccbCX9GEIwaiIe4HLjwA
Revision Through MCQs (RTM) Compilation (June 2021)

(d) Walk Free Foundation


Ans: (c)
Explanation:
• Suicide worldwide in 2019 report was recently released by World Health
Organization.

Refer: https://www.insightsonindia.com/2021/06/23/suicide-worldwide-in-2019/

193. Consider the following statements regarding the Nipah virus:


1. It is a bat-borne virus that causes Nipah virus infection.
2. It can be transmitted directly from human-to-human.
3. There is no treatment or vaccine available for either people or animals.
Which of the given above statements is/are correct?
(a) 1 and 2 only
(b) 2 and 3 only
(c) 1 and 3 only
(d) 1, 2 and 3
Ans: (d)
Explanation:
• S1: Nipah virus is a bat-borne virus that causes Nipah virus infection in
humans and other animals, a disease with a high mortality rate.
• S2: Nipah virus can be transmitted to humans from animals (such as bats or
pigs), or contaminated foods and can also be transmitted directly from
human-to-human.
• S3: There is no treatment or vaccine available for either people or animals.
The primary treatment for humans is supportive care.

Refer: https://www.insightsonindia.com/2021/06/23/possible-antibodies-against-nipah-
virus-detected-in-bats-from-mahabaleshwar-cave/

194. Consider the following statements about the United Nations Human Rights Council:
1. It is an inter-governmental body within the United Nations system.
2. It has the ability to discuss all thematic human rights issues and situations that
require its attention throughout the year.
Which of the given above statements is/are correct?
(a) 1 only
(b) 2 only
(c) Both 1 and 2
(d) Neither 1 nor 2
Ans: (c)
Explanation:
• The Human Rights Council is an inter-governmental body within the United
Nations system made up of 47 States responsible for the promotion and
protection of all human rights around the globe.
• It has the ability to discuss all thematic human rights issues and situations
that require its attention throughout the year. It meets at the UN Office at
Geneva.

Refer: https://www.insightsonindia.com/2021/06/23/at-unhrc-grave-concerns-raised-
over-xinjiang/

Telegram: https://t.me/insightsIAStips
99
Youtube: https://www.youtube.com/channel/UCpoccbCX9GEIwaiIe4HLjwA
Revision Through MCQs (RTM) Compilation (June 2021)

195. Consider the following statements:


1. The Great Barrier Reef is the largest coral reef system in the world.
2. The reef is located in the Solomon Sea.
Which of the given above statements is/are correct?
(a) 1 only
(b) 2 only
(c) Both 1 and 2
(d) Neither 1 nor 2
Ans: (a)
Explanation:
• The Great Barrier Reef is the largest coral reef system in the world. The reef is
located off the coast of Queensland, Australia, in the Coral Sea.


Refer: https://www.insightsonindia.com/2021/06/23/unesco-to-downgrade-status-of-
great-barrier-reef-australia-blames-political-interference/

196. Consider the following pairs:


Places in news Country
1. City of Bam Australia
2. Xinjiang China
3. City of Budapest Austria
Which of the given above pairs is/are correct?
(a) 1 and 2 only
(b) 2 only
(c) 2 and 3 only
(d) 1 and 3 only
Ans: (b)
Explanation:
• From Australia's Great Barrier Reef to the city of Budapest [ Hungary’s
capital, is bisected by the River Danube] and the watery wonderland that is
Venice, several beauty spots risk losing their treasured UN World Heritage
status because of environmental damage, excess development or overtourism.

Refer: https://www.insightsonindia.com/2021/06/23/unesco-to-downgrade-status-of-
great-barrier-reef-australia-blames-political-interference/

197. Consider the following statements about the Land for Life Award:
1. It is the United Nation's highest environmental honour.
2. It is organised by UNCCD every two years.

Telegram: https://t.me/insightsIAStips
100
Youtube: https://www.youtube.com/channel/UCpoccbCX9GEIwaiIe4HLjwA
Revision Through MCQs (RTM) Compilation (June 2021)

Which of the given above statements is/are correct?


(a) 1 only
(b) 2 only
(c) Both 1 and 2
(d) Neither 1 nor 2
Ans: (b)
Explanation:
• About the Land for Life Award:
o Launched at the UNCCD COP (Conference of Parties) 10 in 2011.
o It is considered as the world’s highest reward regarding land
conservation and restoration.
o It is organised by UNCCD every two years.
o This year’s theme was “Healthy Land, Healthy Lives”.
• The Champions of the Earth award
o It is the United Nation’s highest environmental honour.
o The United Nations Environment Programme established Champions
of the Earth in 2005 as an annual awards programme to recognize
outstanding environmental leaders from the public and private sectors,
and from civil society.

Refer: https://www.insightsonindia.com/2021/06/23/un-land-conservation-award/

198. Consider the following statements about United Nations Convention to Combat
Desertification:
1. It is the sole legally binding international agreement linking environment and
development to sustainable land management.
2. The UNCCD Secretariat is administered by World Bank.
Which of the given above statements is/are correct?
(a) 1 only
(b) 2 only
(c) Both 1 and 2
(d) Neither 1 nor 2
Ans: (a)
Explanation:
• About United Nations Convention to Combat Desertification:
o Established in 1994.
o It is the sole legally binding international agreement linking
environment and development to sustainable land management
under the UN.
o It is the only convention stemming from a direct recommendation of the
Rio Conference’s Agenda 21.
o Focus areas: The Convention addresses specifically the arid, semi-arid
and dry sub-humid areas, known as the drylands, where some of the
most vulnerable ecosystems and peoples can be found.
• S2: The permanent secretariat of the Convention was established in Article
23 of the UNCCD. It has been located in Bonn, Germany since January 1999.

Refer: https://www.insightsonindia.com/2021/06/23/un-land-conservation-award/

Telegram: https://t.me/insightsIAStips
101
Youtube: https://www.youtube.com/channel/UCpoccbCX9GEIwaiIe4HLjwA
Revision Through MCQs (RTM) Compilation (June 2021)

199. Consider the following statements:


1. Tea grows well in hot and humid climate.
2. Kerala is the largest producer of tea in India.
3. Tea Board of India is a statutory body under the Ministry of Commerce.
Which of the given above statements is/are correct?
(a) 1 only
(b) 2 and 3 only
(c) 1 and 3 only
(d) 1, 2 and 3
Ans: (c)
Explanation:
• S1: Tea is a tropical and sub-tropical plant and grows well in hot and humid
climate.
• S2: West Bengal is the largest producer of tea in India.

o
• S3: Tea Board of India is a statutory body under the Ministry of Commerce.

Refer: facts for prelims: https://www.insightsonindia.com/2021/06/23/insights-daily-


current-affairs-pib-summary-23-june-2021/

200. Consider the following statements:


1. Natural rubber is a native of warm humid Amazon forests.
2. Natural rubber is naturally suited for the colder conditions in the Northeast.
3. Assam is the largest producers of rubber in India.
Which of the given above statements is/are correct?
(a) 1 only
(b) 2 and 3 only
(c) 1 and 2 only
(d) 1, 2 and 3
Ans: (a)
Explanation:
• Natural rubber is a native of warm humid Amazon forests and is not
naturally suited for the colder conditions in the Northeast, which is one of
the largest producers of rubber in India.
• Kerala accounts for more than 90 percent of the total rubber production in the
country.
• World’s 1st GM rubber sapling:

Telegram: https://t.me/insightsIAStips
102
Youtube: https://www.youtube.com/channel/UCpoccbCX9GEIwaiIe4HLjwA
Revision Through MCQs (RTM) Compilation (June 2021)

o World’s first genetically modified (GM) rubber sapling was recently


planted at the Rubber Board’s Sarutari research farm on the outskirts of
Guwahati in Assam.
o It was developed at the Kerala-based Rubber Research Institute of India
(RRII).
o With additional copies of the gene MnSOD (manganese-containing
superoxide dismutase) inserted in it, the GM rubber is expected to tide
over the severe cold conditions during winter, which is a major factor
affecting the growth of rubber saplings.
o MnSOD gene used in the GM rubber was taken from the rubber plant
itself.

Refer: facts for prelims: https://www.insightsonindia.com/2021/06/23/insights-daily-


current-affairs-pib-summary-23-june-2021/

Telegram: https://t.me/insightsIAStips
103
Youtube: https://www.youtube.com/channel/UCpoccbCX9GEIwaiIe4HLjwA
Revision Through MCQs (RTM) Compilation (June 2021)

RTM- REVISION THROUGH MCQS – 24th -Jun-2021

201. With reference to first-past-the-post (FPTP) voting method, consider the following
statements:
1. It is also known as the simple majority system.
2. In this voting method, the candidate with the highest number of votes in a
constituency is declared the winner.
3. This system is used in India in direct elections to the Lok Sabha and State
Legislative Assemblies.
Which of the given above statements is/are correct?
(a) 1 and 2 only
(b) 2 and 3 only
(c) 1 and 3 only
(d) 1, 2 and 3
Ans: (d)
Explanation:
• The first-past-the-post (FPTP) system is also known as the simple majority
system.
• In this voting method, the candidate with the highest number of votes in a
constituency is declared the winner.
• This system is used in India in direct elections to the Lok Sabha and State
Legislative Assemblies.
• While FPTP is relatively simple, it does not always allow for a truly representative
mandate, as the candidate could win despite securing less than half the votes in
a contest. In 2014, the National Democratic Alliance led by the Bharatiya Janata
Party won 336 seats with only 38.5% of the popular vote. Also, smaller parties
representing specific groups have a lower chance of being elected in FPTP.

Refer: https://www.insightsonindia.com/2021/06/24/what-is-ranked-choice-voting-
which-made-its-debut-in-new-york-mayoral-polls/

202. Consider the following statements about ‘Information Fusion Centre for Indian Ocean
Region’ (IFC-IOR):
1. It is the single point center linking all the coastal radar chains networks along the
Indian coastline and in some neighbouring countries.
2. It will serve countries that have white (commercial ships) and Grey (military vessels)
Shipping Information Exchange agreements with India.
Which of the given above statements is/are correct?
(a) 1 only
(b) 2 only
(c) Both 1 and 2
(d) Neither 1 nor 2
Ans: (a)
Explanation:
• The IFC-IOR was set up in 2018 to coordinate with regional countries on
maritime issues and act as a regional repository of maritime data.
• It presently has linkages with 21 partner countries and 22 multi-national
agencies across the globe.

Telegram: https://t.me/insightsIAStips
104
Youtube: https://www.youtube.com/channel/UCpoccbCX9GEIwaiIe4HLjwA
Revision Through MCQs (RTM) Compilation (June 2021)

• It is located in Gurugram, India.


• The Information Fusion Centre will serve countries that have White
Shipping Information Exchange agreements with India.
o India already has bilateral White Shipping Agreements with 36
countries.

Refer: https://www.insightsonindia.com/2021/06/24/information-fusion-centre-for-
indian-ocean-region-ifc-ior/

203. Recently, which one of the following nation’s ‘Tax Inspectors without Borders’ (TIWB)
programme launched in partnership with India?
(a) Nepal
(b) Pakistan
(c) Bhutan
(d) Myanmar
Ans: (c)
Explanation:
• Bhutan’s Tax Inspectors Without Borders (TIWB) programme launched in
partnership with India.
o This programme is expected to be of about 24 months’ duration.
o The focus of the programme will be in the area of International Taxation
and Transfer Pricing.

Refer: https://www.insightsonindia.com/2021/06/24/tax-inspectors-without-borders-
tiwb-programme/

204. The ‘Tax Inspectors Without Borders’ (TIWB) programme is a joint initiative of:
1. The United Nations Development Programme (UNDP)
2. The Organisation for Economic Cooperation and Development (OECD)
3. The World Customs Organization (WCO)
Select the correct answer using the code below:
(a) 1 and 2 only
(b) 2 and 3 only
(c) 1 and 3 only
(d) 1, 2 and 3
Ans: (a)
Explanation: About Tax Inspectors Without Borders (TIWB) programme:
• It is a joint initiative of the United Nations Development Programme (UNDP)
and the Organisation for Economic Cooperation and Development (OECD).
• The objective of the TIWB Initiative is to enable sharing of tax audit knowledge
and skills with tax administrations in developing countries through a targeted,
real time “learning by doing” approach.
• TIWB is focused on promoting hands-on assistance by sending Experts to build
audit and audit-related skills pertaining to specific international tax matters and
the development of general audit skills within developing tax administrations.

Refer: https://www.insightsonindia.com/2021/06/24/tax-inspectors-without-borders-
tiwb-programme/

Telegram: https://t.me/insightsIAStips
105
Youtube: https://www.youtube.com/channel/UCpoccbCX9GEIwaiIe4HLjwA
Revision Through MCQs (RTM) Compilation (June 2021)

205. Consider the following statements about Competition Commission Of India:


1. It is a constitutional body of the Government of India.
2. It consists of a Chairperson and not more than ten Members appointed by the
Central Government.
Which of the given above statements is/are correct?
(a) 1 only
(b) 2 only
(c) Both 1 and 2
(d) Neither 1 nor 2
Ans: (d)
Explanation:
• Competition Commission of India is the competition regulator in India.
• It is a statutory body of the Government of India responsible for enforcing The
Competition Act, 2002 and promoting competition throughout India and to
prevent activities that have an appreciable adverse effect on competition in India.
• CCI consists of a Chairperson and 6 Members appointed by the Central
Government.

Refer: https://www.insightsonindia.com/2021/06/24/cci-probes-google-for-unfair-
business-practices/

206. Consider the following statements about the Scientific Advisory Board of the United
Nations Secretary-General:
1. It is an intergovernmental body of the United Nations.
2. It is composed of representatives appointed by governments.
3. It mainly produces reports that contribute to the work of the United Nations
Framework Convention on Climate Change (UNFCCC).
Which of the given above statements is/are not correct?
(a) 1 and 3 only
(b) 1 only
(c) 2 and 3 only
(d) 1, 2 and 3
Ans: (d)
Explanation: here the directive word is not correct!!
• S1: The UN Secretary-General announced the creation of the Scientific
Advisory Board on 24 September 2013, during the inaugural meeting of the
High-Level Political Forum on Sustainable Development.
• S2: It will comprise renowned scientists representing various fields of
natural, social and human sciences.
• S3: The central function of the Board will be to provide advice on science,
technology and innovation (STI) for sustainable development to the UN
Secretary-General and to Executive Heads of UN organizations.

Refer: https://www.insightsonindia.com/2021/06/24/climate-crisis-to-hit-sooner-than-
feared/

207. Consider the following statements:


1. The Suez Canal is a natural sea-level waterway running north to south across the
Isthmus of Suez.
2. The Canal connects the Mediterranean Sea and the Red Sea.

Telegram: https://t.me/insightsIAStips
106
Youtube: https://www.youtube.com/channel/UCpoccbCX9GEIwaiIe4HLjwA
Revision Through MCQs (RTM) Compilation (June 2021)

Which of the given above statements is/are correct?


(a) 1 only
(b) 2 only
(c) Both 1 and 2
(d) Neither 1 nor 2
Ans: (b)
Explanation:
• The Suez Canal is an artificial sea-level waterway running north to south
across the Isthmus of Suez in Egypt, to connect the Mediterranean Sea and
the Red Sea.
• The canal separates the African continent from Asia.


Refer: facts for prelims: https://www.insightsonindia.com/2021/06/24/insights-daily-
current-affairs-pib-summary-24-june-2021/

208. Consider the following statements about Pygmy hog (Porcula salvania):
1. It is endemic to the Western Ghats of India.
2. It is listed as ‘Critically Endangered’ on the IUCN Red List of Threatened Species.
3. It is one of the very few mammals that build its own home.
Which of the given above statements is/are correct?
(a) 1 and 2 only
(b) 2 and 3 only
(c) 1 and 3 only
(d) 1, 2 and 3
Ans: (b)
Explanation: About Pygmy hogs:
• They are the world’s rarest and smallest wild pigs.
• S1: The pygmy hog is native to dense alluvial grasslands in the southern
foothills of the Himalayas.
o Endemic to India, they are restricted to very few locations around
Manas National Park in north-western Assam.
• With just around 250 animals in the wild, the pygmy hog is one of the world’s
most threatened mammals.
• S2: Currently listed as ‘Critically Endangered’ on the IUCN Red List of
Threatened Species.

Telegram: https://t.me/insightsIAStips
107
Youtube: https://www.youtube.com/channel/UCpoccbCX9GEIwaiIe4HLjwA
Revision Through MCQs (RTM) Compilation (June 2021)

• The pygmy hog is designated as a Schedule I species in India under the Wildlife
Protection Act, 1972.
• S3: The pygmy hog is one of the very few mammals that build its own home, or
nest, complete with a ‘roof’, but maybe more important than its own
attributes, is the fact that it is an indicator species. Its presence reflects the
health of its primary habitat, the tall, wet grasslands of the region

Refer: facts for prelims: https://www.insightsonindia.com/2021/06/24/insights-daily-


current-affairs-pib-summary-24-june-2021/

209. In which one of the following States is Barnadi Wildlife Sanctuary located?
(a) Assam
(b) Odisha
(c) Nagaland
(d) Kerala
Ans: (a)
Explanation:
• The smallest member of the pig family, the pygmy hog (porcula salvania), is a
critically endangered species. Once found along a narrow strip of tall and wet
grassland plains on the Himalayan foothills – from Uttar Pradesh to Assam,
through Nepal’s terai areas and Bengal’s duars – it was thought to have become
extinct in the 1960s. But in 1971 it was “re-discovered” with a small
population in the Barnadi Wildlife Sanctuary.
o It is situated on the foothills of Himalayas bordering Bhutan in the north
and in Udalguri district & Baksa District of Assam. This sanctuary is
named after the river Bornadi which flows on its western border.

Refer: facts for prelims: https://www.insightsonindia.com/2021/06/24/insights-daily-


current-affairs-pib-summary-24-june-2021/

210. With reference to India’s Manas National Park, consider the following statements:
1. It is spread over four districts.
2. It is one of the natural habitats of red giant flying squirrel.
3. It has the largest population of the endangered Bengal florican to be found anywhere
in India.
Which of the given above statements is/are correct?
(a) 1 and 2 only
(b) 3 only
(c) 2 and 3 only
(d) 1, 2 and 3
Ans: (b)
Explanation:
• S1: The park area falls in the following districts: Chirang, Baksa in the
autonomous territorial region, i.e BTR in the state of Assam in India.
• S2: The red giant flying squirrel is often seen in Namdapha National Park.
o It is endemic to the park and critically endangered.
• S3: It has the largest population of the critically endangered Bengal florican to
be found anywhere.

Telegram: https://t.me/insightsIAStips
108
Youtube: https://www.youtube.com/channel/UCpoccbCX9GEIwaiIe4HLjwA
Revision Through MCQs (RTM) Compilation (June 2021)

Refer: facts for prelims: https://www.insightsonindia.com/2021/06/24/insights-daily-


current-affairs-pib-summary-24-june-2021/

RTM- REVISION THROUGH MCQS – 24th -Jun-2021

211. Consider the following statements:


1. The Electoral Trusts Scheme, 2013 was notified by the Election Commission of
India.
2. The sole object of the electoral trust is to distribute funds only to the eligible political
parties in India.
Which of the given above statements is/are correct?
(a) 1 only
(b) 2 only
(c) Both 1 and 2
(d) Neither 1 nor 2
Ans: (b)
Explanation: About Electoral Trusts Scheme, 2013:
• Electoral Trust is a non-profit organization formed in India for orderly receiving
of the contributions from any person.
• The scheme was notified by the Central Board of Direct Taxes (CBDT).
• Objectives of the Scheme: To lay down a procedure for grant of approval to an
electoral trust which will receive voluntary contributions and distribute the same
to the political parties.
• The sole object of the electoral trust is to distribute the contributions
received by it to the political party, registered under section 29A of the
Representation of the People Act, 1951.
• These Electoral Trust companies are not allowed to accept contributions from
foreign citizens or companies.
• The trust shall also maintain a list of persons from whom contributions have
been received and to whom the same have been distributed.

Refer: https://www.insightsonindia.com/2021/06/25/electoral-trust-declares-donation-of-
rs-3-crore-through-electoral-bonds/

212. Mission Karmayogi was in news recently, is related to which of the following?
(a) Agriculture Reforms
(b) Women empowerment
(c) Socio-Economic empowerment of tribal people
(d) Reform in Indian Bureaucracy
Ans: (d)
Explanation: About Mission Karmayogi:
• The ‘National Programme for Civil Services Capacity Building – Mission
Karmayogi’ was launched to effect a transformational shift from rule based
training to role-based capacity development of all civil services in the country
so as to enhance citizen experience for government services and improve
availability of competent workforce.

Refer: https://www.insightsonindia.com/2021/06/25/task-force-for-mission-karmayogi/

Telegram: https://t.me/insightsIAStips
109
Youtube: https://www.youtube.com/channel/UCpoccbCX9GEIwaiIe4HLjwA
Revision Through MCQs (RTM) Compilation (June 2021)

213. In the context of India which one of the following is the characteristic appropriate for
Bureaucracy?
(a) An agency for widening the scope for parliamentary democracy
(b) An agency for strengthening the structure of federalism
(c) An agency for facilitating the political stability and economic growth
(d) An agency for the implementation of public policy
Ans: (d)
Explanation:
• Public policy formulation and policy implementation are two distinct but closely
interrelated functions of the government. Public policy is laid down by the
legislature or the political authorities, who are vested with the power of giving
policy the requisite legal authority i.e. legitimacy. The policy implementation
aspect is supposed to be in the domain of the executive, i.e., the
bureaucracy or the administrative arm of the government.
• Read more>>

Refer: https://www.insightsonindia.com/2021/06/25/task-force-for-mission-karmayogi/

214. Consider the following statements:


1. Peter Pan syndrome is a mental health condition that can affect one’s quality of life.
2. The World Health Organization does not recognise Peter Pan Syndrome as a health
disorder.
Which of the given above statements is/are correct?
(a) 1 only
(b) 2 only
(c) Both 1 and 2
(d) Neither 1 nor 2
Ans: (c)
Explanation: what is Peter Pan syndrome?
• The syndrome is named after a 1900s fictional character. Peter Pan is a care-
free young boy, who never grows up. The character was created by Scottish
novelist James Matthew Barrie.
• It is said that people who develop similar behaviours — of living life carefree,
finding responsibilities challenging in adulthood, and basically, “never growing
up” — suffer from Peter Pan Syndrome.
• Please note that this syndrome has not been recognised by WHO as a health
disorder.
• It affects people who do not want or feel unable to grow up, people with the body
of an adult but the mind of a child. They don’t know how to or don’t want to stop
being children and start being mothers or fathers.

Refer: https://www.insightsonindia.com/2021/06/25/peter-pan-syndrome-pps/

215. Consider the following statements about Shanghai Cooperation Organisation (SCO):
1. It is an intergovernmental military alliance.
2. China is the supreme decision-making body in the SCO.
3. The SCO Secretariat is located in Tashkent.
Which of the given above statement is/are not correct?
(a) 1 and 2 only
(b) 3 only

Telegram: https://t.me/insightsIAStips
110
Youtube: https://www.youtube.com/channel/UCpoccbCX9GEIwaiIe4HLjwA
Revision Through MCQs (RTM) Compilation (June 2021)

(c) 2 and 3 only


(d) 1, 2 and 3
Ans: (d)
Explanation: here the directive word is not correct!!
• S1: the SCO has repeatedly stressed that it is not a military alliance with
treaty obligations.
o It is a permanent intergovernmental international organization.
• S2: The Heads of State Council (HSC) is the supreme decision-making body in
the SCO. It meets once a year and adopts decisions and guidelines on all
important matters of the organisation.
• S3: The organisation has two permanent bodies — the SCO Secretariat based in
Beijing and the Executive Committee of the Regional Anti-Terrorist Structure
(RATS) based in Tashkent.

Refer: https://www.insightsonindia.com/2021/06/25/sco-meet/

216. With reference to India’s Chandrayaan-2 mission, consider the following statements:
1. It comprised an Orbiter, Lander and Rover to explore the unexplored North Pole of
the Moon.
2. The primary objective of mission was to demonstrate the ability to soft-land on the
lunar surface and operate a robotic rover on the surface.
Which of the given above statements is/are correct?
(a) 1 only
(b) 2 only
(c) Both 1 and 2
(d) Neither 1 nor 2
Ans: (b)
Explanation:
• S1: It comprised an Orbiter, Lander and Rover to explore the unexplored South
Pole of the Moon.
• S2: The primary objective of Chandrayaan 2 was to demonstrate the ability to
soft-land on the lunar surface and operate a robotic rover on the surface.

Refer: https://www.insightsonindia.com/2021/06/25/chandrayaan-2-5/

217. Consider the following statements about the Antarctic Treaty:


1. The primary purpose of the Antarctic Treaty is to completely prohibit military
activities in the Antarctic region.
2. The Treaty applies to the area south of 60° South Latitude, including all ice shelves
and islands.
Which of the given above statements is/are correct?
(a) 1 only
(b) 2 only
(c) Both 1 and 2
(d) Neither 1 nor 2
Ans: (b)
Explanation:
• The primary purpose of the Antarctic Treaty is to ensure "in the interests of
all mankind that Antarctica shall continue forever to be used exclusively for
peaceful purposes and shall not become the scene or object of international

Telegram: https://t.me/insightsIAStips
111
Youtube: https://www.youtube.com/channel/UCpoccbCX9GEIwaiIe4HLjwA
Revision Through MCQs (RTM) Compilation (June 2021)

discord." To this end it prohibits military activity, except in support of


science; prohibits nuclear explosions and the disposal of nuclear waste;
promotes scientific research and the exchange of data; and holds all territorial
claims in abeyance. The Treaty applies to the area south of 60° South
Latitude, including all ice shelves and islands.

Refer: https://www.insightsonindia.com/2021/06/25/antarctic-treaty/

218. Consider the following statements:


1. The Black Sea is bordered by Bulgaria, Georgia, Romania, Russia, Turkey, and
Ukraine.
2. The Black Sea ultimately drains into the Mediterranean Sea, via the Strait of Kerch
and the Aegean Sea.
Which of the given above statements is/are correct?
(a) 1 only
(b) 2 only
(c) Both 1 and 2
(d) Neither 1 nor 2
Ans: (a)
Explanation;
• S1: The Black Sea is bordered by Bulgaria, Georgia, Romania, Russia, Turkey,
and Ukraine.
• S2: The Black Sea ultimately drains into the Mediterranean Sea, via the Turkish
Straits and the Aegean Sea. The Bosporus Strait connects it to the small Sea
of Marmara which in turn is connected to the Aegean Sea via the Strait of the
Dardanelles. To the north, the Black Sea is connected to the Sea of Azov by
the Kerch Strait.


Refer: Facts for Prelims: https://www.insightsonindia.com/2021/06/25/insights-daily-
current-affairs-pib-summary-25-june-2021/

219. Consider the following pairs


1. Nokrek Bio-sphere Reserve: Garo Hills
2. Loktak Lake: Barail Range
3. Namdapha National Park: Dafla Hills
Which of the above pairs is/are correctly matched?
(a) 1 only
(b) 2 and 3 only
(c) 1, 2 and 3
(d) None
Ans: (a)
Explanation:

Telegram: https://t.me/insightsIAStips
112
Youtube: https://www.youtube.com/channel/UCpoccbCX9GEIwaiIe4HLjwA
Revision Through MCQs (RTM) Compilation (June 2021)

• Nokrek National Park, the core area of Nokrek Biosphere Reserve, is a national
park located approximately 2 km from Tura Peak in West Garo Hills district of
Meghalaya, India. UNESCO added this National park to its list of Biosphere
Reserves in May 2009.
• Logtak Lake is in Manipur; Barail Range is in Assam.
• Though Namdapha National Park and Dafla Hill both are in Arunachal Pradesh,
the two are separate entities.

Refer: http://www.megforest.gov.in/wildlife_parks.html

220. Garibi hatao slogan was introduced in which of the following Five Year Plan?
(a) Fourth
(b) Fifth
(c) Second
(d) First
Ans: (b)
Explanation:
• Garibi Hatao (“Remove poverty”) was the theme and slogan of Indira Gandhi’s
1971 election campaign.
• The slogan and the proposed anti-poverty programs that came with it were
designed to give Gandhi an independent national support, based on rural and
urban poor, which would allow her to by-pass the dominant rural castes both in
and out of state and local government.
• The programs created through garibi hatao, though carried out locally, were
funded, developed, supervised, and staffed by Government officials in New Delhi
and Congress Party officials. It was part of the 5th Five-Year Plan.

RTM- REVISION THROUGH MCQS – 26th -Jun-2021

221. With reference to cultural history of India, consider the following statements :
1. Kabir Das' writings had a great influence on the Bhakti movement.
2. The major part of Kabir’s work was collected by the Guru Hargobind.
3. Kabir Das’ ideologies were greatly influenced by Vaishnava saint Swami
Ramananda.
4. Kabir Das and Data Ganj Bakhsh are contemporaries.
Which of the statements given above are correct?
(a) 1 and 3 only
(b) 2 and 4 only
(c) 1, 2 and 3
(d) 1, 2, 3 and 4
Ans: (a)
Explanation:
• S1 and S2: Kabir Das' writings had a great influence on the Bhakti movement
and includes titles like Kabir Granthawali, Anurag Sagar, Bijak, and Sakhi
Granth. The major part of his work was collected by the fifth Sikh guru--Guru
Arjan Dev.
• S3: Swami Ramananda influence: Kabir Das’ ideologies were greatly influenced
by Vaishnava saint Swami Ramananda who accepted Kabir as his disciple.

Telegram: https://t.me/insightsIAStips
113
Youtube: https://www.youtube.com/channel/UCpoccbCX9GEIwaiIe4HLjwA
Revision Through MCQs (RTM) Compilation (June 2021)

• S4: Data Ganj Bakhsh was an 11th-century Persian Sunni Muslim mystic and
theologian.

Refer: https://www.insightsonindia.com/2021/06/26/sant-kabir-das-jayanti/

222. Currently, the “Enforcement Directorate” is operating under the jurisdiction of the:
(a) Ministry of Home Affairs
(b) Ministry of Parliamentary Affairs
(c) Ministry of Finance
(d) Ministry of Personnel, Public Grievances and Pensions
Ans: (c)
Explanation: About Enforcement Directorate:
• The origin of this Directorate goes back to 1st May, 1956, when an
‘Enforcement Unit’ was formed, in the Department of Economic Affairs, for
handling Exchange Control Laws violations under Foreign Exchange
Regulation Act, 1947 (FERA ’47).
• In the year 1957, this Unit was renamed as ‘Enforcement Directorate’.
• Currently, it is part of the Department of Revenue, Ministry of Finance.
• The Organization is mandated with the task of enforcing the provisions of two
special fiscal laws – Foreign Exchange Management Act, 1999 (FEMA) and
Prevention of Money Laundering Act, 2002 (PMLA).

Refer: https://www.insightsonindia.com/2021/06/26/ed-transfers-assets-worth-
%e2%82%b98441-50-cr-to-banks/

223. Consider the following statements about ‘Gujarat International Finance Tec-City’
(GIFT City):
1. It is a business district near Ahmedabad in Gujarat, India.
2. It is India's first operational Greenfield smart city and international financial
services centre.
3. The city is located on the banks of the Sabarmati River.
Which of the given above statements is/are correct?
(a) 1 and 2 only
(b) 1 and 3 only
(c) 2 and 3 only
(d) 1, 2 and 3
Ans: (d)
Explanation:
• S1 and S2: It is a business district near Ahmedabad in Gujarat, India. It is
India's first operational Greenfield smart city and international financial
services center.
• S3: The city is located on the banks of the Sabarmati River.

Refer: https://www.insightsonindia.com/2021/06/26/gujarat-international-maritime-
arbitration-centre-gimac/

224. Consider the following statements about the International Labour Organisation (ILO):
1. It was established as an agency for the League of Nations following World War I.
2. It became the first specialised agency of the United Nations (UN) in the year 1946.
3. It is the only tripartite U.N. agency.

Telegram: https://t.me/insightsIAStips
114
Youtube: https://www.youtube.com/channel/UCpoccbCX9GEIwaiIe4HLjwA
Revision Through MCQs (RTM) Compilation (June 2021)

Which of the given above statements is/are correct?


(a) 1 and 2 only
(b) 2 and 3 only
(c) 1 and 3 only
(d) 1, 2 and 3
Ans: (d)
Explanation:
• Established as an agency for the League of Nations following World War I.
• Established by the Treaty of Versailles in 1919.
• It became the first specialised agency of the United Nations (UN) in the year
1946.
• It got the Nobel Peace Prize in 1969.
• It is the only tripartite U.N. agency. It brings together governments, employers
and workers.
• Headquarters: Geneva, Switzerland.

Refer: https://www.insightsonindia.com/2021/06/26/india-completes-term-as-chair-of-
ilos-governing-body/

225. Consider the following statements about Financial Action Task Force (FATF):
1. It is an inter-governmental body established in 1989 on the initiative of the
Organisation for Economic Co-operation and Development (OECD).
2. Initially it was established to examine and develop measures to combat terrorist
financing.
Which of the given above statements is/are correct?
(a) 1 only
(b) 2 only
(c) Both 1 and 2
(d) Neither 1 nor 2
Ans: (d)
Explanation:
• S1: The Financial Action Task Force (FATF) is an inter-governmental body
established in 1989 on the initiative of the G7.
• S2: Initially it was established to examine and develop measures to combat
money laundering.
o In October 2001, the FATF expanded its mandate to incorporate efforts to
combat terrorist financing, in addition to money laundering.
o In April 2012, it added efforts to counter the financing of proliferation
of weapons of mass destruction.

Refer: https://www.insightsonindia.com/2021/06/26/pakistan-to-remain-on-fatf-grey-list-
3/

226. Consider the following statements regarding ‘Maritime State Development Council’
(MSDC):
1. It is an apex advisory body for the development of the Maritime sector in India.
2. It monitors the development of minor ports, captive ports and private ports in the
Maritime States.
Which of the given above statement is/are correct?
(a) 1 only

Telegram: https://t.me/insightsIAStips
115
Youtube: https://www.youtube.com/channel/UCpoccbCX9GEIwaiIe4HLjwA
Revision Through MCQs (RTM) Compilation (June 2021)

(b) 2 only
(c) Both 1 and 2
(d) Neither 1 nor 2
Ans: (c)
Explanation:
• MSDC is an apex advisory body for the development of the Maritime sector
and aims to ensure integrated development of Major and non-Major Ports. The
MSDC was constituted in May 1997 to assess in consultation with State
Governments, the future development of existing and new Minor Ports by the
respective Maritime States either directly or through captive users and private
participation. Further, MSDC also monitors the development of minor ports,
captive ports and private ports in the Maritime States with a view to ensure
their integrated development with Major Ports and to assess the requirements of
other infrastructure requirements like roads/rail/IWT and make suitable
recommendations to the concerned Ministers.

Refer: Facts for Prelims: https://www.insightsonindia.com/2021/06/26/insights-daily-


current-affairs-pib-summary-26-june-2021/

227. The ‘Project Seabird’ was in news recently, is related to which of the following?
(a) ship breaking/recycling
(b) construction of naval base
(c) indigenous submarine construction
(d) deep ocean exploration
Ans: (b)
Explanation: Project Seabird:
• It is the largest naval infrastructure project for India.
• The project aims at creation of a naval base at Karwar on the west coast of
India.

Refer: Facts for Prelims: https://www.insightsonindia.com/2021/06/26/insights-daily-


current-affairs-pib-summary-26-june-2021/

228. Which of the following is geographically closest to Arunachal Pradesh?


(a) Lhasa
(b) Nyingchi
(c) Shigatse
(d) Nagqu
Ans: (b)
Explanation:
• China has operationalised its first fully electrified bullet train in the remote
Himalayan region of Tibet.
• It connects the provincial capital Lhasa and Nyingchi, a strategically located
Tibetan border town close to Arunachal Pradesh.
• This line is a section of the Sichuan-Tibet Railway.
• The rail line crosses the Brahmaputra river locally called Yarlung Zangbo 16
times.

Telegram: https://t.me/insightsIAStips
116
Youtube: https://www.youtube.com/channel/UCpoccbCX9GEIwaiIe4HLjwA
Revision Through MCQs (RTM) Compilation (June 2021)


Refer: Facts for Prelims: https://www.insightsonindia.com/2021/06/26/insights-daily-
current-affairs-pib-summary-26-june-2021/

229. Consider the following statements with reference to river Brahmaputra,


1. The source of river Brahmaputra is the same as that of Teesta but it flows through
Assam.
2. It flows as Jamuna River before emptying into the Bay of Bengal.
3. The ‘Zayu River’ is a tributary of Brahmaputra River.
Which of the statements given above is/are correct?
(a) 2 only
(b) 1 and 2 only
(c) 3 only
(d) 1, 2 and 3
Ans: (c)
Explanation:
• S1: Teesta River originates from the Pahunri glacier. Brahmaputra originates
in Angsi glacier/ Chemayungdung Glacier. Teesta is a tributary of
Brahmaputra.
• S2: It flows southwest through the Assam Valley as Brahmaputra and south
through Bangladesh as the Jamuna. In the vast Ganges Delta, it merges with
the Padma, the popular name of the river Ganges in Bangladesh, and finally,
after merging with Padma, it becomes the Meghna and from here, it flows as
Meghna river before emptying into the Bay of Bengal.
• S3: The Lohit River or Zayu River is a river in India and China.
o It is a tributary to the Brahmaputra River.
o The river rises in Tibet Autonomous Region, in the Kangri Garpo range,
where it is known as the Zayü River.


Refer: Facts for Prelims: https://www.insightsonindia.com/2021/06/26/insights-daily-
current-affairs-pib-summary-26-june-2021/

Telegram: https://t.me/insightsIAStips
117
Youtube: https://www.youtube.com/channel/UCpoccbCX9GEIwaiIe4HLjwA
Revision Through MCQs (RTM) Compilation (June 2021)

230. ‘Maguri Motapung Beel Wetland’ is located in:


(a) Arunachal Pradesh
(b) Assam
(c) Meghalaya
(d) Manipur
Ans: (b)
Explanation:
• Maguri Motapung Beel is less than 10 km south of the more famous Dibru-
Saikhowa National Park and part of the Dibru-Saikhowa Biosphere Reserve.
• The wetland derives its name from ‘Magur’, local word for the catfish Clarius
batrachus, once found here in abundance. Motapung is a village nearby, and
Beel is the Assamese word for wetland.

Refer: Facts for Prelims: https://www.insightsonindia.com/2020/07/20/insights-daily-


current-affairs-pib-summary-20-july-2020/

RTM- REVISION THROUGH MCQS – 28th -Jun-2021

231. With reference to India’s Smart Cities mission, consider the following statements:
1. The Mission is operated as a Centrally Sponsored Scheme.
2. The Union Ministry of Urban Development is responsible for implementing the
mission in collaboration with the state governments.
Which of the given above statements is/are correct?
(a) 1 only
(b) 2 only
(c) Both 1 and 2
(d) Neither 1 nor 2
Ans: (c)
Explanation:
• S1: It is a Centrally Sponsored Scheme.
• S2: The Union Ministry of Urban Development is responsible for implementing
the mission in collaboration with the state governments of the respective cities.

Refer: https://www.insightsonindia.com/2021/06/28/india-smart-cities-awards-2020/

232. Consider the following statements about Pradhan Mantri Awas Yojana (URBAN):
1. It is a flagship Mission of Government of India being implemented by Ministry of
Housing and Urban Affairs (MoHUA).
2. It covers the entire urban area consisting of Statutory Towns and Notified Planning
Areas except Special Area Development Authorities.
Which of the given above statements is/are correct?
(a) 1 only
(b) 2 only
(c) Both 1 and 2
(d) Neither 1 nor 2
Ans: (a)

Telegram: https://t.me/insightsIAStips
118
Youtube: https://www.youtube.com/channel/UCpoccbCX9GEIwaiIe4HLjwA
Revision Through MCQs (RTM) Compilation (June 2021)

Explanation:
• S1: Pradhan Mantri Awas Yojana – Urban (PMAY-U), a flagship Mission of
Government of India being implemented by Ministry of Housing and Urban
Affairs (MoHUA), was launched on 25th June 2015.
• S2: The Mission covers the entire urban area consisting of Statutory Towns,
Notified Planning Areas, Development Authorities, Special Area Development
Authorities, Industrial Development Authorities or any such authority
under State legislation which is entrusted with the functions of urban planning
& regulations.

Refer: https://www.insightsonindia.com/2021/06/28/pradhan-mantri-awas-yojana-
urban-7/

233. The Gujarat Prohibition Act (1949) was in news recently, is related to which of the
following?
(a) prevention of begging in the State
(b) promotion and enforcement of alcohol prohibition in the State
(c) secure exclusive state control over forests
(d) none of the above
Ans: (b)
Explanation: The Gujarat Prohibition Act, 1949:
• Introduced by the then Bombay province as Bombay Prohibition Act, 1949 to
overhaul the law relating to intoxicating drugs and narcotics total prohibition.
• It is an Act relating to the promotion and enforcement of alcohol
prohibition in the Bombay State.
• The Bombay state was divided into the states of Maharashtra and Gujarat in
1960.
• Gujarat adopted the prohibition policy in 1960 and subsequently chose to
enforce it with greater rigidity.
• In 2011, it renamed the Act as Gujarat Prohibition Act.

Refer: https://www.insightsonindia.com/2021/06/28/the-gujarat-prohibition-act-1949/

234. Consider the following statements about African Swine Fever (ASF):
1. The organism which causes ASF is a DNA virus of the Asfarviridae family.
2. It is not a threat to human health and cannot be transmitted from pigs to humans.
3. As of now, there is no approved vaccine.
Which of the given above statements is/are correct?
(a) 1 and 3 only
(b) 2 and 3 only
(c) 1 and 2 only
(d) 1, 2 and 3
Ans: (d)
Explanation:
• S1: The organism which causes ASF is a DNA virus of the Asfarviridae family.
• S2: ASF is not a threat to human health and cannot be transmitted from
pigs to humans.
• S3: As of now, there is no approved vaccine, which is also a reason why
animals are culled to prevent the spread of infection.

Telegram: https://t.me/insightsIAStips
119
Youtube: https://www.youtube.com/channel/UCpoccbCX9GEIwaiIe4HLjwA
Revision Through MCQs (RTM) Compilation (June 2021)

Refer: https://www.insightsonindia.com/2021/06/28/african-swine-fever-2/

235. Consider the following statements about International Atomic Energy Agency:
1. It was set up as the world's "Atoms for Peace" organization in 1957 within the United
Nations family.
2. It inhibit the use of nuclear energy for any military purpose, including nuclear
weapons.
3. It works only with its Member States.
Which of the given above statements is/are correct?
(a) 1 and 2 only
(b) 1 only
(c) 2 only
(d) 2 and 3 only
Ans: (a)
Explanation:
• S2: The International Atomic Energy Agency is an international organization that
seeks to promote the peaceful use of nuclear energy, and to inhibit its use
for any military purpose, including nuclear weapons.
• S1: The IAEA is the world's centre for cooperation in the nuclear field. It was set
up as the world's "Atoms for Peace" organization in 1957 within the United
Nations family.
• S3: The Agency works with its Member States and multiple partners
worldwide to promote the safe, secure and peaceful use of nuclear technologies.

Refer: https://www.insightsonindia.com/2021/06/28/nuclear-site-images-wont-be-
shared-with-iaea-iran/

236. Recently, the Corporate Affairs Ministry has expanded the turnover and borrowing
thresholds for Small and Medium sized Companies (SMC). In this context, consider the
following statements:
1. The Corporate Affairs Ministry has increased the turnover threshold for SMCs to Rs
50 crore from Rs 10 crore.
2. SMCs are permitted to avail a number of exemptions under the Company
(Accounting Standards) Rules 2021 to reduce the complexity of regulatory filings for
smaller firms.
Which of the given above statements is/are correct?
(a) 1 only
(b) 2 only
(c) Both 1 and 2
(d) Neither 1 nor 2
Ans: (b)
Explanation:
• The Corporate Affairs Ministry has expanded the turnover and borrowing
thresholds for Small and Medium sized Companies (SMC).
• What is the change?
o The Corporate Affairs Ministry has increased the turnover threshold for
SMCs to Rs 250 crore from Rs 50 crore, and the borrowing threshold
to Rs 50 crore from Rs 10 crore.

Telegram: https://t.me/insightsIAStips
120
Youtube: https://www.youtube.com/channel/UCpoccbCX9GEIwaiIe4HLjwA
Revision Through MCQs (RTM) Compilation (June 2021)

o SMCs are permitted to avail a number of exemptions under the Company


(Accounting Standards) Rules 2021 to reduce the complexity of
regulatory filings for smaller firms.

Refer: https://www.insightsonindia.com/2021/06/28/how-small-and-medium-companies-
will-benefit-from-higher-thresholds/

237. Recently, the researcher’s from which of the following country have claimed to found
an ancient human skull that could belong to an altogether new species of humans?
(a) Israel
(b) Germany
(c) India
(d) China
Ans: (d)
Explanation: Dragon Man:
• Researchers from China have claimed to have found an ancient human skull
that could belong to an altogether new species of humans.
• The skull was found in north-east China’s Harbin city.
• It has been dubbed the “Dragon Man” or Homo longi, a name that has been
derived from the Long Jiang or Dragon river in the Heilongjiang province of
China where the city of Harbin is located.
• They could be over 146,000 years old.

Refer: facts for prelims: https://www.insightsonindia.com/2021/06/28/insights-daily-


current-affairs-pib-summary-28-june-2021/

238. Consider the following statements about Bengal monitor or common Indian monitor:
1. It is carnivorous and poisonous.
2. It is protected under Schedule I of WPA.
3. It is listed under the ‘Least Concern’ category on the International Union for
Conservation of Nature Red List.
Which of the given above statements is/are correct?
(a) 1 and 3 only
(b) 2 only
(c) 2 and 3 only
(d) 1, 2 and 3
Ans: (c)
Explanation:
• The Bengal monitor or common Indian monitor (Varanus bengalensis) is a large
lizard that is mainly terrestrial.
• The monitors are carnivorous and non-poisonous.
• The monitor lizard is protected under Schedule I of WPA but is regularly killed
for its meat, blood and oil.
• The species is listed under the ‘Least Concern’ category on the International
Union for Conservation of Nature Red List.

Refer: facts for prelims: https://www.insightsonindia.com/2021/06/28/insights-daily-


current-affairs-pib-summary-28-june-2021/

Telegram: https://t.me/insightsIAStips
121
Youtube: https://www.youtube.com/channel/UCpoccbCX9GEIwaiIe4HLjwA
Revision Through MCQs (RTM) Compilation (June 2021)

239. Consider the following statements about Bankim Chandra Chattopadhyay:


1. He is notable for his play Nil Darpan (1860).
2. He brought out a monthly magazine called Bangadarshan in 1872.
3. He wrote the national song Vande Mataram.
Which of the given above statements is/are correct?
(a) 1 and 2 only
(b) 2 and 3 only
(c) 1 and 3 only
(d) 1, 2 and 3
Ans: (b)
Explanation: Bankim Chandra Chattopadhyay:
• He was a Bengali poet and writer.
• He wrote the national song Vande Mataram.
• His novel Anandamath — which was set in the background of the Sanyashi
Bidroho (rebellion of monks in late 18th century) — is considered to be one of
key works on Bengal’s nationalism.
• His first Bengali fiction is called ‘Durgeshnondini’ published in 1865.
• He also wrote other famous novels like Kapalkundala in 1866, Mrinalini in
1869, Vishbriksha in 1873, Chandrasekhar in 1877, Rajani in 1877, Rajsimha
in 1881 and Devi Chaudhurani in 1884.
• He brought out a monthly magazine called Bangadarshan in 1872.
• His first fiction to appear in print was the English novel Rajmohan’s Wife.
• Nil Darpan or The Indigo Planting Mirror was a Bengali play written by
Dinabandhu Mitra in 1858-'59.

Refer: facts for prelims: https://www.insightsonindia.com/2021/06/28/insights-daily-


current-affairs-pib-summary-28-june-2021/

240. Consider the following statements:


1. Vande Mataram is a poem written in Bengali by Bankim Chandra Chatterjee.
2. The poem was first sung by Rabindranath Tagore in the 1896 session of the Indian
National Congress.
Which of the given above statements is/are correct?
(a) 1 only
(b) 2 only
(c) Both 1 and 2
(d) Neither 1 nor 2
Ans: (b)
Explanation:
• S1: Vande Mataram is a poem written in Sanskrit by Bankim Chandra
Chatterjee.
• The poem was first sung by Rabindranath Tagore in the 1896 session of the
Indian National Congress.

Refer: facts for prelims: https://www.insightsonindia.com/2021/06/28/insights-daily-


current-affairs-pib-summary-28-june-2021/

Telegram: https://t.me/insightsIAStips
122
Youtube: https://www.youtube.com/channel/UCpoccbCX9GEIwaiIe4HLjwA
Revision Through MCQs (RTM) Compilation (June 2021)

RTM- REVISION THROUGH MCQS – 29th -Jun-2021

241. Consider the following statements about Attorney General for India (AGI):
1. AGI is a part of the Union Executive.
2. AGI holds office during the pleasure of the President.
3. AGI has the right of audience in all Courts in India.
Which of the given above statements is/are correct?
(a) 1 and 2 only
(b) 2 and 3 only
(c) 1 and 3 only
(d) 1, 2 and 3
Ans: (d)
Explanation:
• S1: The Attorney General for India is the central government’s chief legal advisor,
and its primary lawyer in the Supreme Court of India. He is a part of the Union
Executive.
• S2: He is appointed by the President of India under Article 76(1) of the
Constitution and holds office during the pleasure of the President.
• S3: The Attorney General has the right of audience in all Courts in India as
well as the right to participate in the proceedings of the Parliament, though not
to vote.

Refer: https://www.insightsonindia.com/2021/06/29/attorney-general-3/

242. Consider the following statements about United Nations Office on Drugs and Crime:
1. It is a member of the United Nations Development Group.
2. It is mandated by its member States to assist in the implementation of both United
Nations Convention against Corruption (UNCAC) and United Nations Convention
against Transnational Organized Crime (UNTOC).
Which of the given above statements is/are correct?
(a) 1 only
(b) 2 only
(c) Both 1 and 2
(d) Neither 1 nor 2
Ans: (c)
Explanation:
• S1: It is a member of the United Nations Development Group.
• S2: The United Nations Office on Drugs and Crime (UNODC) is mandated by its
Member States to assist in the implementation of both United Nations
Convention against Corruption (UNCAC) and United Nations Convention
against Transnational Organized Crime (UNTOC), which along with the UN
Drug Conventions of 1961, 1971 and 1988 underpin all the operational work of
UNODC.

Refer: https://www.insightsonindia.com/2021/06/29/international-day-against-drug-
abuse-and-illicit-trafficking-2021/

Telegram: https://t.me/insightsIAStips
123
Youtube: https://www.youtube.com/channel/UCpoccbCX9GEIwaiIe4HLjwA
Revision Through MCQs (RTM) Compilation (June 2021)

243. In the context of contribution of PC Mahalanonobis to Statistics, consider the


following statements:
1. He is known as the father of Indian Statistics.
2. He established the Indian Statistical Institute (ISI) in 1931 in Bombay.
3. He helped in the establishment of Central Statistical Organisation (CSO) and the
National Sample Survey (NSS).
Which of the given above statements is/are correct?
(a) 1 and 2 only
(b) 2 and 3 only
(c) 1 and 3 only
(d) 1, 2 and 3
Ans: (c)
Explanation:
• S1: He is referred to as the chief architect of Indian statistical system as well as
father of statistical science in India.
• S2: He established the Indian Statistical Institute (ISI) in 1931 in Kolkata.
The institute founded the journal Sankhya, along the lines of Karl Pearson’s
Biometrika. In 1959 it was made an autonomous body of Ministry of Statistics &
Programme Implementation.
• S3: He also helped in the establishment of Central Statistical Organisation
(CSO), the National Sample Survey (NSS) and the Annual Survey of Industries
(ASI).

Refer: https://www.insightsonindia.com/2021/06/29/statistics-day-3/

244. The ‘Cyber Capabilities and National Power’ report is released by which of the
following?
(a) World Economic Forum
(b) International Telecommunication Union
(c) International Atomic Energy Agency
(d) International Institute for Strategic Studies
Ans: (d)
Explanation:
• The Cyber Capabilities and National Power Report was recently released by the
International Institute for Strategic Studies (IISS).
• The report analyses the cyber ecosystem of each state and how it intersects with
international security, economic competition and military affairs.

Refer: https://www.insightsonindia.com/2021/06/29/cyber-capabilities-and-national-
power-report-iiss/

245. Consider the following statements about World Intellectual Property Organization
(WIPO):
1. It is one of the 15 specialized agencies of the United Nations (UN).
2. It is responsible for administering and monitoring the operation of the TRIPS (Trade-
Related Aspects of Intellectual Property Rights) Agreement.
Which of the given above statements is/are correct?
(a) 1 only
(b) 2 only
(c) Both 1 and 2

Telegram: https://t.me/insightsIAStips
124
Youtube: https://www.youtube.com/channel/UCpoccbCX9GEIwaiIe4HLjwA
Revision Through MCQs (RTM) Compilation (June 2021)

(d) Neither 1 nor 2


Ans: (a)
Explanation:
• S1: The World Intellectual Property Organization (WIPO) is one of the 15
specialized agencies of the United Nations.
• S2: The Agreement on Trade-Related Aspects of Intellectual Property Rights
(TRIPS) is an international legal agreement between all the member nations of
the World Trade Organization (WTO).
o The TRIPS Council is responsible for administering and monitoring the
operation of the TRIPS Agreement.

Refer: https://www.insightsonindia.com/2021/06/29/us-digital-millennium-copyright-
act/

246. Consider the following statements:


1. The Baihetan Dam is the world’s second-biggest hydroelectric dam.
2. The dam is being constructed on Jinsha River, a tributary of the Mekong River.
Which of the given above statements is/are correct?
(a) 1 only
(b) 2 only
(c) Both 1 and 2
(d) Neither 1 nor 2
Ans: (a)
Explanation: Baihetan Dam:
• It is the world’s second-biggest hydroelectric dam under construction in
southwestern China.
• The dam is being constructed on Jinsha river, a tributary of the Yangtze.
• The 289-m-tall Baihetan Dam will have 16 generating units with a capacity of 1
million kilowatts each.

Refer: facts for prelims: https://www.insightsonindia.com/2021/06/29/insights-daily-


current-affairs-pib-summary-29-june-2021/

247. The term “2-deoxy-D-glucose” was in news recently, is related to which of the
following?
(a) anti-septic cream
(b) anti-Covid drug
(c) some cosmetic product
(d) agriculture pesticide
Ans: (b)
Explanation:
• Dr Reddy’s Laboratories has announced the commercial launch of drug 2-deoxy-
D-glucose (2-DG).
• It is an anti-Covid drug developed by the Defence Research and
Development Organisation (DRDO).
• The drug is approved for emergency use as an adjunct therapy to the standard of
care in the treatment of coronavirus patients in hospital settings.

Refer: facts for prelims: https://www.insightsonindia.com/2021/06/29/insights-daily-


current-affairs-pib-summary-29-june-2021/

Telegram: https://t.me/insightsIAStips
125
Youtube: https://www.youtube.com/channel/UCpoccbCX9GEIwaiIe4HLjwA
Revision Through MCQs (RTM) Compilation (June 2021)

248. With reference to ‘Electronic Vaccine Intelligence Network’ (eVIN), consider the
following statements:
1. It is an indigenously developed technology.
2. It monitors the temperature of the cold chain through a smartphone application.
3. It supports the central government’s Universal Immunization Programme.
Which of the given above statements is/are correct?
(a) 1 and 2 only
(b) 2 and 3 only
(c) 1 and 3 only
(d) 1, 2 and 3
Ans: (d)
Explanation:
• Electronic Vaccine Intelligence Network is an indigenously developed
technology that digitises vaccine stocks and monitors the temperature of the
cold chain through a smartphone application. The innovative eVIN was first
launched across 12 states in 2015 to support better vaccine logistics
management at cold chain points. eVIN supports the central government’s
Universal Immunization Programme by providing real-time information on
vaccine stocks and flows, and storage temperatures across all cold chain points
across states and UTs.
• CoWIN:
o CoWIN is an extension of an electronic vaccine intelligence network,
eVIN, which is used to collect real-time feedback of the vaccination
programmes.
o It is a cloud-based IT solution for planning, implementing, monitoring
and evaluating COVID-19 vaccination in the country.

Refer: facts for prelims: https://www.insightsonindia.com/2021/06/29/insights-daily-


current-affairs-pib-summary-29-june-2021/

249. Consider the following statements about Agni-P (Prime) missile:


1. It is a new-generation nuclear-capable ballistic missile.
2. It is a canisterised missile with range of more than 5000 km.
3. It has improved parameters including manoeuvring and accuracy compared to
earlier Agni class of missiles.
Which of the given above statements is/are correct?
(a) 1 and 2 only
(b) 2 and 3 only
(c) 1 and 3 only
(d) 1, 2 and 3
Ans: (c)
Explanation: Agni-P (Prime) missile:
• It is new-generation nuclear-capable ballistic missile.
• It was successfully test-fired by the Defence Research and Development
Organisation (DRDO).
• Agni-P is a new-generation advanced variant of Agni class of missiles. It is a
canisterised missile with range capability between 1,000 and 2,000 km.
• Canisterisation of missiles reduces the time required to launch the missile while
improving its storage and mobility.

Telegram: https://t.me/insightsIAStips
126
Youtube: https://www.youtube.com/channel/UCpoccbCX9GEIwaiIe4HLjwA
Revision Through MCQs (RTM) Compilation (June 2021)

• Agni-P has improved parameters including manoeuvring and accuracy compared


to earlier Agni class of missiles
• The longest of the Agni series, Agni-V, an Inter-Continental Ballistic Missile
(ICBM) with a range of more than 5,000 km, has already been tested several
times and validated for induction.

Refer: facts for prelims: https://www.insightsonindia.com/2021/06/29/insights-daily-


current-affairs-pib-summary-29-june-2021/

250. Consider the following statements:


1. Fukuoka Prize is given annually to distinguished people to foster and increase
awareness of Asian cultures.
2. Noted journalist P. Sainath has been selected as one of the three recipients of the
Fukuoka Prize for 2021.
Which of the given above statements is/are correct?
(a) 1 only
(b) 2 only
(c) Both 1 and 2
(d) Neither 1 nor 2
Ans: (c)
Explanation:
• Noted journalist P. Sainath has been selected as one of the three recipients of
the Fukuoka Prize for 2021.
• The Fukuoka Prize is given annually to distinguished people to foster and
increase awareness of Asian cultures, and to create a broad framework of
exchange and mutual learning among the Asian people.
• Eleven Indians have received the Fukuoka Prize so far.
• The prize was established in 1990.

Refer: facts for prelims: https://www.insightsonindia.com/2021/06/29/insights-daily-


current-affairs-pib-summary-29-june-2021/

RTM- REVISION THROUGH MCQS – 30th -Jun-2021

251. With reference to Parliamentary Privilege’s, consider the following statements:


1. According to the Constitution, the powers, privileges and immunities of Parliament
and MP's are to be defined by Committee of Privileges.
2. The Speaker of Lok Sabha or Rajya Sabha (RS) Chairperson is the first level of
scrutiny of a privilege motion.
Which of the given above statements is/are correct?
(a) 1 only
(b) 2 only
(c) Both 1 and 2
(d) Neither 1 nor 2
Ans: (b)
Explanation:
• S1: According to the Constitution, the powers, privileges and immunities of
Parliament and MP's are to be defined by Parliament. No law has so far been

Telegram: https://t.me/insightsIAStips
127
Youtube: https://www.youtube.com/channel/UCpoccbCX9GEIwaiIe4HLjwA
Revision Through MCQs (RTM) Compilation (June 2021)

enacted in this respect. In the absence of any such law, it continues to be


governed by British Parliamentary conventions.
• S2: The Speaker/RS chairperson is the first level of scrutiny of a privilege
motion.
o The Speaker/Chair can decide on the privilege motion himself or herself
or refer it to the privileges committee of Parliament.
o If the Speaker/Chair gives consent under relevant rules, the member
concerned is given an opportunity to make a short statement.

Refer: https://www.insightsonindia.com/2021/06/30/parliamentary-privileges-2/

252. Consider the following statements:


1. The ‘One Nation One Ration Card’ scheme is aimed at reducing the inter-state
migration and regional disparities in India.
2. It is being implemented under the Integrated Management of PDS (IMPDS.)
Which of the given above statements is/are correct?
(a) 1 only
(b) 2 only
(c) Both 1 and 2
(d) Neither 1 nor 2
Ans: (b)
Explanation:
• S1: ONORC was also launched with the purpose that no poor person should be
deprived of getting subsidised food grains under the food security scheme when
they shift from one place to another.
o The reform especially empowers the migratory population mostly
labourers, daily wagers, urban poor like rag pickers, street dwellers,
temporary workers in organised and unorganised sectors, domestic
workers etc, who frequently change their place of dwelling to be self
reliant in food security.
• S2: It is being implemented under the Integrated Management of PDS (IMPDS).

Refer: https://www.insightsonindia.com/2021/06/30/one-nation-one-ration-card-onorc/

253. Consider the following statements about the Drugs Controller General of India (DCGI):
1. It controls the prices of pharmaceutical drugs in India.
2. It is responsible for approval of licenses of specified categories of drugs in India.
Which of the given above statements is/are correct?
(a) 1 only
(b) 2 only
(c) Both 1 and 2
(d) Neither 1 nor 2
Ans: (b)
Explanation:
• S1: The National Pharmaceutical Pricing Authority (NPPA) is a government
regulatory agency that controls the prices of pharmaceutical drugs in India.
• S2: Under the Drug and Cosmetics Act, the regulation of manufacture, sale and
distribution of Drugs is primarily the concern of the State authorities while the
Central Authorities are responsible for approval of New Drugs, Clinical Trials in
the country, laying down the standards for Drugs, control over the quality of

Telegram: https://t.me/insightsIAStips
128
Youtube: https://www.youtube.com/channel/UCpoccbCX9GEIwaiIe4HLjwA
Revision Through MCQs (RTM) Compilation (June 2021)

imported Drugs, coordination of the activities of State Drug Control


Organisations and providing expert advice with a view of bring about the
uniformity in the enforcement of the Drugs and Cosmetics Act.
• Drug Controller General of India is responsible for approval of licenses of
specified categories of Drugs such as blood and blood products, I. V. Fluids,
Vaccine and Sera.

Refer: https://www.insightsonindia.com/2021/06/30/govt-gives-nod-for-cipla-to-import-
modernas-vaccine/

254. With reference to Organization of the Petroleum Exporting Countries (OPEC), consider
the following statements:
1. It is an intergovernmental organization of more than 25 Member countries.
2. It is headquartered in Vienna, Austria.
3. According to its statutes, OPEC membership is open to any country that is a
substantial exporter of oil and shares the ideals of the organization.
Which of the given above statements is/are correct?
(a) 1 and 2 only
(b) 2 and 3 only
(c) 1 and 3 only
(d) 1, 2 and 3
Ans: (b)
Explanation:
• S1 and S2: OPEC, which describes itself as a permanent intergovernmental
organization, was created in Baghdad in September 1960 by founding members
Iran, Iraq, Kuwait, Saudi Arabia, and Venezuela. The headquarters of the
organization are in Vienna, Austria, where the OPEC Secretariat, the executive
organ, carries out OPEC’s day-to-day business.
• S3: According to its statutes, OPEC membership is open to any country that is a
substantial exporter of oil and shares the ideals of the organization.

Refer: https://www.insightsonindia.com/2021/06/30/opec-and-rising-oil-prices/

255. With reference to Currency Swap Agreement (CSA), consider the following statements:
1. It is an arrangement between two friendly countries to involve in trading in their
own local currencies.
2. As per the arrangements, both countries pay for import and export trade at the
predetermined rates of exchange.
Which of the given above statements is/are correct?
(a) 1 only
(b) 2 only
(c) Both 1 and 2
(d) Neither 1 nor 2
Ans: (c)
Explanation: What is this Currency Swap Arrangement (CSA)?
• It is an arrangement between two friendly countries to involve in trading in their
own local currencies.
• As per the arrangements, both countries pay for import and export trade at the
predetermined rates of exchange, without bringing in third country currency like
the US Dollar.

Telegram: https://t.me/insightsIAStips
129
Youtube: https://www.youtube.com/channel/UCpoccbCX9GEIwaiIe4HLjwA
Revision Through MCQs (RTM) Compilation (June 2021)

Refer: https://www.insightsonindia.com/2021/06/30/lanka-banking-on-1-bn-india-swap-
deal/

256. Consider the following statements:


1. Recently, the National Ganga Council (NGC) has released an updated atlas of glacial
lakes that are part of the Ganga river basin.
2. The Atlas is available on Bhuvan portal of National Remote Sensing Centre (NRSC),
ISRO.
Which of the given above statements is/are correct?
(a) 1 only
(b) 2 only
(c) Both 1 and 2
(d) Neither 1 nor 2
Ans: (b)
Explanation:
• Ministry of Jal Shakti has released an updated atlas of glacial lakes that are
part of the Ganga river basin.
• About 4,707 glacial lakes have been mapped in the Ganga basin.
• Glacial lakes with water spread area greater than 0.25 ha were mapped using
Resourcesat-2 Linear Imaging Self Scanning Sensor-IV satellite data.
• The Atlas is available on Bhuvan portal of National Remote Sensing Centre
(NRSC), ISRO.

Refer: Facts for Prelims: https://www.insightsonindia.com/2021/06/30/insights-daily-


current-affairs-pib-summary-30-june-2021/

257. Consider the following pairs:


GI tags State
1. Kaji Nemu Odisha
2. Hmaram Mizoram
3. Khola Chilli Andhra Pradesh
4. Kadaknath Madhya Pradesh
Which of the above pairs is/are correctly matched?
(a) 2 and 4 only
(b) 1, 2 and 3 only
(c) 1 and 4 only
(d) 2, 3 and 4 only
Ans: (a)
Explanation:
• Kaji Nemu::Assam
• Khola Chilli:: Goa
• Kadaknath or Kali Masi is a breed of chicken originating from the Jhabua and
Dhar districts of western Madhya Pradesh.
o It received GI tag in 2017.
o It is popular for its black meat which is famous for its meat quality,
texture, taste, and excellent medicinal values.
o It is also famous for its low cholesterol and high protein content.

Refer: Facts for Prelims: https://www.insightsonindia.com/2021/06/30/insights-daily-


current-affairs-pib-summary-30-june-2021/

Telegram: https://t.me/insightsIAStips
130
Youtube: https://www.youtube.com/channel/UCpoccbCX9GEIwaiIe4HLjwA
Revision Through MCQs (RTM) Compilation (June 2021)

258. Consider the following pairs :


Place of Pilgrimage: Location
1. Srisailam : Nallamala Hills
2. Omkareshwar : Satmala Hills
3. Pushkar : Mahadeo Hills
Which of the above pairs is/are correctly matched?
(a) 1 only
(b) 2 and 3 only
(c) 1 and 2 only
(d) 1 and 3 only
Ans: (a)
Explanation:
• The town (as well as the mandal) of Srisailam is reputed for the shrine of Lord
Mallikarjuna on the flat top of Nallamala Hills.
• Omkareshwar is situated on the Mandhata hills in Madhya Pradesh, known in
ancient times as Shivpuri. The temple dates back to the Krita Yuga. River
Narmada flows here around the Mandhata hill in the form of an Om (Omkaram).
• The Mahadeo Hills are a range of hills in Madhya Pradesh state of central India.
But, Pushkar lies in Ajmer, Rajasthan.

Refer: UPSC CSE 2015

259. With reference to ‘dugong’, a mammal found in India, which of the following
statements is/are correct?
1. It is a herbivorous marine animal.
2. It is found along the entire coast of India.
3. It is given legal protection under Schedule 1 of the Wildlife (Protection) Act, 1972.
Select the correct answer using the code given below.
(a) 1 and 2
(b) 2 only
(c) 1 and 3
(d) 3 only
Ans: (c)
Explanation:
• The dugong is the only living representative of the once-diverse family
Dugongidae; its closest modern relative, Steller’s sea cow (Hydrodamalis gigas),
was hunted to extinction in the 18th century. The dugong is the only strictly
marine herbivorous mammal, as all species of manatee use fresh water to some
degree.
• In India, it is found off the Gujarat Coast (Gulf of Kutch), Tamil Nadu coast (Gulf
of Mannar), Palk Bay and Andaman and Nicobar Islands.

Refer: UPSC CSE 2015

260. In India, markets in agricultural products are regulated under the


(a) Essential Commodities Act, 1955
(b) Agricultural Produce Market Committee Act enacted by States
(c) Agricultural Produce (Grading and Marking) Act, 1937
(d) Food Products Order, 1956 and Meat and Food Products Order, 1973
Ans: (b)

Telegram: https://t.me/insightsIAStips
131
Youtube: https://www.youtube.com/channel/UCpoccbCX9GEIwaiIe4HLjwA
Revision Through MCQs (RTM) Compilation (June 2021)

Explanation:
• Agricultural Markets in most parts of the Country are established and regulated
under the State APMC Acts. The whole geographical area in the State is divided
and declared as a market area wherein the markets are managed by the Market
Committees constituted by the State Governments. Once a particular area is
declared a market area and falls under the jurisdiction of a Market Committee,
no person or agency is allowed freely to carry on wholesale marketing activities.

Refer: UPSC CSE 2015

Telegram: https://t.me/insightsIAStips
132
Youtube: https://www.youtube.com/channel/UCpoccbCX9GEIwaiIe4HLjwA

You might also like